Vous êtes sur la page 1sur 159

Histria da Matemtica atravs de Problemas

Volume 1 Mrio Olivero

UFF Instituto de Matemtica EB Centro de Estudos de Pessoal

Curso de Instrumentao para o Ensino da Matemtica

Apoio:

Fundao Cecierj / Consrcio Cederj


Rua Visconde de Niteri, 1364 Mangueira Rio de Janeiro, RJ CEP 20943-001 Tel.: (21) 2334-1569 Fax: (21) 2568-0725 Presidente Masako Oya Masuda Vice-presidente Mirian Crapez Coordenao do Curso de Matemtica UFF - Regina Moreth UNIRIO - Luiz Pedro San Gil Jutuca

Material Didtico
ELABORAO DE CONTEDO PRODUO GRFICA

Mrio Olivero
CAPA

Patricia Seabra

Eduardo Bordoni

O material constante desta disciplina, Histria da Matemtica atravs de Problemas, foi produzido sob o auspcio de Convnio de cooperao tcnico-acadmica entre o Exrcito Brasileiro e a Universidade Federal Fluminense.

O48h Olivero, Mrio. Histria da matemtica atravs de problemas / Mrio Olivero. Rio de Janeiro : UFF / CEP EB, 2007. 160p. (Curso de Instrumentao para o Ensino de Matemtica). ISBN: 85-98569-36-4 1. Matemtica Histria. 2. Matemtica Problemas. CDD - 510.09
Publicado por: Centro de Estudos de Pessoal (CEP) Copyright 2005 Centro de Estudos de Pessoal

Todos os direitos reservados ao Centro de Estudos de Pessoal (CEP) Praa Almte. Jlio de Noronha S/N - Leme - Tel (0xx21) 2275-0100 22010-020 Rio de Janeiro - Brasil

2009/2

Nenhuma parte deste material poder ser reproduzida, armazenada ou transmitida de qualquer forma ou por qualquer meio, seja ele eletrnico, mecnico, fotocpia ou gravao, sem autorizao do CEP e do autor.

Governo do Estado do Rio de Janeiro

Governador Srgio Cabral Filho

Secretrio de Estado de Cincia, Tecnologia e Inovao Alexandre Cardoso

Universidades Consorciadas

Apresentao ca
H alguns anos eu resolvi aprender a jogar tnis. Inscrevi-me em um curso e a e passei a receber duas aulas semanais. Foi uma tima experincia. Hoje eu o e jogo bastante bem, gosto muito do esporte e ganhei uma grande diverso. No a entanto, a experincia deu-me mais do que isso. e Sou professor de Matemtica h muito tempo e as aulas de tnis deram-me a a a e oportunidade de relembrar o outro lado da atividade. Por exemplo, recordei-me da importncia dos exerc a cios repetidos para assimilao de novos contedos. ca u Tive a felicidade de encontrar um professor que me deu ateno e encorajaca mento, corrigindo minhas (muitas) imperfeies com pacincia e bom humor. co e Reavaliei a importncia da auto-disciplina para atingir objetivos estabelecidos. a Com certeza, a experincia afetou minha vida prossional. Passei a ser mais e tolerante e generoso como professor. Aprender algo novo ajudou-me a levar em conta a perspectiva do aluno, que precisa ser encorajado, aprender a valorizar a organizao e a disciplina, precisa encontrar alegria na atividade de aprender ca e descobrir motivaes para atingir os objetivos que lhe so propostos. co a Voc se encontra numa posio semelhante ` minha, nesse momento. Est e ca a a prestes a vivenciar uma experincia de aprendizado. Retomar os estudos ree quer uma atitude corajosa. Parabns! Voc merece uma recepo calorosa. e e ca Nesta disciplina, Histria da Matemtica Atravs de Problemas, nosso principal o a e objetivo que voc goste ainda mais de Matemtica. e e a Aqui, voc ter a oportunidade de ver a Matemtica sob um novo prisma. e a a Perceber como as diferentes reas matemticas, tais como Algebra, Geometria, a a a Clculo, Anlise Matemtica e outras, se relacionam e se inuenciam, assim a a a como certas questes (os tais problemas da Matemtica) motivaram (e contio a 3

Apresentao ca

Histria da Matemtica o a

nuam motivando) novas descobertas. E bem provvel que voc passe a gostar a e ainda mais dos diversos temas com que lidamos na Matemtica, uma vez que a descobrir como eles surgiram e se desenvolveram. a E claro que num projeto como esse certas opes devem ser feitas. No poss co a e vel cobrir todos os temas, mesmo aqueles de maior importncia. A escolha daqueles a que representaro o nosso painel foram feitas em funo de meu gosto pessoal a ca (sim, todos ns temos nossas preferncias matemticas) assim como das minhas o e a muitas inabilidades. De qualquer forma, se o contedo apresentado motiv-lo u a a buscar ainda mais informaes sobre aquilo que cou faltando, despertando a co sua curiosidade, dar-me-ei por satisfeito. Essa experincia dever afetar, tambm, sua atuao prossional. Aps cure a e ca o sar a disciplina voc estar melhor preparado para apresentar os contedos de e a u Matemtica, colocando-os em um contexto histrico e mostrando suas conexes a o o com outros temas. Seja, portanto, bem-vindo ` Histria da Matemtica Atravs a o a e de Problemas!

Viso Geral da Disciplina a


Unidade 1: Trs Famosos Antigos Problemas e Assunto: Principais caracter sticas da Matemtica. Importncia dos problemas a a
e questes para seu desenvolvimento. o

Onde encontrar: Textos 1 e 2. Carga horria: 4h a Unidade 2: O Dilema de Pitgoras a Assunto: Matemtica do Antigo Egito e da Mesopotmia. Diferenas entre a a a c
Matemtica desses povos e a Matemtica grega. Os pitagricos. a a o

Onde encontrar: Textos 3 a 5. Carga horria: 6h a Unidade 3: Teoria das Propores de Eudoxo co Assunto: Primeira crise da Matemtica: segmentos no-comensurveis. Cona a a
ceito de innito. Soluo para a crise dada por Eudoxo. ca

Onde encontrar: Textos 6 a 9. Carga horria: 6h a Unidade 4: O Quinto Postulado da Geometria Euclideana Assunto: Elementos de Euclides. Problema do Quinto Postulado de Euclides.
Geometrias no-euclidianas. a

Onde encontrar: Textos 10 a 15. Carga horria: 7h a Unidade 5: Resoluo das Equaes Algbricas ca co e Assunto: Matemtica rabe e indiana. Surgimento do zero na Matemtica. a a a
Soluo para o problema das equaes cbicas. ca co u 7

Viso Geral a

Histria da Matemtica o a

Onde encontrar: Textos 16 a 19. Carga horria: 7h a Unidade 6: Uma Nova Matemtica para um Mundo Novo a Assunto: Panorama matemtico antes do Clculo. O Clculo segundo Newton a a a
e Leibniz.

Onde encontrar: Textos 20 a 23. Carga horria: 7h a Unidade 7: A Equao de Euler: ei + 1 = 0 ca Assunto: Caracter sticas da Matemtica do sculo 17 e 18. A Matemtica de a e a
Euler. Notao matemtica. ca a

Onde encontrar: 24 e 25. Carga horria: 7h a Unidade 8: Construo dos Nmeros Reais Cauchy e Dedekind ca u Assunto: Caracter sticas da Matemtica do m do sculo 18 e in do sculo a e cio e
19. A Matemtica de Gauss. Cortes de Dedekind. Contribuies de Cauchy e a co Weierstrass para a Anlise Matemtica. a a

Onde encontrar: 26 a 30. Carga horria: 6h a Unidade 9: Teoria de Conjuntos e Nmeros Transnitos de Cantor u Assunto: Caracter sticas da Matemtica no m do sculo 19 e in do sculo a e cio e
20. Contribuies feitas por Cantor e Hilbert. co

Onde encontrar: Textos 31 a 34. Carga horria: 6h a Unidade 10: Nmeros e Codicao de Mensagens u ca Assunto: Nmeros primos. Criptograa. u Onde encontrar: Textos 35 a 38. Carga horria: 4h a

Unidade 1
Trs Famosos Antigos Problemas e
Nesta unidade didtica, voc conhecer trs grandes problemas da antigidade a e a e u que desempenharam papis relevantes no desenvolvimento da Matemtica. e a Mas, primeiramente, vamos colocar certas coisas em perspectiva. Anal de contas, precisamos de algum tempo para nos conhecer melhor. Assim, antes de nos lanarmos nesta jornada, importante considerar a c e questo tratada em nosso primeiro texto. a

Texto 1: O que Matemtica? e a


Na verdade, pretendemos que voc pense um pouco sobre esse tema, que dee manda mais esforo do que podemos dispor em alguns minutos. Por exemplo, c h um livro de cerca de quinhentas pginas, escrito por Courant e Robbins, cujo a a t tulo , precisamente, O que Matemtica? e e a Veja, a Matemtica lida com duas idias fundamentais: multiplicidade e espao. a e c Desde os primrdios os seres humanos se valem desses conceitos. Contar as o reses de um rebanho ou os frutos de uma colheita, avaliar a rea de campo a de pastagem, de um campo a ser cultivado ou o volume de um vaso contendo gros so tarefas que demandam conceitos matemticos. a a a Dessa forma, podemos dizer que nmeros e guras geomtricas so elemenu e a tos fundamentais da Matemtica. Podemos at ensaiar uma resposta: a Maa e temtica a cincia dos nmeros e guras geomtricas, assim como as relaes a e e u e co que possam existir entre eles. Mesmo sentindo que a resposta contm parte da verdade, no podemos deixar de e a 9

UD 1

Histria da Matemtica o a

perceb-la incompleta. Nossa expectativa que, ao m do estudo da disciplina, e e voc possa ter constru sua prpria resposta para a questo. e do o a

1.1 Algumas caracter sticas da Matemtica a


Agora, mudando um pouco o foco da nossa ateno, observe que, apesar da ca diculdade que a maioria das pessoas tem para explicar o que a Matemtica, e a no muito dif detectar quando h matemtica em determinada situao. a e cil a a ca Quem nunca usou a expresso to certo quanto dois e dois so quatro? a a a E comum, no entanto, que as pessoas tenham uma viso parcial do que constitui a a Matemtica. Vejamos alguns aspectos que a caracterizam e distinguem das a demais cincias. e Geralmente, quando se trata de Matemtica, os nmeros so as primeiras coisas a u a mencionadas, no acha? Contudo, apesar da importncia que eles tm, a Maa a e temtica no lida apenas com nmeros, mas tambm com formas, assim como a a u e estuda as relaes entre esses objetos. co
O artista grco holands a e Maurits Cornelius Escher (1898 - 1970) aplicou em suas obras vrios truques de a iluso tica e perspectiva a o destorcida e repeties de co certos padres, conseguindo o assim um forte impacto visual. Entre seus temas favoritos esto a a metamorfose, a representao ca de innito e situaes co paradoxais.

Por exemplo, ao observarmos algumas gravuras de Escher, no podea mos deixar de notar a maneira como ele explora as simetrias e usa os padres, o que d um certo ar mao a temtico `s gravuras. Eis aqui uma a a de suas citaes: Para mim, permaco nece aberta a questo se (este traa balho) pertence ao mundo da matemtica ou da arte. a Muito bem, vamos aumentar nossa coleo de informaes sobre a Matemtica: ca co a ela lida com nmeros e formas, estuda padres e busca relaes entre seus u o co objetos. Enm, trata com uma multitude de idias, submetendo-as a diferentes e pontos de vista, comparando-as e buscando suas inter-relaes. co E como um cenrio onde uma enorme diversidade de atitudes, de perspectivas, se a opem e se inuenciam mutuamente. O particular ilustra o geral, o cont o nuo se ope ao discreto, per o odos em que a atitude mais formal prevalece se intercalam 10

UD 1

Histria da Matemtica o a

com per odos onde a intuio abre novos caminhos e assim por diante. ca Alm disso, Matemtica uma cincia que difere de todas as outras na forma e a e e como estabelece a verdade. A verdade cient ca, na Matemtica, estabelecida a e a partir de um conjunto m nimo de armaes, chamadas axiomas, por meio co de um conjunto de regras lgicas bem estabelecidas. E o chamado mtodo o e dedutivo. Nas outras cincias, a verdade estabelecida por experimentos cient e e cos. E por isso que, em muitos casos, uma nova teoria toma o lugar da anterior, que j no consegue explicar os fenmenos que prev. a a o e Basta comparar, s para se ter um exemplo, a evoluo histrica do conhecio ca o mento sobre o universo, em particular sobre o funcionamento do sistema solar, com a estabilidade vivida na Matemtica, simbolizada nos Elementos, de Eua clides, uma coleo de livros escritos em, aproximadamente, 250 a.C. Os Eleca mentos s tiveram menos edies do que a B o co blia, e so to corretos hoje como a a quando foram escritos.

1.2 A diversidade matemtica a


Um outro aspecto que chama a ateno sobre a Matemtica sua diversica a e dade. Em muitas l nguas, a palavra matemtica usada no plural. H tantas a e a ramicaes e sub-reas na Matemtica contempornea que praticamente co a a a e imposs acompanhar os desenvolvimentos mais recentes em todas as suas vel frentes de pesquisa. Essa caracter stica da Matemtica, ter uma face voltada para questes de cunho a o exclusivamente matemtico que costuma ser chamada de matemtica pura a a e outra voltada para os problemas surgidos nas outras cincias a matemtica e a aplicada a torna uma cincia cheia de surpresas. Para espanto at de muitos e e de seus criadores, teorias que nasceram no campo da matemtica pura, sem a nenhuma aparente aplicabilidade, podem encontrar seu caminho aplicado, e vice-versa. E como na msica, quando temas sacros e profanos so trocados. u a Finalmente, uma das caracter sticas da Matemtica, com a qual nos ocuparemos a agora, at o m desta unidade, a nsia de resolver problemas. Podemos dizer e e a at que se trata da principal atividade dos matemticos. Um matemtico feliz e a a e aquele que acabou de resolver um bom problema e ao fazer isso descobriu mais 11

UD 1 uma poro de novos problemas para pensar. ca

Histria da Matemtica o a

Voc ver que, ao longo do tempo, algumas questes desaaram a criatividade e a o de geraes de matemticos, norteando, estimulando a criao matemtica. co a ca a Essas questes funcionam como molas propulsoras, movendo as fronteiras do o conhecimento cada vez mais adiante, como no caso de Alexandre Grothendieck.

1.3 Um grande matemtico do sculo 20 a e


Grothendieck passou os anos de 1953 a 1955 na Universidade de So Paulo. a

Alexandre Grothendieck nasceu em 1928, em Berlim, e mudou-se para a Frana c em 1941. Seus trabalhos inovadores tiveram grande impacto em diversos campos da Matemtica, devido especialmente ao seu alto grau de abstrao. Em a ca 1966 recebeu a Medalha Fields, que assim como um Prmio Nobel da Mae e temtica. a O nmero de outubro de 2004 da revista Notices of the American Mathematical u Society traz um artigo sobre um dos mais relevantes matemticos do sculo 20, a e Alexandre Grothendieck. Nesse artigo aprendemos que os primeiros anos de Grothendieck, durante a Segunda Guerra, foram caticos e traumticos e sua formao educacional no o a ca a fora nada boa. No entanto, a atitude de enfrentar os problemas, as questes o da Matemtica, j estava presente. Ele escreveu suas memrias, intituladas a a o Rcoltes et Semailles (algo como Colheita e Semeadura), em que faz o seguinte e comentrio: a
O que menos me satisfazia, nos meus livros de matemtica [do liceu], era a a total ausncia de alguma denio sria da noo de comprimento [de uma e ca e ca curva], de rea [de uma superf a cie], de volume [de um slido]. Prometi a o mim mesmo preencher esta lacuna assim que tivesse uma chance.

Detectar a falta de preciso na denio desses conceitos, quando ainda era um a ca aluno do ensino mdio, uma prova da profunda percepo matemtica dessa e e ca a extraordinria pessoa. a Durante o estudo da disciplina, voc ir vericar como so relevantes as e a a preocupaes apontadas por Alexandre Grothendieck. Mas, est na hora de co a reetir um pouco sobre as idias expostas at aqui. e e Vamos ` primeira atividade. a 12

UD 1

Histria da Matemtica o a

Atividade 1
Para ajud-lo nessa tarefa, tente dar sua prpria resposta ` questo o que a o a a Matemtica? Guarde esta resposta para rel-la quando tiver completado o e a e estudo da disciplina. Faa, tambm, uma lista sucinta das caracter c e sticas da Matemtica apresentaa das no texto. Voc poderia acrescentar outras? e

Tudo que voc estudou at aqui constitui uma introduo para o tema do e e ca prximo texto: trs problemas famosos e antigos, diretamente relacionados a o e uma das caracter sticas mais valorizadas em um matemtico: a criatividade. a Essa caracter stica se manifesta, especialmente, na resoluo de problemas. ca Em muitos casos, a atitude de inconformismo diante das respostas dadas `s a antigas questes pelas geraes anteriores marcou o in da carreira de o co cio matemticos famosos, como teremos a oportunidade de ver no decorrer de a nossa jornada.

Texto 2: Trs Famosos Problemas e


O primeiro passo na resoluo de um problema consiste na sua correta forca mulao. Ou seja, para resolver um problema, melhor saber, precisamente, o ca e que devemos fazer e do que dispomos para chegar ` soluo. a ca Os trs problemas clssicos da Geometria grega so sobre como realizar uma e a a construo geomtrica usando apenas rgua e compasso. Veja seus enunciados: ca e e

Trisseco do ngulo: ca a
Dado um ngulo, construir um outro ngulo com um tero de sua a a c amplitude.

Duplicao do cubo: ca
Dado um cubo, construir outro cubo com o dobro do volume do anterior.

Quadratura do c rculo:
Dado um c rculo, construir um quadrado com a mesma rea. a 13

UD 1

Histria da Matemtica o a

Antes de falarmos sobre eles, vamos entender o signicado da expresso consa truo com rgua e compasso. ca e Note que esses problemas esto colocados no contexto da geometria formulada a por Euclides. Por isso, as construes com rgua e compasso so tambm conheco e a e cidas por construes euclidianas, apesar de os termos rgua e compasso co e no aparecerem nos livros de Euclides. a Assim, as solues deveriam obedecer apenas certos procedimentos, por assim co dizer, seguir regras muito bem estabelecidas. Veja, na teoria euclidiana, a rgua pode ser usada para construir um segmento, e to longo quanto se queira, que contenha dois pontos dados. Em particular, a essa rgua no graduada. Ou seja, no podemos utiliz-la para medir. e a e a a J o compasso pode ser usado para construir a circunferncia de centro em um a e dado ponto A e que passa por um dado ponto B. Assim, o compasso deve ter pernas to compridas quanto precisarmos. a Procure, ento, realizar as atividades a seguir. a

Atividade 2
Usando rgua e compasso, construa a mediatriz de um segmento dado. Voc e e sabe dividir um segmento dado em uma outra proporo qualquer, assim como ca 2 por 3? Construa tambm um tringulo equiltero. Voc poderia construir mais um e a a e pol gono regular, usando apenas rgua e compasso? e Quais pol gonos regulares podem ser constru dos com rgua e compasso? e Vamos, agora, falar um pouco do primeiro problema.

2.1 A trisseco do ngulo ca a


Repare que, usando rgua e compasso, no dif construir a bissetriz de e a e cil um ngulo dado. Basta construir uma circunferncia com centro no vrtice a e e do ngulo, marcando sobre os lados do ngulo, dois pontos (eqidistantes do a a u vrtice). Em seguida, construindo dois c e rculos de mesmo raio, com centros 14

UD 1

Histria da Matemtica o a

nos respectivos pontos obtidos nos lados do ngulo, determine o ponto que, a juntamente com o vrtice original, dene a reta bissetriz. e Tal sucesso encoraja a considerao do prximo passo: dividir o ngulo em ca o a trs partes iguais. Para certos ngulos espec e a cos, o problema tem soluo. ca Como poss construir, com rgua e compasso, um tringulo eqiltero, e vel e a u a podemos construir um ngulo de trinta graus, que divide o ngulo reto em trs a a e partes iguais. No entanto, o problema proposto nos pede para estabelecer um procedimento que funcione para qualquer ngulo dado. a Muitas tentativas de soluo para o problema foram dadas, mas cada uma ca delas apresentava uma falha. Tambm, pudera, o problema no tem soluo. e a ca Voc deve notar que demonstrar que no h uma soluo (que sirva para um e a a ca ngulo dado qualquer) uma tarefa muito dif a e cil. O problema era conhecido dos antigos gregos e a resposta (negativa) s foi obtida no sculo XIX, pelo o e francs Pierre Laurent Wantzel. Apesar da genialidade de Wantzel, preciso e e dizer que sua soluo depende de conceitos algbricos desenvolvidos ao longo ca e de vrios sculos, por vrias geraes de matemticos. Ou seja, o problema s a e a co a o foi solucionado quando se mudou o foco da questo, passando-se a buscar uma a prova de que no tem soluo. a ca Wantzel foi, ainda, o responsvel pela resposta, tambm negativa, a outro a e problema: a duplicao do cubo. ca

2.2 A duplicao do cubo ca


Nesse caso, a situao mais radical. Enquanto certos ngulos especiais podem ca e a ser divididos em trs, usando rgua e compasso, apesar de no haver um mtodo e e a e geral que sirva para um ngulo genrico, no se pode duplicar qualquer cubo. a e a Veja, o problema o seguinte: dado um cubo (ou seja, conhecendo o lado de e um dado cubo) devemos construir, com rgua e compasso, um cubo que tenha, e exatamente, o dobro de seu volume. Novamente, os gregos conheciam uma verso simples do problema. Scrates foi a o um dos mais originais pensadores de que temos not cia, mas tudo que sabemos de sua obra nos foi legado por Plato, que estudara com ele. Apesar de no ter a a sido um matemtico, Scrates retratado em um dos dilogos de Plato, numa a o e a a conversa com Mnon sobre a virtude, ensinando um jovem e inculto escravo a e 15

UD 1 duplicar um quadrado.

Histria da Matemtica o a

Isto , dado um quadrado, construir com rgua e compasso um novo quadrado e e que tenha o dobro de sua rea. Na primeira tentativa, o jovem dobra o lado a do quadrado dado e Scrates o faz ver o erro cometido. Em seguida, Scrates o o mostra-lhe a gura de um quadrado com os pontos mdios de seus lados unidos e por segmentos que formam um quadrado menor. Ento, Scrates ajuda o rapaz a o a lembrar-se que a rea do quadrado constru sobre a diagonal do quadrado a do menor tem o dobro de sua rea. a Finalmente, o ultimo problema consiste em construir com rgua e compasso e um quadrado de rea igual ` de um c a a rculo dado. Por isso o nome quadratura do c rculo.

2.3 A quadratura do c rculo


Novamente, o problema s foi resolvido muito tempo depois de ter sido proo posto. Em 1882, o matemtico alemo Ferdinand von Lindemann demonstrou a a a impossibilidade de efetuar a quadratura do c rculo usando apenas construes co com rgua e compasso. e Como isso foi feito? Apostamos que voc est curioso. Realmente, vamos e a gastar o tempo que nos resta desta unidade para que voc tenha ao menos e uma idia geral dos argumentos dados por Wentzel e por Lindemann. e
Carl Friedrich Gauss (1777 1855) Seu lema era: Pouca sed matura, algo como Pouco, porm maduro. e Suas contribuies cobrem co quase todas as reas da a Matemtica, como a Geometria, Teoria de Nmeros e Anlise Complexa. u a Foi tambm f e sico e astrnomo. O primeiro o problema importante que ele resolveu, aos 19 anos, foi a descoberta de uma construo com rgua e ca e compasso de um pol gono de 17 lados. Veja, desde o per odo clssico da a Matemtica na Grcia antiga, a e os unicos pol gonos regulares que podiam ser constru dos com rgua e compasso eram e o tringulo, o quadrado e o a pentgono. a

2.4 Algebrizao dos problemas geomtricos ca e


Como podemos demonstrar que imposs efetuar uma determinada conse vel truo com rgua e compasso? Veja que, para mostrar que uma certa consca e truo poss ca e vel, basta faz-la. e O caminho para as demonstraes foi aberto por um dos maiores matemticos co a de todos os tempos: Carl Friedrich Gauss. A idia algebrizar o problema. Por e e exemplo, para duplicar o quadrado de lado 1, devemos construir um quadrado de lado 2. Mas, como podemos construir o quadrado de lado 1, temos a sua diagonal que mede 2. O que Wantzel conseguiu provar, a partir das idias de Gauss, foi: e 16

UD 1

Histria da Matemtica o a

Se C um ponto obtido por uma construo com rgua e compasso a partir de e ca e dois pontos dados A e B, ento o quociente q das distncias AC e AB tem as a a seguintes propriedades: 1. q a raiz de um polinmio com coecientes inteiros, no todos nulos. e o a Nesse caso q chamado de um nmero algbrico. e u e 2. Se p(x) for um polinmio de grau m o nimo entre todos os polinmios com o coecientes inteiros, no todos nulos, dos quais q uma raiz, ento o a e a grau de p(x) uma potncia de 2. e e Veja como a duplicao do quadrado se encaixa nesse esquema. O nmero ca u 2

pode ser constru com rgua e compasso, pois a diagonal de um quadrado do e e de lado 1. Realmente, 2 uma das ra do polinmio x2 2, de coecientes e zes o inteiros. Ora, para duplicarmos o cubo de lado 1, ter amos que construir um segmento 3 de comprimento 2. Esse nmero algbrico raiz do polinmio x3 2, que u e e o tem grau (m nimo) 3, que no uma potncia de 2. a e e No caso do problema da diviso de um ngulo em trs partes iguais, a algea a e brizao do problema tambm resulta em uma equao cbica. ca e ca u Na questo da quadratura do c a rculo (de raio 1, por exemplo,) ter amos que construir com rgua e compasso um quadrado cujo lado medisse . Lindee mann mostrou que no raiz de nenhuma equao polinomial com coecientes a e ca inteiros no todos nulos. Ou seja, no um nmero algbrico e, portanto, a a e u e tambm no um nmero algbrico. e a e u e Estamos chegando ao m dessa unidade. Esperamos que sua curiosidade a respeito dos diversos aspectos da Matemtica tenha sido aguada e a c apresentamos mais duas atividades.

Atividade 3
Faa uma lista sucinta das diferentes caracter c sticas da Matemtica apresentaa das nessa unidade e acrescente algumas por voc mesmo. e

17

UD 1

Histria da Matemtica o a

Atividade 4
Na sua opinio, o que teria incomodado tanto o matemtico Grothendieck na a a leitura dos livros de Matemtica em que estudou? Se estivesse estudando nos a livros que ns usamos hoje em dia, ele teria uma opinio diferente? o a A histria desses trs problemas clssicos da Matemtica mostra como, em o e a a muitos casos, a importncia no est na resposta de uma certa questo, seja a a a a ela positiva ou no, mas nos mtodos usados para chegar at ela. No a e e a devemos nos decepcionar com o fato de no podermos duplicar o cubo, por a exemplo, pois a profundidade e a riqueza das idias desenvolvidas para chegar e ` resposta negativa nos compensam largamente. a E nossa expectativa, tambm, que o enfoque sobre os problemas passe a fazer e parte de sua maneira de ver a Matemtica, pois nisso consiste, em enorme a parte, a sua vitalidade e importncia. a

18

Unidade 2
O Dilema de Pitgoras a
Nesta unidade didtica voc descobrir como a Matemtica tornou-se uma a e a a cincia, no sentido de ter uma maneira bem denida de se e estabelecer a verdade. Isso ocorreu com o surgimento da cultura grega, por volta de 600 a.C., quando os primeiros matemticos de que temos not passaram a fazer e responder a cia perguntas que comeam com por que, alm daquelas que c e comeam com como. c No entanto, aps os primeiros triunfos dessa jovem fora criativa, surgiu uma o c grande crise, conhecida como o dilema de Pitgoras. a

Texto 3: A Matemtica dos Esticadores de Cordas a


Para entender melhor essa histria, voc precisa conhecer um pouco o contedo o e u matemtico produzido pelos povos que habitavam as margens do rio Nilo, na a Africa, e a regio entre os rios Tigre e Eufrates, no Oriente Mdio, cujas culturas a e antecederam a grega e, certamente, a inuenciaram. Uma das mais fascinantes civilizaes antigas de que temos not desenvolveuco cia se `s margens do rio Nilo, no norte da Africa a civilizao eg a ca pcia. Todos ns sabemos como foi requintada a cultura desse povo que adorava o gatos, constru pirmides monumentais para enterrar seus reis embalsamados a a os faras que eles acreditavam serem descendentes de seus deuses. o Entre tantas coisas dignas de nota a respeito dos antigos eg pcios est o fato de a eles terem desenvolvido uma forma de escrita os hierglifos deixando-nos, o 19

UD 2
No primeiro volume da coleo Mar de Histrias, ca o uma antologia do conto mundial, organizada por Aurlio Buarque de Holanda e e Paulo Rnai, editado pela o Nova Fronteira, h um conto a eg pcio, chamado A histria o de Rampsinitos.

Histria da Matemtica o a

assim, relatos e registros de suas conquistas culturais. Aristteles acreditava que a atividade matemtica surgira no Egito, criada por o a seus sacerdotes, uma vez que eles dispunham de tempo ocioso. Por mais interessante que seja essa possibilidade, devemos levar em conta a verso dada por Herdoto, chamado de pai da Histria. Ele armava que a a o o Geometria havia sido inventada no Egito, devido `s cheias anuais do rio Nilo, a que fertilizavam suas margens, o que era timo para a agricultura. No entanto, o quando as guas retornavam ao leito normal do rio, as marcaes dos terrenos a co precisavam ser refeitas. E por isso que Demcrito, lsofo grego que teria visitado o Egito, chamava o o os matemticos locais de esticadores de cordas, que eles usavam para fazer a as demarcaes. Veja que geometria formada pelas palavras gregas geo, que co e quer dizer terra, e metria, que quer dizer medida.

3.1 Revelao de obscuros segredos ca


H duas importantes fontes de informaes sobre o tipo de matemtica pratia co a cada no antigo Egito. So dois papiros, conhecidos como Papiro de Moscou, a que data de 1850 a.C., e Papiro Rhind, de 1650 a.C., que se encontra no Museu Britnico. Apesar do Papiro Rhind iniciar com a promessa de apresentar ao a leitor um estudo completo de todas as coisas, conhecimento de tudo o que existe, revelao dos mais obscuros segredos, os dois so colees de proca a co blemas resolvidos, todos relativamente simples. Os estudantes desses papiros aprenderiam com os exemplos a calcular a quantidade de tijolos usados para construir uma rampa de um dado tamanho ou a quantidade de cestos de pes a sucientes para alimentar os escravos necessrios para executar uma certa tarefa a e assim por diante. Apesar da propaganda um pouco enganosa, esses papiros cumpriam um papel essencial na transmisso dos conhecimentos. Os antigos a eg pcios conheciam a importncia dos exemplos na aprendizagem. a O Papiro Rhind nos revela como eles dividiam, extra am ra zes quadradas, resolviam problemas equivalentes a equaes lineares, lidavam com progresses co o aritmticas. Eles usavam 3.16 como uma aproximao de . O problema 14 e ca deste documento ensina a calcular o volume de um tronco de pirmide. a 20

UD 2

Histria da Matemtica o a

3.2 Um exemplo da aritmtica eg e pcia


Particularmente interessante a maneira como eles efetuavam o produto de e dois inteiros. Para multiplicar, por exemplo, 19 por 42, usamos o algoritmo da multiplicao baseado no sistema numrico posicional. Os eg ca e pcios usavam outra coisa, uma vez que no dispunham de tal facilidade. O mtodo deles se a e baseia no fato que multiplicar e dividir por 2 relativamente simples. e Comeamos dispondo os dois nmeros a serem multiplicados, um ao lado do c u outro, e constru mos uma tabela com duas colunas de nmeros. Veja a seguir. u Para obter a coluna da esquerda, basta seguir dobrando o nmero anterior a u cada nova linha. Na coluna da direita, fazemos o contrrio, dividindo o nmero a u por dois e colocando-o na linha de baixo, subtraindo 1 antes da diviso nos a casos em que ele for mpar. Nestes casos, fazemos uma pequena marca para destacar aquelas linhas das demais. Vamos aprender atravs de um exemplo, como faziam os antigos escribas eg e pcios. Comeamos colocando os nmeros 42 e 19 na primeira linha da tabela e como c u 19 um nmero e u mpar, a destacaremos com uma marca. 42 19 /

Para construir a segunda linha, dobramos o nmero 42 e colocamos o resultado, u 84, logo abaixo dele. Subtraindo 1 de 19 e dividindo por 2, obtemos 9 (= 18/2), que colocamos abaixo dele. Essa segunda linha tambm marcada, uma vez e e que 9 e mpar. 42 84 19 9 / /

Prosseguimos assim at obter 1 na coluna da direita. Veja, a seguir, a tabela e completa. 42 84 168 336 672 21 19 9 4 2 1 / /

UD 2

Histria da Matemtica o a

Finalmente, para obter o resultado do produto, basta somar os nmeros da u coluna da esquerda correspondentes `quelas linhas que foram marcadas: a 19 42 = 42 + 84 + 672 = 798. Parece mgica, mas no . Esse algoritmo de multiplicao se baseia na exa a e ca panso do multiplicando na base 2. a Note que 19 = 16 + 2 + 1 = 24 + 21 + 20 . Isto , as marcas indicam e precisamente as correspondentes potncias de 2 que aparecem na composio do e ca nmero. Veja, se considerarmos apenas a segunda coluna da tabela, trocarmos u a marca por 1 e colocarmos 0 na sua ausncia, obtemos a expanso de 19 na e a 19 9 4 2 1 1 1 0 0 1 base 2. Para isso, basta dispor os d gitos obtidos, da direita para a esquerda, escrevendo na horizontal essa nova coluna. Veja a expanso de 19 na base 2: a (10011)2 . Usamos o ndice 2 para distinguir esse nmero (19 na base dois) de 10 011 (dez u mil e onze). Se multiplicarmos um nmero por uma soma de potncias de 2, claro que o u e e resultado ser a soma das correspondentes dobras, dobras de dobras e assim por a diante, at completar o resultado. e Apresentamos uma atividade para voc praticar. e

Atividade 5
Na tabela a seguir, complete as colunas e efetue o produto de 26 por 31 usando o algoritmo apresentado. 31 62 124 26 13

Note que a expanso de 26 na base 2 (11010)2 , pois 26 = 16 + 8 + 2 = a e (1 24 ) + (1 23 ) + (0 22 ) + (1 21 ) + (0 20 ). Realmente, o primeiro 22

UD 2

Histria da Matemtica o a

d gito da direita para a esquerda na expanso de 26 na base 2 zero, indicando a e que 1 = 20 no faz parte das parcelas, uma vez que 26 par. a e E interessante notar que o algoritmo conhecido pelos eg pcios nos ensina a obter a expanso de um dado nmero na base 2. Usar apenas dois d a u gitos, 0 e 1, para denotar os nmeros, a base da construo dos nossos modernos computadores. u e ca

3.3 Os eg pcios e o Teorema de Pitgoras a


Os eg pcios sabiam, ` sua maneira, o reverso do Teorema de Pitgoras: a a Se os lados a, b e c de um tringulo satisfazem a relao a ca a2 + b2 = c2 , ento o tringulo retngulo. a a e a Para obter um ngulo reto, to necessrio para suas construes, eles usaa a a co vam doze pedaos de corda de mesmo comprimento amarrados uns aos outros, c formando um lao. Esticando propriamente esse lao obtinham um tringulo c c a retngulo de lados 3, 4 e 5 e o desejado ngulo. Veja a gura a seguir. a a
s s s s s s s s s s s s s s s s s s s s s s s s

No entanto, os eg pcios no deixaram nenhum registro de como chegaram ` a a concluso de que o tringulo de lados 3, 4 e 5 retngulo, nem est claro que a a e a a eles conheciam outros tringulos com a mesma propriedade. a Os matemticos que viveram na regio chamada Mesopotmia, entre os rios a a a Tigre e Eufrates, chegaram mais longe do que seus colegas eg pcios em suas descobertas matemticas. A palavra Mesopotmia vem do grego e a a signica entre rios. 23

UD 2

Histria da Matemtica o a

Texto 4: Nos Jardins Suspensos da Babilnia o


Assim como no caso do Nilo, os rios Tigre e Eufrates garantiam a fertilidade na Mesopotmia, favorecendo o desenvolvimento da agricultura e o surgimento de a civilizaes. Mas, enquanto as margens do Nilo foram ocupadas apenas pelos co eg pcios, a Mesopotmia foi habitada por vrios povos. Entre eles destacamos: a a sumrios inventaram a primeira escrita de que temos not chamada e cia cuneiforme, registrada em tabuletas de barro; babilnios criaram o primeiro conjunto de leis, o Cdigo de Hamurabi. o o Os sumrios foram absorvidos pelos babilnios, que tiveram sua fase mais dee o senvolvida por volta de 575 a.C., durante o reinado de Nabucodonossor.

4.1 As triplas babilnicas o


As conquistas matemticas desses povos caram registradas em tabuletas de a argila. A mais famosa conhecida como Plimpton 322, na qual est registrada e a uma fam de pares de nmeros, que geram triplas pitagricas. Uma tripla lia u o pitagrica formada por trs nmeros inteiros a, b e c, tais que c2 = a2 + b2 . o e e u Por exemplo, os dois primeiros nmeros dessa tabuleta so 119 e 169. Realu a mente, juntos com 120 eles geram um tringulo retngulo. a a 1692 = 28 561 = 14 161 + 14 400 = 1192 + 1202 . Portanto, (119, 120, 169) uma tripla pitagrica. e o A tabuleta Plimpton 322 revela uma cultura matemtica mais rica do que a a eg pcia. Os matemticos da Mesopotmia usavam um sistema numrico de a a e base 60 (o nosso sistema decimal, de base 10), bastante adequado ao estudo e da astronomia, que eles conheciam muito bem. O uso do sistema numrico e sexagesimal foi passado para a cultura grega que o preservou, pelo menos, nessa rea. E por isso que dividimos o c a rculo em 360 graus, a hora em 60 minutos, e assim por diante. Esses povos sabiam que os nmeros u 2uv, u2 v 2 e u2 + v 2 24

UD 2 geram uma tripla pitagrica, pois o

Histria da Matemtica o a

(u2 + v 2 )2 = (u2 v 2 )2 + (2uv)2 . Isto , se tomarmos u = 12 e v = 5, obtemos a tripla pitagrica (122 52 , 2 e o 125, 122 +52 ) = (119, 120, 169). No entanto, eles usavam essa tcnica apenas e para nmeros u e v relativamente primos e tais que seus fatores primos fossem u apenas 2, 3 ou 5, os fatores primos de 60.

Atividade 6
Usando u = 64 = 26 e v = 27 = 33 , gere a segunda tripla pitagrica que o aparece em Plimpton 322. Sabendo que 4601 e 6649 so nmeros na prxima tripla em Plimpton 322, a u o descubra os correspondentes geradores u e v, assim como o terceiro nmero. u No in do sculo 6 a. C., a cidade de Mileto, na Jnia, assistiu ao cio e o surgimento de uma nova cultura, que dominaria o mundo por aproximadamente mil anos e inuenciaria a maneira de pensar e produzir cincia at os nossos dias. e e

Texto 5: O Surgimento da Matemtica Grega a


Enquanto eg pcios e babilnios armazenavam conhecimentos e os transmitiam o `s novas geraes sem maiores questionamentos, os matemticos gregos pasa co a saram a buscar razes para explicar os resultados. Alm disso, uniram a essa o e atitude o rigor lgico, que pautava toda sua atitude cient o ca. Foi uma mudana c extraordinria. a Outra caracter stica introduzida pelos gregos foi a personalizao da Matemtica. ca a Tales de Mileto o primeiro matemtico de que temos not e a cia. Ele deve ter sido um personagem muito especial. As muitas lendas e histrias associadas ao o seu nome atestam isso.

5.1 As contribuioes de Tales de Mileto c


Os principais resultados matemticos associados a ele so: a a 25

UD 2 todo dimetro divide o c a rculo em duas partes;

Histria da Matemtica o a

os ngulos da base de um tringulo issceles so iguais; a a o a ngulos opostos pelo vrtice, formados por duas retas que se intersectam, a e so iguais; a dois tringulos com dois ngulos e um lado iguais so congruentes; a a a todo ngulo inscrito num semic a rculo reto. e Esse ultimo resultado conhecido como Teorema de Tales. Na verdade, to e dos esses fatos eram conhecidos dos matemticos das outras culturas que o a antecederam. No entanto, cabe a ele o mrito de ter providenciado suas dee monstraes. co Veja, no caso de Teorema de Tales, considere o ngulo de vrtice em B, inscrito a e no semic rculo ABC. Usando um segmento auxiliar que liga B ao centro do semic rculo, dividimos ABC em dois tringulos issceles, pois OA, OB e OC a o so raios do semic a rculo. Veja a ilustrao. ca

q O

Usando o resultado sobre tringulos issceles, sabemos que os ngulos denotados a o a por e por so iguais. Como a soma dos ngulos internos do tringulo ABC a a a igual a dois ngulos retos, obtemos e a + + ( + ) = 2 ngulos retos. a Portanto, + um ngulo reto, como quer e a amos demonstrar. Tales tambm deu contribuies ` Filosoa. e co a 26

UD 2

Histria da Matemtica o a

5.2 Pitgoras e o seu teorema a


A gura mais lembrada desse per odo inicial da matemtica grega foi, sem a dvida, Pitgoras. Entre os vrios resultados atribu u a a dos a ele, o mais famoso o Teorema de Pitgoras, sobre tringulos retngulos. Como voc pode ver, e a a a e esse resultado era, essencialmente, conhecido pelas culturas que o antecederam, mas atribu a ele sua primeira demonstrao. Conta a histria que Pitgoras e da ca o a teria sacricado cem bois quando descobriu a prova do teorema. Dif de crer, cil uma vez que Pitgoras teria sido vegetariano. De qualquer forma, no sabemos a a exatamente qual foi a demonstrao de Pitgoras. A demonstrao ocial, ca a ca digamos assim, apresentada no livro 1 dos Elementos, de Euclides, o pice do e a livro, que parece ter sido escrito para apresent-la. a

5.3 Uma demonstrao do Teorema de Pitgoras ca a


Voc conhece a demonstrao baseada na identidade algbrica (a + b)2 = a2 + e ca e 2ab + b2 ? Bem, aqui est ela. a Primeiro, observe que essa identidade pode ser demonstrada pelo diagrama a seguir.

Note que (a + b)2 a rea do quadrado de lado a + b, que est dividido em e a a dois quadrados de lados a e b e em quatro tringulos retngulos agrupados dois a a a dois em dois retngulos de lados a e b. a A rea de cada um dos quatro tringulos a a e ab . Assim, a rea do quadrado a 2 maior igual ` soma das reas dos dois quadrados menores com a rea dos e a a a 27

UD 2 quatro tringulos: a (a + b)2 = a2 + b2 + 4 ab 2

Histria da Matemtica o a

= a2 + 2ab + b2 .

Agora, um novo arranjo dos tringulos dentro do quadrado maior revela em seu a interior um quadrado de lado c, a hipotenusa do tringulo retngulo de catetos a a a e b.

Esse diagrama nos diz que (a + b)2 = 2ab + c2 . Reunindo ambas informaes, obtemos (a + b)2 = a2 + b2 + 2ab = c2 + 2ab. co Portanto, a2 + b2 = c2 . Isso conclui a demonstrao. ca Pitgoras fundou uma irmandade cujos interesses iam alm da Matemtica. a e a Os membros dessa irmandade atribu todas as suas descobertas a ele. am Apresentamos, a seguir, uma lista de algumas de suas descobertas, alm da e demonstrao do Teorema de Pitgoras. ca a

5.4 Outras contribuies dos pitagricos co o


Estudo das mdias aritmtica e e a+b 2ab , geomtrica ab e harmnica e o , 2 a+b assim como as relaes entre elas. co 28

UD 2

Histria da Matemtica o a

Estudo dos nmeros perfeitos e dos pares de nmeros amigveis. Chamau u a mos (m, n) um par de nmeros inteiros positivos de nmeros amigveis, u u a se a soma dos divisores prprios de um deles igual ao outro e vice-versa. o e Por exemplo, os divisores prprios de 220 so 1, 2, 4, 5, 10, 20, 11, 22, o a 44, 55 e 110, cuja soma 284. Agora, os divisores prprios de 284 so 1, e o a 2, 4, 71 e 142, cuja soma 220. e Na unidade didtica 4 voc conhecer um fato muito interessante a resa e a peito dos nmeros perfeitos, aqueles cuja soma de seus divisores prprios u o igual a ele mesmo, como 6 = 1 + 2 + 3. e Os pitagricos conheciam os cinco slidos regulares. o o Eles demonstravam uma grande considerao para os nmeros e buscavam ca u conhec-los muito bem. Distinguiam entre eles os chamados nmeros e u gurados, que contavam certos arranjos geomtricos, como os nmeros e u triangulares e quadrados. E interessante como a perspectiva geomtrica prevaleceu na cultura grega. e Mesmo os resultados que hoje obter amos por outras maneiras eram estudados de uma forma geomtrica. e Veja um exemplo a seguir.

5.5 Nmeros gurados e um resultado da teoria de nmeros u u


n(n + 1) , para n inteiro maior ou igual a 1, eram conhe2 cidos pelos pitagricos como nmeros triangulares. Isso porque eles podem ser o u Os nmeros da forma u dispostos num diagrama na forma de tringulos. Aqui esto alguns deles. a a 10 15

...

Agora consideraremos os nmeros quadrados. Esses podem ser representados u em diagramas na forma de quadrados, da o seu nome:

29

UD 2

Histria da Matemtica o a 25

16

...

Veja um resultado de teoria de nmeros que os pitagricos demonstrariam u o usando, digamos assim, simples geometria. Teorema: A soma de uma seqncia de nmeros ue u mpares, comeando c de 1, um nmero quadrado: e u 1 = 1 1+3 = 4 1+3+5 = 9 1 + 3 + 5 + 7 = 16 1 + 3 + 5 + 7 + 9 = 25 Podemos entender o que est acontecendo olhando o diagrama a seguir, assim a como os pitagricos o zeram h mais ou menos 2500 anos. o a 1 +3 +5 +7 +9 +11 36

Atividade 7
Use um esquema semelhante para mostrar que a soma de dois nmeros trianu gulares subseqentes, como 6 e 10, um nmero quadrado. u e u

Atividade 8
O nmero 12 285 um elemento de um par de nmeros amigveis. Descubra o u e u a outro nmero desse par. u 30

UD 2

Histria da Matemtica o a

5.6 Segmentos comensurveis e a primeira crise na Matemtica a a


Um conceito usado pelos pitagricos era a comensurabilidade de dois segmeno tos. Dois segmentos a e b so ditos comensurveis se existir uma unidade de a a comprimento que mea, de maneira exata, ambos segmentos. Isto , dados dois c e segmentos comensurveis a e b, existe um segmento u e nmeros inteiros p e q a u p tais que a = p u e b = q u. A razo de a por b . a e q Os pitagricos acreditavam que quaisquer dois segmentos seriam comensurveis. o a Por isso, a descoberta de um par de segmentos no comensurveis gerou uma a a crise matemtica sem precedentes. Isso ocorreu quando eles estudaram a razo a a entre a diagonal e o lado de um quadrado. O fato de a diagonal e o lado de um dado quadrado no serem comensurveis a a equivalente a 2 no ser um nmero racional. Realmente, se tomarmos um e a u quadrado de lado, digamos, 1, pelo Teorema de Pitgoras, o quadrado de sua a diagonal ser igual a 2. a

O fato de 2 no poder ser expresso na forma p/q, para nmeros inteiros p a u e q, signica que no existe segmento u tal que lado do quadrado = q u e a diagonal do quadrado = p u. Essa descoberta gerou uma crise monumental, pois isso colocava em xeque toda a crena deles, de que a Matemtica seria capaz de expressar qualquer coisa da c a natureza. Devemos lembrar que nmero para os pitagricos signica nmero u o u inteiro e as poss veis razes seriam os chamados nmeros racionais. o u Do ponto de vista prtico, todo racioc a nio matemtico que empregava a sua posio de que quaisquer dois segmentos so comensurveis estava invalidado. ca a a Isso deixava um enorme mal-estar, pois os resultados demonstrados usando essa informao estavam, subitamente, invalidados. Era urgente descobrir uma nova ca 31

UD 2

Histria da Matemtica o a

cadeia de racioc que pudesse substituir esse elo partido e contornar o terr nio vel imbrglio. o Quem resolveu o problema foi Eudoxo de Cnido. Mas isso ser assunto para a a prxima unidade didtica. No entanto, antes de terminarmos esta unidade, o a veja uma demonstrao. ca

5.7

2 no um nmero racional a e u
p2 seja igual q2

A demonstrao de que no h dois nmeros inteiros p e q tais que ca a a u

a 2 est no livro Primeiros Anal a ticos, de Aristteles. Ela do tipo reduo ao o e ca absurdo. Vamos supor que haja dois inteiros p e q tais que uma armao absurda. ca Realmente, suponhamos que tais nmeros existam. Ento, podemos tom-los u a a de tal forma que eles so primos entre si (no tm fatores comuns). Geometria a e camente, estamos supondo que a unidade u escolhida a maior poss e vel. Em particular, isso signica que eles no so ambos pares. Por outro lado, como a a p2 = 2 q 2 , podemos concluir que p par, uma vez que seu quadrado um e e nmero par. Assim, existe um nmero inteiro r, tal que p = 2 r. u u Voltando ` equao original, temos (2 r)2 = p2 = 2 q 2 , donde conclu a ca mos que q 2 = 2 r2 . Mas, isso signica que q tambm par, o que contraria o fato e e original que os nmeros p e q no so ambos pares. Fim da demonstrao! u a a ca As civilizaes que se desenvolveram `s margens do Nilo e entre o Tigre e o co a Eufrates no foram as unicas nem as primeiras a produzir e a usar a conhecimentos matemticos. Praticamente todas as civilizaes de que temos a co not desenvolveram algum tipo de conhecimento matemtico. Dignos de cia a nota so os casos da matemtica chinesa, indiana e, no nosso continente, a a algumas civilizaes andinas. co Por razes de ordem prtica, consideramos apenas a matemtica do Antigo o a a Egito e da Mesopotmia devido especialmente `s suas conexes com a a a o matemtica grega. No deixe de procurar informaes sobre as outras culturas a a co assim que tiver alguma oportunidade. 32 p2 = 2 e produzir q2

Unidade 3
Teoria das Propores de Eudoxo co
Na unidade anterior, voc aprendeu como o surgimento da cultura grega, no e in do sculo IV a.C., mudou profundamente a concepo que o homem cio e ca tinha do universo, sua maneira de pensar e de produzir cincia. e Ousadia e inovao so palavras que facilmente associamos a este fenmeno ca a o cultural que desencadeou uma onda de criatividade, se estendeu por centenas de anos e deu base ` nossa concepo de losoa e de cincia. a ca e

Texto 6: A Primeira Grande Crise na Matemtica a


Os primeiros matemticos gregos tomaram o volume de conhecimento maa temtico acumulado ao longo de milnios pelas culturas que oresceram na a e Mesopotmia e no Egito e o moldaram ` sua maneira. a a No entanto, a descoberta de dois segmentos no comensurveis, o lado e a a a diagonal de um quadrado, gerou uma crise profunda, perturbando essa ordem por eles criada com a fora de um cataclismo. c Para entender a razo de tamanha comoo preciso lembrar da maneira como a ca e os gregos passaram a conceber a Matemtica, introduzindo o mtodo dedutivo. a e Isso o que chamamos de axiomatizao da Matemtica. Esta , basicamente, e ca a e a mesma maneira como fazemos Matemtica at hoje. a e Em poucas palavras, o seguinte: o mtodo dedutivo usa as regras denidas e e pela lgica (outra inveno dos gregos) para demonstrar as armaes mao ca co temticas, os teoremas, usando resultados anteriores. Esse processo precisa a comear em algum lugar. Os pontos de partida so armaes aceitas como c a co 33

UD 3

Histria da Matemtica o a

verdadeiras, chamadas axiomas. As teorias matemticas, isto , as colees de a e co teoremas estabelecidos, permanecero para sempre. Novas teorias podem ser a constru das sobre este alicerce, ele as suportar. E uma situao totalmente a ca diferente de outras cincias, como a Biologia ou a F e sica. Nestas cincias, e acontece de novas teorias surgirem derrubando as anteriores.

6.1 A questo dos segmentos no comensurveis, mais uma vez a a a

Observe que, na terminologia atual, os termos postulado e axioma querem dizer a mesma coisa, so sinnimos. a o No passado, no entanto, dava-se o nome de axioma `s a armaes que eram co evidentes por si mesmas e tinham que ser admitidas necessariamente como verdadeiras, j postulado a poderia ser demonstrado, mas era tomado como verdadeiro e usado sem demonstrao. A priori, a ca armao chamada de ca postulado ainda no fora a aceita como verdadeira pela pessoa a quem era endereada. Por isso o nome, c uma vez que postular tambm indica um pedido. e

Neste quadro, os axiomas funcionam como verdadeiras pedras fundamentais sobre as quais toda a estrutura repousa. O primeiro axioma apresentado no primeiro livro dos Elementos de Euclides a armao: e ca Dados dois pontos, h um segmento de reta que os une. a A terminologia antiga postulado. Veja, a palavra axioma, que agora usamos, e originalmente signicava dignidade ou valor. Muito bem, os pitagricos consideravam como axioma, ou seja, assumiam como o verdadeira, a armao: ca Quaisquer dois segmentos so comensurveis. a a A descoberta de que o lado de um quadrado qualquer e a sua diagonal no so a a comensurveis, equivalentemente 2 no da forma p/q, para inteiros p e q, a a e com q no nulo, signicou que a armao no mais poderia ser usada como a ca a axioma. Isso invalidou todas as demonstraes que haviam sido feitas usando co essa armao, de maneira direta ou indireta. Ou seja, uma srie de teoremas ca e caram, subitamente, sem suas demonstraes. Isto , no eram mais teoremas. co e a Numa palavra: desastre. Realmente, os pitagricos acreditavam que tudo que h no universo poderia ser o a descrito pela Matemtica. Eles acreditavam na mxima: a a Todas as coisas so nmeros. a u A existncia de dois segmentos no comensurveis ameaava esta armao, e a a c ca pois os nmeros a que eles se referiam eram os nmeros racionais. u u 34

UD 3

Histria da Matemtica o a 2 mostra

Veja, o fato da relao entre o lado e a diagonal de um quadrado ser ca

que existem relaes f co sicas que no podem ser representadas em termos dos a nmeros racionais. Por isso, eles chamavam essa razo de alogos, o inexprim u a vel. Neste ponto, preciso dizer uma palavra em favor dos pitagricos. O erro coe o metido sutil. A idia a seguinte: parece razovel que possamos medir dois e e e a segmentos quaisquer usando apenas mltiplos de uma certa unidade. Ora, basta u que tomemos essa unidade sucientemente pequena, no mesmo? Por exema e plo, dadas duas distncias, se no for poss medir ambas usando quilmetros a a vel o de maneira justa (algo assim como 13 km ou 307 km), talvez possamos faz-lo e usando metros ou mesmo mil metros. Foi nisso que os antigos gregos acreditaram. Alm do mais, h o aspecto prtico, que no podemos esquecer. At hoje, e a a a e usamos aproximaes racionais para expressar todas as grandezas do mundo que co nos cerca. A tecnologia nos ajuda a melhorar essas aproximaes. co No entanto, no poss encontrar uma unidade de comprimento que mea, a e vel c de maneira justa, o lado e a diagonal de um quadrado, por menor que seja, pois 2 no um nmero racional, como vimos na unidade didtica anterior. a e u a A crise gerada pela existncia de segmentos no comensurveis perdurou at que e a a e um matemtico genial apresentasse uma idia nova, que alavancaria a questo. a e a Esse matemtico foi Eudoxo, nascido na ilha de Cnido, um contemporneo de a a Plato, fundador de uma escola de losoa, chamada Academia, que tanto a inuenciou nossa cultura.

Atividade 9
Voc conhece outro exemplo de um par de magnitudes no comensurveis? e a a Use o fato de ser irracional para mostrar que o raio r de um c rculo e sua circunferncia, 2 r, so no comensurveis. e a a a Voc conhece duas reas que sejam no comensurveis? Pense no problema da e a a a quadratura do c rculo.

Antes de contarmos um pouco da histria de Eudoxo e de suas idias para o e resolver o problema dos segmentos no comensurveis, vamos falar sobre a a a noo de innito, como era vista naquele tempo. ca 35

UD 3

Histria da Matemtica o a

Texto 7: Problemas com Innito


O que causou toda a diculdade, isto , a existncia de segmentos no coe e a mensurveis, de certa forma, o innito, um adversrio fenomenal. Essa crise a e a colocava os matemticos da poca frente a um conceito que tem provocado, ao a e longo da histria da cincia, e da Matemtica em particular, algumas de suas o e a maiores diculdades, mas que tem gerado, tambm, alguns de seus melhores e resultados. O problema reside no fato de, mesmo no sendo 2 um nmero a u racional, podermos encontrar nmeros racionais arbitrariamente prximos a ele. u o

7.1

2 e as fraoes cont c nuas

Queremos obter uma seqncia de nmeros racionais que estejam mais e mais ue u prximos a 2. Uma maneira de fazer isso expressando esse nmero como o e u uma frao cont ca nua. Voc no precisa ser um expert no assunto, que muito e a e interessante, para entender a idia geral. e Observe que 2 = 1+ 21 = ( 2 1)( 2 + 1) = 1+ = 2+1 1 = 1+ . 2+1 Se voc achou que essa uma maneira estranha de escrever e e mais podemos fazer. 1 2 = 1+ = 1+ 2+1 1 1 1+ 2+1 = +1 1 1 2+ 2+1 2, veja o que

Prosseguindo assim, obtemos 2 = 1+ 1 1 2+ 2+1 = 1+ 2+ 1 1 1 2+ 2+1 36 = 1+ 2+ 2+ 1 1 1 1 2+ 2+1

UD 3

Histria da Matemtica o a

Esse processo pode ser continuado por tantas vezes quanto quisermos, gerando uma espcie de frao prolongada, representada por e ca 2 = 1+ 2+ 2+ 2+ 2+ 2+ 1 1 1 1 1 1 2 + ...

Se interrompermos esse processo, obtemos um nmero racional que ser uma u a aproximao racional de 2. Realmente, uma aproximao de 2 com 9 casas ca ca decimais 1.414213562, enquanto e 1+1/(2+1/(2+1/(2+1/(2+1/(2+1/(2+1/2)))))) = 577 = 1.414215686. 408

Uma frao cont ca nua algo assim como um fractal algbrico. e e Agora vamos dar uma pausa para voc fazer um exerc e cio.

Atividade 10
Use a igualdade 1+ 5 1 = 1+ 2 1+ 5 2 2.

para gerar uma frao cont ca nua, como foi feito no caso de

1+ 5 , Use essa frao cont ca nua para calcular uma boa aproximao do nmero ca u 2 conhecido como razo urea e uma boa aproximao de 5. a a ca Continuando, lembramos que a noo innito de enorme importncia para a ca e a Matemtica, mas as diculdades que apresenta so igualmente fenomenais, a a como voc pode perceber, no caso da crise gerada pelo descobrimento da e existncia de comprimentos no comensurveis. e a a 37

UD 3

Histria da Matemtica o a

Essas crises se repetem ao longo da histria e cada superao representa um o ca avano monumental. O surgimento do Clculo, no sculo 17, e a compreenso c a e a das teorias desenvolvidas por Cantor, no in do sculo 20, cio e so exemplos disso. a

Texto 8: Zeno e seus Paradoxos a


A disputa nito versus innito quase to antiga quanto a Matemtica e e a a suas diculdades indicam a importncia da questo. E claro que esse problema a a transcende a Matemtica. a Os paradoxos de Zeno so resultados dessa antiga disputa. Para que voc a a e entenda como eles se colocam preciso ter uma idia do contexto cultural onde e e eles surgiram. Do ponto de vista da losoa, o principal debate est na questo da verdadeira a a
Zeno de Elia a e Paradoxo (do grego oo) signicava, originalmente, opinio errada, a em oposio a ortodoxo, que ca signicava opinio correta. a Com o passar do tempo a palavra paradoxo passou a indicar as armaoes c auto-contraditrias, como eu o estou mentindo. Se admitirmos que a frase e verdadeira, surge uma contradio. Se admitirmos ca que ela falsa, ocorre a e mesma coisa.

existncia de algo que seja innito. Todos concordam que o conjunto dos e nmeros naturais uma coisa potencialmente innita. Algo como um innito u e virtual. No entanto, como diria Aristteles, esse innito s existe nas nossas o o mentes. Anaximandro, disc pulo de Tales de Mileto, concebia o universo como uma innidade de mundos que existem desde sempre e que existiro durante um a tempo inesgotvel. Dessa forma, ele inaugurou a questo posicionando-se a a a favor da existncia de algo innito. e Em posio radicalmente oposta a Anaximandro, Parmnides acreditava que ca e o universo seria constitu de um unico objeto. Essa concepo monista do do ca universo implica a negao de qualquer movimento. Isso porque a existncia de ca e algum movimento demanda uma posio inicial e uma posio nal, contrarica ca ando a unicidade do universo. Muito bem, Zeno era disc a pulo de Parmnides e queria dar suporte ` teoria e a de seu mestre, mostrando que o movimento seria apenas uma iluso. Para a tanto, produziu quatro famosos argumentos com os quais pretendia mostrar que admitir a existncia de movimento implicaria algum tipo de absurdo. Esses e argumentos so conhecidos como paradoxos de Zeno. a a Um desses paradoxos arma ser imposs levantar-se da cadeira onde voc est vel e a sentado e caminhar at a porta mais prxima. Isto porque primeiro voc teria e o e 38

UD 3

Histria da Matemtica o a

que caminhar a metade desta distncia e depois teria que caminhar a metade a da metade que estaria faltando. Em seguida, a metade do que restou e assim por diante, interminavelmente. O absurdo que este paradoxo apresenta se deve ` negao do innito. Se a ca admitirmos, como normalmente o fazemos, ser poss percorrer uma innidade vel de pontos em um intervalo nito de tempo, podemos refut-lo facilmente. Ou a seja, Zeno nega o innito para concluir que o movimento um absurdo. a e Este paradoxo de Zeno pode ser apresentado de uma maneira matemtica. a a Veja: Um ponto movido da posio 0 na direo da posio 1, na reta real, da e ca ca ca seguinte forma: primeiro ele atinge a posio 1/2, depois a posio 3/4, em ca ca seguida 7/8, depois 15/16, e assim por diante. No n-simo estgio, o ponto se e a encontrar na posio 1 a ca 1 . Logo, imposs chegar at a posio 1 pois, e vel e ca 2n para chegar at l, o ponto teria que percorrer uma innidade de estgios. e a a

Atividade 11
Voc poderia mostrar que a equao anterior no tem soluo? Isso implica a e ca a ca impossibilidade de mover o ponto da posio 0 para a posio 1? ca ca Alm disso, lembre-se da frmula de soma dos termos de uma progresso e o a geomtrica para calcular a seguinte soma: e 1 1 1 1 + + + + n. 2 4 8 2 Voc sabe a frmula da soma innita dos termos de uma progresso geomtrica? e o a e Essa frmula s vlida para as progresses geomtricas cujas razes satisfazem o oe a o e o uma determinada condio. Qual condio essa? A progresso correspondente ca ca e a ao paradoxo de Zeno satisfaz a essa condio? a ca Um outro paradoxo de Zeno conhecido como Aquiles e a tartaruga. Aquiles, a e o mais famoso corredor da antiga Grcia, aposta uma corrida com a sbia tare a taruga. Considerando sua morosidade, a tartaruga pede a Aquiles uma pequena vantagem: que ela possa iniciar a corrida na metade do percurso. Aquiles cede, pois corre duas vezes mais rpido que a tartaruga. Muito bem, segundo Zeno, a a 39

UD 3

Histria da Matemtica o a

ele perde a corrida. Na verdade, segundo Zeno, a corrida nunca mais termia nar e a tartaruga estar sempre na frente de Aquiles. Isso porque, quando ele a a atinge o ponto de onde a tartaruga largou, a metade da raia, ela j avanou at a c e a metade da metade que lhe faltava percorrer. Aquiles segue at esse ponto, e mas a tartaruga j se encontra na metade de sua prxima etapa, e assim por a o diante. Resumindo, a tartaruga sempre estar na frente do magn a co Aquiles. Note que, para o argumento funcionar como Zeno o quer, preciso admitir a e que o espao e o tempo so cont c a nuos e o movimento uniforme (velocidade e constante). Alm disso, a maneira como Zeno descreve a histria sugere que e a o Aquiles e a tartaruga passariam por uma innidade de etapas, metade de metade, depois a metade do que faltou, e assim por diante. Para refutar esse paradoxo, basta que lembremos da nossa concepo de moca vimento. Admitimos que poss percorrer uma innidade de posies (cada e vel co um dos pontos entre a partida e a chegada) em uma innidade de instantes (um para cada posio), mas num intervalo limitado de tempo. ca Voc pode buscar os outros paradoxos de Zeno e analis-los. Eles seguem o e a a mesmo padro: negao do innito com mais algumas consideraes, que implia ca co cam a no existncia de movimento. Essa formulao equivalente ao seguinte: a e ca e a admisso da existncia de movimento com mais algumas consideraes, que a e co implicam a existncia de innidades. e Nossa concepo admite innidades. Numa linguagem moderna, aceitamos a ca frmula matemtica o a lim 1 1 2n = 1.

n+

Uma das diculdades que encontramos ao tentar entender a maneira como os antigos enfrentavam os problemas matemticos est no fato de ns, de uma a a o certa maneira, j sabermos as solues. a co Nesta seo vamos fazer um esforo de compreender a questo da existncia ca c a e de segmentos no comensurveis da maneira como ela estava colocada para os a a pitagricos. o A recompensa ser apreciar a genialidade de Eudoxo. a 40

UD 3

Histria da Matemtica o a

Texto 9: Eudoxo e a Teoria da Proporo ca


Na raiz do problema est o fato de que os gregos antigos tinham uma viso a a geomtrica da matemtica. Eles no dispunham das ferramentas algbricas e a a e que dispomos hoje, uma vez que essas s vieram a ser desenvolvidas posterioro mente. A notao matemtica faz uma diferena fundamental na resoluo dos ca a c ca problemas. Veja como podemos colocar a questo com a ajuda da Algebra: a

Queremos comparar dois comprimentos x e y.

Suponhamos que exista um certo comprimento u tal que x = mu e y = nu, com m e n dois inteiros. Ora, se m for maior do que n, x maior do que y. e Caso contrrio, x menor do que y. a e Como voc sabe, os pitagricos acreditavam que essa possibilidade ocorreria e o para cada par de comprimentos. Mas, como voc sabe, isso no acontece para e a o lado e a diagonal de um dado quadrado, 1 e 2. Portanto, nesta altura, 1 e 2 no poderiam ser comparados. a

O quinto livro dos Elementos de Euclides apresenta uma teoria que resolve esta questo. A quarta denio desse livro chamada Axioma de Eudoxo e foi a a ca e ele atribu pelo grande Arquimedes. Ela diz: da

Duas magnitudes podem ser comparadas quando um mltiplo de u cada uma delas for maior do que a outra.

Veja, segundo essa denio, um comprimento e uma rea no so magnitudes ca a a a comparveis. No entanto, a diagonal do quadrado maior do que seu lado e, a e por sua vez, menor do que o dobro deste lado. e 41

UD 3

Histria da Matemtica o a

Assim, segundo Eudoxo, 1 e

2 so comparveis. a a

Mas restava uma outra etapa, ainda mais dif cil. Como denir a igualdade de duas razes de magnitudes comparveis? Ou seja, queremos estabelecer a o a igualdade a est para b assim como c est para d. a a Ns dizemos, simplesmente, a est para b assim como c est para d se, e somente o a a se, a d = c b. Mas, lembre-se, os pitagricos (assim como Eudoxo) no o a dispunham da multiplicao. ca Note que eles j sabiam como fazer para os pares de magnitudes comparveis. a a Suponha que existam magnitudes u e v, assim como nmeros inteiros m, n, p u e q, tais que a = m u, b = n u, c = p v e d = q v. Ento, dizemos a que a est para b assim como c est para d se, e somente se, m q = n p. (E a a fcil multiplicar nmeros inteiros!) a u Mas, como eles poderiam estabelecer que 1 est para a para 2? Veja a brilhante soluo de Eudoxo. Ela aparece como a quinta denio do ca ca quinto livro dos Elementos de Euclides. Em termos atuais o seguinte: e a est para b assim como c est para d se, e somente se, para a a quaisquer inteiros m e n, vale: (1) ma < nb se, e somente se, mc < nd; (2) ma = nb se, e somente se, mc = nd; 42 2 assim como 2 est a

UD 3 (3) ma > nb se, e somente se, mc > nd.

Histria da Matemtica o a

Atividade 12
Use a denio de Eudoxo para convencer-se de que 1 est para ca a 2 est para 2. Ou seja, a 1 2 = . 2 2 2 assim como

Observe que, nesse contexto, nmero inteiro quer dizer nmero positivo. Os u u nmeros negativos s foram introduzidos posteriormente. u o O mrito dessa denio est no fato de que ela permitiu que os antigos e ca a gregos dispusessem da estrutura dos nmeros reais. O progresso feito por essa u teoria s se compara aos trabalhos sobre os nmeros reais realizados por o u Cauchy, Weierstrass e Dedekind, matemticos do sculo 19, dos quais a e voltaremos a falar.

9.1 Eudoxo e a rea do c a rculo


Nascido em 408 a.C., em Cnido, uma pequena ilha grega prxima da atual o Turquia, Eudoxo estudou Astronomia, Medicina, Geograa e Filosoa, alm e de Matemtica, com importantes mestres e em diferentes lugares por onde a viajou. Estudou na Itlia com Arquitas, que fora aluno de Plato. Arquitas a a estava interessado no problema da duplicao do cubo. Chegou a estudar na ca Academia de Plato, em Atenas, por um breve per a odo. Como era muito pobre, morava em um bairro da periferia de Atenas, nas bases do monte Piraeus, zona porturia, e percorria diariamente um longo caminho de ida e volta at a escola a e de Plato. a Ele retornou a sua nativa ilha onde contribuiu como legislador, atuando na vida pblica. Escreveu livros sobre astronomia, meteorologia e outros temas, ensinou u essas disciplinas e construiu um observatrio. Eudoxo morreu em Cnido, no ano o 355 a.C. Do ponto de vista matemtico, como voc viu, resolveu a primeira grande crise a e que a Matemtica enfrentara. Veja, a seguir, como suas idias resultavam em a e teoremas. Eudoxo demonstrou que 43

UD 3

Histria da Matemtica o a a rea de um c a rculo proporcional ao quadrado de seu dimetro. e a

Para isso, ele usou um resultado observado por Antifo, que fora o primeiro a a sugerir que a rea do c a rculo poderia ser calculada em termos de pol gonos regulares nele inscritos. Aqui est o resultado de Antifo: a a Um 2n -gono regular inscrito em um c a rculo ocupa mais do que 1 1 n1 de sua rea. a 2 Por exemplo, um quadrado ocupa mais do que a metade da rea do c a rculo em que est inscrito. a Voc j tem conhecimentos sucientes para resolver o prximo exerc e a o cio. Vamos a ele.

Atividade 13
Mostre que a rea de um quadrado ocupa mais do que a metade da rea do a a c rculo em que ele est inscrito. a Mostre que a rea de um octgono regular ocupa mais do que a o c rculo em que est inscrito. a Sugesto. Olhe para o seguinte desenho: a 3 da rea do a 4

44

UD 3

Histria da Matemtica o a Agora, examinemos a argumentao de Eudoxo. ca

Veja, precisamos mostrar que a rea de um c a rculo proporcional ao quadrado e de seu dimetro. Em s a mbolos, devemos mostrar que existe uma constante K, tal que K d2 = rea(C), a onde C o c e rculo de dimetro d. a A constante K a mesma para qualquer c e rculo. Portanto, se aplicarmos a frmula para o c o rculo de dimetro 1, obtemos a K = rea(c a rculo de dimetro 1). a

Atividade 14
Calcule o valor de K em termos de .

Como voc sabe, s h trs possibilidades: e o a e (a) K d2 < rea(C), a (b) K d2 = rea(C) ou a (c) K d2 > rea(C). a Vamos mostrar que as possibilidades (a) e (c) levam a contradies e, portanto, co s restar a possibilidade (b). o a Vamos, ento, supor que (a) ocorre. Ou seja, a K d2 < rea(C). a Agora, segundo o Axioma de Arquimedes, que na verdade o atribui a Eudoxo, podemos escolher um nmero n sucientemente grande, de tal maneira que u 1 2n1 rea(C) < rea(C) K d2 . a a

Como n um nmero grande, 1/2n1 sucientemente pequeno para que e u e 1/2n1 rea(C) ainda seja menor do que a diferena (positiva) a c rea(C) K d2 . a 45

UD 3 Podemos reescrever a desigualdade anterior na forma 1 1 2n1 rea(C) > K d2 . a

Histria da Matemtica o a

Vamos denotar por An (C) a rea do n-gono regular inscrito no c a a rculo C. Usando essa notao, o resultado de Antifo ca a e An (C) > 1 1 2n1 rea(C). a

Assim, dessas duas inequaes, obtemos co An (C) > 1 1 2n1 rea(C) > K d2 . a

Pacincia, estamos prximos ao m. Antifo tambm sabia que An (C), a rea e o a e a no n-gono inscrito em C, igual a d2 An , (d2 vezes An , a rea do n-gono a e a a inscrito no c rculo de dimetro 1). a Mas, lembre-se, K rea do c ea rculo de dimetro 1, portanto maior do que An , a a rea do n-gono nele inscrito. a a Colocando tudo isso junto, temos, K d2 > An d2 = An (C) > K d2 . Ora, isso uma contradio! Portanto, a possibilidade que deu in a tudo e ca cio isso, (a) K d2 < rea(C), no ocorre. a a E poss construir uma linha de argumentao que exclui, tambm, a possibivel ca e lidade (c) K d2 > rea(C). a Portanto, como Eudoxo armou, K d2 = rea(C). a Nessa unidade didtica voc aprendeu como foi resolvida a primeira grande a e crise da Matemtica. a Na prxima, voc conhecer um pouco mais a estrutura dos Elementos de o e a Euclides, assim como um panorama das ultimas conquistas dessa cultura que cou conhecida com a Era de Ouro da matemtica grega. a 46

Unidade 4
O Quinto Postulado da Geometria Euclidiana
Nesta unidade didtica voc conhecer uma das questes que ocupou a a e a o ateno de muitos matemticos, desde os tempos de Euclides at os dias de ca a e Gauss, quando foi denitivamente esclarecida, de maneira surpreendente. A questo era se o Quinto Postulado, formulado por Euclides, decorreria a (ou no) como conseqncia dos outros postulados. a ue Para entender melhor essa histria, bom aprender um pouco sobre o o e contexto em que surgiu a chamada geometria euclidiana.

Texto 10: Os Elementos de Euclides


At 300 a.C., Atenas havia sido o principal plo cultural da Antigidade, mas e o u essa primazia passou para uma cidade da Africa Alexandria, no Egito, que abrigou a maior biblioteca da Antigidade. Melhor seria cham-la papiroteca, u a pois chegou a ter em seu acervo mais de 600 000 rolos de papiro, os livros daquela poca. e Neste ambiente extremamente prop para o estudo e para a pesquisa, a Macio temtica grega atingiu o apogeu e viveu sua era de ouro. A escola de Maa temtica de Alexandria foi fundada por Euclides e gerou matemticos fabulosos, a a como Aristarco, Arquimedes, Apolnio e Eratstenes. o o O ncleo do conhecimento matemtico que havia sido desenvolvido pelas geraes u a co anteriores foi compilado em uma coleo de 13 livros, todos extremamente suca cintos, geralmente apresentados em um s volume, sob o t o tulo de Elementos. O organizador dessa obra foi Euclides. 47
Euclides (325 - 265 a.C.)

UD 4

Histria da Matemtica o a

O sucesso dessa coletnea foi tamanho que todas as outras colees semelhana co tes, escritas antes dela, pereceram. Isso porque os livros eram copiados a mo a e os trabalhos considerados superados no eram mais reproduzidos. a 10.1 Breve descrio dos Elementos ca Os seis primeiros livros dos Elementos apresentam a geometria plana. O primeiro deles comea com 23 denies, seguidas de 5 postulados e mais 5 noes c co co comuns que essencialmente estabelecem a existncia de objetos matemticos e a e, por assim dizer, as regras do jogo. Lembre-se: postulados e noes comuns co so os axiomas da teoria apresentada por Euclides, a que chamamos (muito a propriamente) de geometria euclidiana. Por exemplo, o primeiro postulado garante a existncia da reta que contm dois e e pontos dados. Uma das noes comuns a armao: Coisas que so iguais co e ca a ` mesma coisa so iguais entre si. a a O quinto postulado chamado Postulado das Paralelas e difere dos quatro e anteriores, por ser bem mais elaborado. Com o cenrio estabelecido pelas denioes, pelos postulados e noes comuns a c co (os axiomas da teoria), so apresentadas 48 proposies. O primeiro livro a co e um verdadeiro tour de force, cujo objetivo apresentar, justamente nas duas e ultimas proposies, o Teorema de Pitgoras e a sua armao inversa. co a ca Caso voc disponha de acesso ` rede internet, poder visitar a pgina e a a a
Diagrama ilustrativo da demonstrao do Teorema de ca Pitgoras, como a e apresentada nos Elementos.

www.mat.uc.pt/jaimecs/euclid/1parte.html e ler uma traduo para ca portugus deste primeiro livro. e O quinto livro apresenta o trabalho de Eudoxo, sobre a teoria das propores, co enquanto o livro seis as aplica ` geometria plana. a A seguir, continuamos a falar sobre os Elementos, com ateno ca especial a nmeros. u

Texto 11: Euclides e os Nmeros u


Os livros sete, oito e nove lidam com teoria de nmeros. Os principais resultados u a apresentados incluem: 48

UD 4

Histria da Matemtica o a

algoritmo de Euclides, para calcular o mximo divisor comum de dois a nmeros; u prova de que h uma innidade de nmeros primos; a u prova de que todo nmero inteiro positivo da forma u n = 2m1 (2m 1) um nmero perfeito, sempre que o nmero 2m 1 for primo. e u u

11.1 A innitude dos primos


Todos ns sabemos da grande utilidade do algoritmo de Euclides e a prova de o que h uma innidade de primos uma prola matemtica. A idia mostrar a e e a e e que, dada uma lista de nmeros primos, sempre podemos encontrar um outro u primo que no est na lista. Veja: a a Dados p1 , p2 , p3 , . . . , pm , nmeros primos, considere o nmero u u p = (p1 p2 p3 pm ) + 1. que maior do que qualquer um dos pi . e Agora, ou p primo, e teremos o extra primo (que no est na lista original), e a a ou p tem fatores primos que no esto listados. a a A razo a seguinte: se algum dos primos listados originalmente for um fator a e de p, esse nmero dividiria a ambos os nmeros (p1 p2 p3 pm ) + 1 u u e p1 p2 p3 pm . Portanto, dividiria a diferena entre eles, o que c e um absurdo, pois essa diferena 1 e o menor primo o 2. c e e A unica imperfeio nesse livro a falta de uma demonstrao rigorosa do fato ca e ca conhecido como Teorema da Fatorizao Unica. Isto , todo nmero (inteiro ca e u positivo) se decompe, de maneira unica, como um produto de fatores primos, o a menos da ordem: 6 = 2 3 = 3 2. Esse teorema foi demonstrado por Gauss, muito tempo depois.

Atividade 15
Considere a lista 2, 3 e 5, de nmeros primos, e obtenha um extra nmero primo u u usando a estratgia usada na demonstrao apresentada. e ca 49

UD 4 Faa a mesma coisa com a lista 3, 5 e 7. c

Histria da Matemtica o a

11.2 Nmeros perfeitos u


O conhecimento que os gregos tinham sobre os nmeros mostra seu enorme u interesse pela Matemtica. O estudo dos nmeros perfeitos herana dos a u e c matemticos pitagricos a o Um nmero inteiro positivo perfeito se for igual ` soma de seus divisores u e a prprios. Por exemplo, os divisores prprios de 6 so: 1, 2 e 3. Como o o a 1 + 2 + 3 = 6, ele um nmero perfeito, assim como 28 e 496. e u Voc pode constatar isso usando a frmula dada por Euclides, listada anteriore o mente. Por exemplo, para m = 3, 28 = 22 (23 1) = 4 7. A frmula dada por Euclides gera nmeros perfeitos desde que 2m 1 seja um o u nmero primo. Portanto, uma boa pergunta seria: quando 2m 1 primo? u e a E fcil descobrir muitos casos em que esse nmero no primo. u a e Usando o fato de 2ab 1 = (2a )b 1b e as fatorizaes do tipo co x2 y 2 = (x y) (x + y) x3 y 3 = (x y) (x2 + xy + y 2 ) x4 y 4 = (x y) (x3 + x2 y + xy 2 + y 3 ) x5 y 5 = (x y) (x4 + x3 y + x2 y 2 + xy 3 + y 4 ) obtemos 2a b 1 = (2a 1) (2a )b1 + (2a )b2 + + 2a + 1 . Assim, se m no primo, ento 2m 1 tambm no primo, pois pode ser a e a e a e fatorado por 2n 1, para algum fator n de m. Resta a pergunta: se m primo, 2m 1 primo? e e Se a resposta fosse positiva, estaria provado que h uma innidade de nmeros a u perfeitos, uma vez que h uma innidade de nmeros primos. No entanto, como a u o nmero 211 1 se decompe como 23 89, a resposta no. u o e a 50

UD 4

Histria da Matemtica o a

O tema dos nmeros perfeitos e dos primos da forma 2m 1 continua interesu sando os matemticos at hoje. Os nmeros da forma 2m 1 so chamados a e u a nmeros de Mersenne, em homenagem ao padre Marin Mersenne (1588 - 1648). u Esse padre armou, no prefcio de um livro, que os expoentes a 2, 3, 5, 7, 13, 17, 19 e 31 geram nmeros perfeitos. u A partir da os clculos cam dif , a ceis para serem efetuados a mo, pois os a nmeros se tornam muito grandes. u Posteriormente, Leonhard Euler (1707 - 1783) provou que todo nmero perfeito u par da forma descrita pelos gregos, fechando essa parte da histria que hae o via comeado na Antigidade. No entanto, vrios problemas envolvendo essas c u a questes continuam a nos desaar at hoje. Por exemplo, mesmo com a ajuda o e de computadores, s conhecemos 32 nmeros perfeitos. Alm disso, no se o u e a sabe se h uma innidade deles, assim como no se sabe se h algum nmero a a a u perfeito mpar. Apesar de que, se houver algum, ele ser maior do que 10300 . a Ainda sobre nmeros, o dcimo livro apresenta a teoria dos nmeros u e u irracionais, trabalho devido ao matemtico Teeteto, adequado por Euclides `s a a idias introduzidas por Eudoxo. e Agora, continuando o texto sobre os Elementos, vamos enfocar os ultimos livros, que tratam de Geometria Espacial.

Texto 12: Geometria Espacial


O livro 11 apresenta, principalmente, as denies e fatos bsicos. A proposio co a ca 2 do livro 12 foi apresentada na unidade didtica anterior. E a prova dada por a Eudoxo que a razo das reas de dois c a a rculos igual ` razo dos quadrados e a a dos seus dimetros. Ou seja, a rea do c a a rculo proporcional ao quadrado do e seu dimetro. a A tcnica desenvolvida por Eudoxo, chamada mtodo da exausto, tambm e e a e e usada para mostrar que a razo dos volumes de duas esferas proporcional ` a e a razo dos cubos de seus dimetros. Essa idias seriam muito bem aproveitadas a a e por Arquimedes. 51

UD 4

Histria da Matemtica o a

O ultimo livro dedicado aos poliedros regulares, conhecidos como Slidos de e o Plato. Por exemplo, dada a prova de que h, precisamente, cinco desses a e a slidos. o

Tudo indica que esse livro foi baseado num tratado sobre o assunto escrito por Theaetetus. A clareza e preciso com que esses livros foram escritos teve grande inuncia a e no desenvolvimento posterior da Matemtica. Mesmo com suas imperfeies, a co permanecer como uma obra a mpar, um verdadeiro tributo aos esforos feitos c pelas geraes de matemticos que viveram a co a Era de Ouro da Matemtica na Grcia. a e

Texto 13: A Questo do Quinto Postulado a


O Postulado das Paralelas foi a armao dos Elementos que mais deu trabalho ca ` comunidade matemtica. Por muito tempo, os matemticos tentaram provar a a a que ele decorreria das armaes anteriores. Ou seja, na nossa linguagem, em co vez de axioma, ele seria um teorema. Gauss foi o primeiro matemtico a crer na impossibilidade de se provar que o a Quinto Postulado decorreria dos quatro anteriores, como atesta uma de suas cartas enviada a um matemtico chamado Franz Taurinos, em 1824. a Este fato foi demonstrado por Eugenio Beltrami (1835 - 1900) e tambm por e outros matemticos. Assim, os quatro primeiros axiomas mais o Postulado das a Paralelas estabelecem a teoria que chamamos geometria euclidiana e um timo e o modelo para a nossa realidade do dia-a-dia. 52

UD 4

Histria da Matemtica o a

No entanto, se considerarmos os quatro primeiros axiomas e tomarmos por axioma a negao do Quinto Postulado, obteremos uma teoria to consistente ca a quanto a geometria euclidiana. Essa teoria chamada Geometria Hiperblica. e o Veja bem, a geometria hiperblica a teoria que obtemos ao trocarmos, na o e geometria euclidiana, o Quinto Postulado pelo seguinte axioma:

Axioma Hiperblico: Existem uma reta r e um ponto P no pero a tencente a r tais que pelo menos duas retas (distintas) contm P e e so paralelas a r. a
Nikolai Ivanovitch Lobachevsky (1793 - 1856) Matemtico russo que, assim a como Gauss e Bolyai, considerou uma geometria sem o Quinto Postulado.

Os primeiros matemticos a produzirem resultados nessa nova rea matemtica a a a foram Nikolai Ivanovitch Lobachevsky e Janos Bolyai. A comunidade matemtica custou a aceitar e a entender essas novas idias e tanto Lobachevsky a e quanto Bolyai caram sem receber, em seu tempo, os mritos por seus feitos. e Uma poss razo para isso pode ter sido o fato de que os teoremas nessa vel a nova geometria so muito estranhos, se comparados com os similares da geoa metria euclidiana. Nesse contexto, o Teorema de Pitgoras falso, assim como a e a frmula da distncia que conhecemos da geometria anal o a tica. Foi dif para a cil comunidade acostumar-se com o fato de duas retas poderem estar to prximas a o quanto quisermos, sem ter qualquer interseo. ca Em 1868, Beltrami conseguiu um modelo euclidiano para a geometria hiperblica. o Isso, mais um resultado provado por Lobachevsky, mostra que a geometria hiperblica consistente se, e somente se, a geometria euclidiana consistente. o e e Em 1882, Henri Poincar construiu um segundo modelo euclidiano para a geoe metria hiperblica, usando idias que remontam a Apolnio, um dos grandes o e o matemticos do passado, ligado ` escola de Matemtica de Alexandria. a a a Este modelo para a geometria hiperblica conhecido como disco de Poincar, o e e no qual as retas so representadas por arcos de c a rculos cujos extremos so a perpendiculares ao bordo do disco. 53
Janos Bolyai (1802 - 1860), lho de um matemtico, a descobriu, independentemente de Gauss e Lobachevsky, a existncia e da geometria hiperblica. o

UD 4

Histria da Matemtica o a

Henri Poincar (1854 - 1912) e foi um matemtico a extraordinrio. Deu a contribuioes em diversas c reas da matemtica, a a especialmente na Topologia, uma rea que ganhou grande a importncia ao longo do a sculo 20. Nessa rea ele e a props a chamada Conjectura o de Poincar, um problema e que atravessou o sculo e dando trabalho aos melhores matemticos do mundo. Em a 2003, o russo Grigori Perelma apresentou uma soluo para ca o problema. Se a sua soluo ca for aceita pela comunidade, ele pode ganhar um prmio e de um milho de dlares. a o

Os dimetros do disco tambm so considerados retas. Dois arcos que no se a e a a intersectam representam duas retas paralelas. Isso inclui os arcos que se intersectam no bordo, pois o modelo considera apenas o interior do disco. Como voc j deve estar imaginando, retas perpendiculares so arcos que no se ine a a a tersectam. H, tambm, um modelo que usa, no lugar do disco, um semiplano. Neste a e caso, as retas so as semi-retas perpendiculares ao bordo e semic a rculos cujo centro pertence ao bordo. Dessa forma, as extremidades desses semic rculos intersectam o bordo do semiplano ortogonalmente. Nessa geometria, a soma dos ngulos internos de um tringulo menor do que 180 , e pode variar de a a e tringulo para tringulo. Alm disso, tringulos com os mesmos ngulos tm as a a e a a e mesmas reas. a Assim, o Quinto Postulado o axioma que caracteriza a geometria euclidiana. e Mudanas nessa armao geram outras teorias, que so chamadas geometrias c ca a no-euclidianas. A Geometria Hiperblica uma delas. a o e Observe como a gravura do artista grco holands Maurits Cornelius Escher, a e a seguir, lembra esse tipo de geometria.

54

UD 4

Histria da Matemtica o a

Um outro exemplo de geometria no-euclidiana a chamada geometria el a e ptica, e foi estudada por Riemann. Essa geometria pode ser vista como a superf de cie uma esfera, na qual as retas so os grandes c a rculos. Nessa geometria, a soma dos ngulos internos de um tringulo maior do que 180 . a a e

Voc j estudou bastante at aqui. Faa uma interrupo na leitura e procure e a e c ca realizar a atividade que propomos.

Atividade 16
Usando o modelo de Poincar, para a geometria hiperblica, e o modelo da e o esfera com grandes c rculos, para a geometria el ptica, desenhe tringulos e a comprove que a soma de seus ngulos internos menor do que 180 , no caso a e da hiperblica e maior do que 180 , no caso da el o ptica.

Georg Friedrich Bernhard Riemann (1826 - 1866) contribui profundamente para o avano da Matemtica c a contempornea, graas a sua a c enorme criatividade. Riemann foi, entre outras coisas, o precursor do conceito de variedade diferencivel. Essa a noo permite levar as idias ca e do clculo diferencivel a um a a n de abstrao muito vel ca elevado e tem aplicaoes em c muitas reas alm da a e Matemtica. a

Texto 14: Crepsculo Dourado de uma Epoca u


Entre os nomes famosos associados ` escola de Alexandria esto Aristarco (310 a a - 250 a.C.) e Eratstenes (275 - 195 a.C.), que, alm de matemticos, tambm o e a e eram astrnomos. o Aristarco considerava verdadeiras as seguintes armaes sobre o sistema solar: co a lua, a terra e o sol so corpos esfricos; a e a terra gira em torno do sol e a lua gira em torno da terra; os raios solares viajam em linhas retas; a lua reete a luz solar. Os eclipses solares ocorrem quando a lua passa entre a terra e o sol, bloqueando a luz solar e os eclipses lunares ocorrem quando a terra passa em frente ao sol, projetando sua sombra sobre a lua. Voc pode ver que, para a poca, Aristarco tinha uma viso bastante correta e e a do sistema solar. Muito bem, usando esses fatos, sem qualquer artefato como uma luneta ou um telescpio, ele calculou a razo da distncia da terra at o o a a e sol pela distncia da terra at a lua. a e 55

UD 4

Histria da Matemtica o a

A idia a seguinte: quando a lua est na sua fase quarto crescente, pode ser e e a vista no cu simultaneamente com o sol e o tringulo de vrtices sol, lua e terra e a e um tringulo retngulo. e a a
T L

s s t S

Medindo o ngulo LT S, Aristarco pde calcular a razo T S/T L, desenhando a o a um tringulo semelhante. a Feito grandioso, tambm, foi o de Eratstenes: mediu o raio da terra. Para e o isso, usou seus conhecimentos de geograa. Primeiro, considerou que os raios solares atingem paralelamente a terra, pois o sol est to distante que, ao fazer a a isso, estaria cometendo um erro desprez vel. Eratstenes sabia, por ter lido em textos conservados na biblioteca de Aleo xandria, que a cidade de Siene (atualmente incorporada em Assu) se encona trava exatamente sobre o Trpico de Cncer. Isso porque no dia 21 de junho, o a o solst cio de vero, o sol do meio-dia iluminava as guas de um poo proa a c fund ssimo. Sabia, assim, que nesse momento o sol incidia perpendicularmente sobre essa cidade. Ento, Eratstenes procurou saber qual seria a posio do a o ca sol, no mesmo dia e instante, sobre a cidade de Alexandria. Para isso, mediu a sombra de um obelisco e concluiu que os raios solares e o obelisco formavam um ngulo de 7 1/5, a quinquagsima parte de um c a e rculo.

Alexandria raios solares paralelos Siene (Assu) a

Portanto, o ngulo formado pelo raio solar e pelo obelisco, que est na direo a a ca do dimetro da terra que passa por Alexandria, o mesmo ngulo do setor a e a circular cujo arco tem extremidades em Siene (Assu) e Alexandria. a 56

UD 4

Histria da Matemtica o a

Conta a lenda que ele pagou a um escravo para medir a distncia de Alexandria a at Siene (Assu), que de 800 km. De posse dessas informaes e usando e a e co geometria elementar, calculou a circunferncia da terra: e 360 800 = 40 000 km. 7 1/5 Assim, chegou ao raio da terra: 6 366, 2 km. Impressionante, no ? a e

Texto 15: Eureka!


Muito bem, Eratstenes foi grande amigo de Arquimedes, possivelmente o maior o matemtico da Antigidade. Vrias coisas que sabemos a respeito das conquisa u a tas cient cas feitas por Arquimedes devemos a cartas que ele escreveu para Eratstenes. Eles se conheceram em Alexandria, onde Arquimedes estudou. o Nesse ponto, cabe a pergunta: o que torna algum como Arquimedes to genial? e a o que o distingue de seus pares, eles mesmos to elevados, como Aristarco, a Eratstenes e outros? o A originalidade , certamente, um fator que indica essa distino. Arquimedes e ca a teve de sobra. Outra o reconhecimento de sua genialidade pelos outros e matemticos. As obras de Arquimedes foram estudadas e citadas por muitos a grandes matemticos, de diferentes pocas e diferentes culturas. Mas, h uma a e a coisa mais que o torna mpar. Arquimedes dominou todo o conhecimento matemtico de sua poca, o fez seu e o moldou ` sua maneira. No se deixou a e a a prender pelas imposies conservadoras que limitavam os mtodos e as solues co e co aos problemas matemticos de seu tempo. a A inuncia de Plato na maneira de pensar dos matemticos daquele tempo e a a fora frutuosa, culminando na produo dos Elementos, que tm um papel fundaca e mental na nossa maneira de pensar a Matemtica at hoje. A idia de verdade a e e absoluta que a Matemtica busca expressar, apesar de ser apenas a sua sombra. a No entanto, essa atitude tambm traz uma forte limitao. Veja, o problema de e ca dividir um dado ngulo em trs partes iguais, por exemplo, no tem soluo no a e a ca mbito das regras estabelecidas pelos axiomas xados por Euclides: construes a co com rgua e compasso. No entanto, o problema tem soluo, se admitirmos e ca outros mtodos, que podemos chamar de mecnicos. Muito bem, Arquimedes e a rompeu com essas limitaes, estudou e usou essas novas idias com o mesmo co e 57
Arquimedes (287 - 212 a.C.) nasceu e morreu na cidade de Siracusa, atualmente na Sic lia. Filho de um astrnomo chamado F o deas, foi enviado ` Alexandria onde a estudou. De volta a sua cidade participou ativamente da vida da corte e produziu uma quantidade enorme de resultados matemticos e a cient cos. Arquimedes era o modelo do cientista distra do, que vivia envolto em seus pensamentos, como atestam vrias anedotas. A mais a pitoresca conta como saiu tempestuosamente de uma banheira gritando eureka! eureka! pela rua, sem se dar conta de sua nudez. Eureka quer dizer achei, encontrei! Arquimedes acabara de ter uma idia de como resolver o e famosos problema da coroa do rei Hiero. a

UD 4

Histria da Matemtica o a

rigor que caracterizou os trabalhos das geraes anteriores. Mais ainda, Arquico medes aplicou esse rigor cient co no estudo da F sica, da Mecnica, criando a novidades que usamos at hoje, como o sistema de bombeamento, chamado Pae rafuso de Arquimedes, e os seus conhecimentos sobre alavancas e polias, para mover grandes pesos.

15.1 Arquimedes e a rea do c a rculo Para descrever, mesmo rapidamente, os resultados obtidos por Arquimedes, levar amos mais de uma unidade didtica, fugindo do propsito desse nosso a o trabalho. Mas, para que voc aprecie um pouco de sua produo matemtica, e ca a veja como ele provou que a rea do c a rculo de raio r r2 . e Veja a bel ssima aproximao de , dada por Arquimedes: ca 10 1 < <3+ 71 7

3+

Essa desigualdade reete, at a terceira casa decimal, a desigualdade 3.140 < e 3.141 < 3.142. Impressionante, no? a Muito bem, Arquimedes chegou a essa aproximao a partir da seguinte armao, ca ca tomada como verdadeira por ele: Dado um c rculo qualquer, h um segmento de reta que mais a e comprido do que o per metro de qualquer pol gono convexo nele inscrito e mais curto do que o per metro de qualquer pol gono convexo circuscrito no c rculo. Qualquer outro segmento que satisfaa c essa propriedade tem o mesmo comprimento. Esse comprimento e igual ` circunferncia deste c a e rculo. Na primeira parte da armao ele caracteriza a circunferncia do c ca e rculo e na segunda estabelece a sua unicidade. Existe e unico, como gostamos de dizer e em Matemtica. a Arquimedes partiu do ponto deixado por Eudoxo a rea de um dado c a rculo e proporcional ao quadrado de seu dimetro para provar, usando o mtodo de a e exausto, que: a 58

UD 4

Histria da Matemtica o a a rea do c a rculo igual ` rea do tringulo retngulo cujos catetos e aa a a so o seu raio e a sua circunferncia. a e

r 2r

A demonstrao segue a mesma linha de racioc usada por Eudoxo para provar ca nio a proporcionalidade (ver a unidade anterior), considerando separadamente as possibilidades de a rea do c a rculo ser maior do que a rea do tringulo, o que a a leva a um absurdo, assim como a possibilidade de ser menor. O episdio da morte de Arquimedes coberto por um pouco de lenda, que o e ajuda a reforar a imagem do cientista imerso em seus pensamentos, desligado c do mundo real. Segundo consta, ele morreu por no ter respondido ao a chamado de um soldado romano, por estar profundamente envolvido com um cil problema de geometria. E dif crer que um homem que esteve to atento `s a a necessidades de seus conterrneos, que tenha tido tantas idias prticas, no a e a a tenha percebido a aproximao e o chamado do soldado. ca E bom lembrar que as invenes blicas de Arquimedes haviam aigido e co e causado muitas baixas entre os soldados romanos que cercavam Siracusa, no intuito de tom-la. E bom lembrar que Marcellus, o general romano que a comandava o cerco de Siracusa, dera ordens expressas para que a vida de Arquimedes fosse poupada. De qualquer forma, o episdio nos lembra que a poca de ouro dos gregos o e estava chegando ao m. Outras civilizaes surgiam e o co poder mudava de mos. a A cultura grega ainda continuaria a exercer sua inuncia, como o faz at os e e nossos dias, mas era tempo para novas idias e novas contribuies. e co Alguns nomes importantes para a Matemtica ainda continuaram a aparecer, a como Apolnio, contemporneo de Arquimedes, com seu tratado monumental o a (mais de 400 teoremas) sobre as cnicas. Houve tambm Hiparco, que tanto o e contribuiu para a fundamentao do que chamamos trigonometria, Menelau ca 59

UD 4

Histria da Matemtica o a

de Alexandria, j na era crist, que fundamentou a trigonometria esfrica, o a a e neo-pitagrico Nicomaco de Gerasa. o Outro grande matemtico foi Diofanto, que viveu j no terceiro sculo da era a a e crist, e dedicou-se ` teoria de nmeros. Diofanto escreveu uma srie de 13 a a u e livros cujo conjunto era intitulado Aritmtica, dos quais conhecemos nove (seis e at 1973, quando mais trs foram descobertos, em traduo rabe). e e ca a A importncia de Diofanto reside, tambm, no fato de ter sido o primeiro a e matemtico a usar uma notao simblica para expresses algbricas. Isso lhe a ca o o e rendeu, com justia, o t c tulo de pai da Algebra. Mas como lgebra uma a e palavra proveniente do rabe, deixaremos essa histria para a a o prxima unidade didtica. o a Aqui esto mais algumas atividades que o convidam a entrar, pelo menos um a pouco, no universo da Matemtica daqueles dias. a

Atividade 17
Suponha que uma coroa de m kg tenha sido feita usando uma mistura de ouro e prata. Se a densidade do ouro for denotada por e a densidade da prata for denotada por , o volume da coroa dado por: e v = mx x + ,

onde x o peso (em quilogramas) de ouro usado na coroa. e Conhecendo a massa da coroa (5 kg, segundo consta a histria, Hiero tinha o a um pescoo fort c ssimo), como Arquimedes descobriu a quantidade certa de ouro e prata usados para fazer a coroa?

Atividade 18
Considere o tringulo de nmeros a seguir: a u 1 3 7 13 15 60 9 17 5 11 19

UD 4 Voce capaz escrever a prxima linha? e a outra? e o

Histria da Matemtica o a

Some os nmeros de cada uma dessas linhas. O que voc pode observar? u e Muito bem, esse tringulo aparece no livro Introduo ` Aritmtica, escrito por a ca a e Nicomaco, no qual ele nota que a soma dos nmeros na n-sima linha n3 . u e e

Atividade 19
Uma das questes que interessava a Diofanto era a seguinte: escreva um dado o nmero, que a soma de dois quadrados, como a soma de outros dois quadrados. u e Por exemplo, 65 a soma de 64 com 1, dois quadrados. e Voc seria capaz de escrever 65 como a soma de outros dois quadrados? e Bem, a resposta de Diofanto a seguinte: e 65 se escreve como a soma de dois quadrados de maneiras diferentes pois o produto de 13 por 5, sendo cada um deles a soma de dois e quadrados. Ser que Diofanto conhecia a identidade algbrica a e (a2 + b2 ) (c2 + d2 ) = (ac bd)2 + (ad bc)2 ?

61

Unidade 5
Resoluo das Equaes Algbricas ca co e
Muito bem, continuando a Histria da Matemtica, voc conhecer como o a e a ocorreu a transio entre o per ca odo de ouro da matemtica grega para a poca a e do renascimento cultural e cient co vivido na Europa no sculo 16. e Durante toda a Idade Mdia, o contedo matemtico alcanado pelos gregos e u a c foi preservado e enriquecido pelos rabes. a E um longo cap tulo da Histria da Matemtica marcado pelo respeito que a o a cultura rabe teve pelos resultados obtidos anteriormente. a Mas no foi s isso, neste per a o odo tambm ocorreram grandes contribuies de e co matemticos indianos. a

Texto 16: H Mouros na Costa! a


Uma das maneiras de perceber a inuncia de uma certa cultura em nossas e vidas a presena de suas palavras no dia-a-dia. e c Azeite, alfazema, beringela, algarismo, mesquinho, arroz, decifrar, algarve, laranja, zero, alicate, javali, oxal, refm, at, alicerce... a e e Responda depressa: o que todas essas palavras tm em comum? e Acertou quem disse: so todas provenientes do rabe! a a Elas chegaram at o nosso idioma devido ` ocupao da Pen e a ca nsula Ibrica pelos e rabes, que resultou num per a odo de convivncia de duas culturas. e 63

UD 5

Histria da Matemtica o a

Com o decl das civilizaes grega, e depois romana, foi a vez dos rabes se nio co a expandirem. Em 711 eles cruzaram o Estreito de Gibraltar e ocuparam Crdoba o e Toledo, na Espanha. At o sculo 12 permaneceram tambm em territrio e e e o portugus e s sa e o ram de Granada, na Espanha, em 1492. Tinham uma cultura rica e eram capazes de absorver e transformar os conhecimentos dos povos que dominavam ou das culturas com que mantinham contato. Por exemplo, conhecemos o livro do astrnomo grego Cludio Ptolomeu, escrito o a no sculo 2, por Almagesto, devido ` sua traduo rabe. Alm disso, os rabes e a ca a e a usaram idias gregas para construir o astrolbio, que permitiu maior segurana e a c na navegao. ca Os livros sobre medicina, escritos por Al Razi (864 - 930) e Avicena (980 - 1037), foram usados na Europa at o sculo 17. Mais ainda, os rabes difundiram o e e a uso do papel no lugar do papiro, que eles trouxeram da China. Isso tornou a produo de livros mais barata, facilitando a difuso de conhecimento. ca a Como os rabes eram originrios de regies de clima inspito, tinham grande a a o o conhecimento sobre irrigao e semeadura. ca Alm de tudo isso, o amor que tinham pela abstrao e pelos nmeros deixou e ca u uma marca forte na Matemtica. Da a India trouxeram o zero, aperfeioaram os c algarismos e contribu ram para a parte mais abstrata da Matemtica, que leva a um nome rabe a Algebra. a Mas, vamos comear falando de um nmero muito especial o zero. c u

Texto 17: Decifrando o Zero


A maioria dos nossos alunos caria surpresa ao saber quanto tempo demorou para que o zero se estabelecesse no cenrio matemtico. Para entender um a a pouco melhor essa histria, preciso lembrar que o zero tem duas funes a o e co desempenhar no sistema numrico. Veja, usamos o zero para expressar certos e nmeros, uma vez que nosso sistema numrico posicional. Assim, os nmeros u e e u 272, 2720 e 2072 podem ser diferenciados uns dos outros. A outra funo ser ca e um nmero em si mesmo. u J os babilnios usavam um sistema numrico posicional, mas mesmo assim a o e o zero no se estabeleceu nessa cultura. Tambm alguns gregos, astrnomos, a e o usavam um sistema numrico posicional, mas isso cou restrito a um nmero e u 64

UD 5 muito pequeno de pessoas.

Histria da Matemtica o a

O zero se estabeleceu, denitivamente, como um s mbolo para representar nmeros num sistema posicional, na u India. Inscries que datam do ano 876 co descrevem um problema (sobre plantaes de ores, para a produo de guirco ca landas) em que os nmeros 270 e 50 so denotados como o fazemos hoje. u a J sua outra funo, como nmero, deu mais trabalho aos matemticos. Foi a ca u a um longo caminho do concreto para o abstrato. Veja, partir de idias tais e como quatro irmos, quatro objetos, para chegar ` idia do nmero 4, exige um a a e u enorme esforo de abstrao. Da para o zero, que representaria o nada, assim c ca como nmeros negativos, exige uma maior abstrao. Como se isso no fosse u ca a o bastante, h ainda a questo de estender a esses novos nmeros a aritmtica a a u e j estabelecida para os positivos. a Alguns matemticos da a India deniram regras para as operaes aritmticas enco e volvendo nmeros positivos, zero e nmeros negativos. Primeiro Brahmagupta e u u depois Mahavira e, perto de 500 anos depois de Brahmagupta, Bhaskara. Tudo funcionou bem, a menos da operao de diviso, onde eles tentavam estabelecer ca a o resultado de uma diviso por zero. a Bhaskara tentou resolver o problema da seguinte forma:
Uma quantidade dividida por zero se torna uma frao cujo denominador ca zero. A essa frao atribu uma quantidade innita. Esta quane ca e da tidade (aquilo que tem zero como seu divisor) no alterada, indepena e dentemente de quanto a ela seja acrescentado ou subtra do, assim como no h mudanas no innito e imutvel Deus quando mundos so criados a a c a a ou destru dos, apesar de inmeras ordens de seres serem absorvidos ou u criados.
x= Bhaskara (1114 - 1185) o e nome que associamos a frmula que resolve a o equao do segundo grau ca ax2 + bx + c = 0, b b2 4ac . 2a

Ele tambm era conhecido e por Bhaskaracharya, que signica Bhaskara, o professor.

Usando a notao atual, Bhaskara estava tentando resolver o problema coloca cando n = . 0 Por mais tentadora que parea essa soluo, ela acarreta vrios problemas. c ca a Primeiro, introduz um nmero extra, o . Isso j condena a frmula. Mas, o u a o pior ainda est por vir, pois ela implica que 0 igual a qualquer nmero a e u n, provocando a contradio de que todos os nmeros seriam iguais. ca u 65

Em 1655, por sugesto de a John Wallis, um dos precursores do Clculo, o a s mbolo passou a ser usado para indicar innito. Anteriormente era usado como uma alternativa para M , que representa 1 000 em algarismos romanos.

UD 5

Histria da Matemtica o a

cil E dif crer que a soluo para o problema seja to simples: basta estabelecer ca a que no se divide por zero, e pronto! De qualquer forma, esta questo afeta a a muitos iniciantes na Matemtica, at hoje. Precisamos lembrar nossos alunos a e constantemente que no se divide por zero. a Estender certas denies tais como elas valem para um dado conjunto de co objetos matemticos, para um conjunto maior, pode ser uma dura tarefa. Esse a foi, de certa forma, o problema enfrentado pelos matemticos da a India, quando tentavam denir as operaes aritmticas, conhecidas para o caso de co e nmeros positivos para incluir o zero e os negativos. u Veja a atividade.

Atividade 20
Veja o que est errado com a seguinte prova de que 1 igual a 2: a e 1 = 1 + 0 + 0 + 0 + ... = = 1 + (1 1) + (1 1) + (1 1) + . . . = = 1 + 1 + (1 + 1) + (1 + 1) + (1 + 1) + . . . = = 1 + 1 + 0 + 0 + 0 + ... = = 2. Seria poss denir uma adio que inclu uma innidade de parcelas? vel ca sse A representao dos nmeros por um sistema posicional, que demanda dez ca u algarismos, bem como as regras que denem as operaes bsicas da co a Aritmtica, envolvendo o zero e nmeros negativos, um importante avano e u e c matemtico que foi transmitido para a cultura rabe e, posteriormente, para a a as culturas europias. e Os algarismos hindu-arbicos, com o sistema numrico posicional e as fraes a e co decimais, so provas de que nossa cultura deve muito aos esforos das a c geraes anteriores. De qualquer forma, esse sistema no foi rapidamente co a absorvido e, por muito tempo, o sistema posicional e o sistema numrico e representado por letras conviveram lado a lado. Hoje, essa maneira antiga de denotar os nmeros tem uma presena residual. u c 66

UD 5

Histria da Matemtica o a

Algo assim como uma passagem do analgico para o digital. Falando nisso, o que horas so? a E claro que est na hora de passarmos ` Algebra. a a

Texto 18: A Algebra Surge no Cenrio Matemtico a a


Abu Jafar Mohamed ibn Musa al-Khwarizmi foi o primeiro matemtico da Casa a da Sabedoria, uma espcie de universidade fundada pelo califa al-Mamum, no e in do sculo 9, em Bagd. Ele escreveu um livro sobre matemtica elementar: cio e a a uma compilao de regras para soluo aritmtica de equaes lineares e de ca ca e co segundo grau, baseado nos trabalhos de Diofante, chamado Hisab al-jabr walmuqabalah. A palavra rabe al-jabr signica combinar, reunir, e a traduo a ca latina desse t tulo deu origem ` palavra lgebra. O livro inicia com uma frase a a do tipo Assim falou al-Kwarizmi... , cuja traduo latina resultou no termo ca algoritmo. De qualquer forma, a semente estava lanada. Tambm entre os matemticos c e a da India oresceu a Algebra. Com a introduo da teoria de nmeros negativos, ca u Brahmagupta (598 - 670) podia dar a soluo completa para equaes lineares ca co diofantinas, do tipo 4x + 6y = 2. Esse tipo de equao s tem soluo (inteira) se o m.d.c. (maior divisor comum) ca o ca dos coecientes (4 e 6) dividir o termo constante (2). Resolva o seguinte problema, proposto por Brahmagupta.

Atividade 21
Em quanto tempo quatro fontes, todas abertas ao mesmo tempo, enchero uma a cisterna, as quais sozinhas a encheriam num dia, em meio dia, num quarto de um dia e num quinto de um dia? Bhaskara, como j vimos, um matemtico da a e a India, cujo nome permanece at e hoje. Como era comum em sua poca, ele redigia os problemas propostos de e forma potica. Veja um exemplo. e 67

UD 5

Histria da Matemtica o a
De um enxame de abelhas pretas, a raiz quadrada da metade voou para um jasmineiro e oito nonos das abelhinhas caram para trs. Sobraram duas, a uma femeazinha voando em torno de uma or de ltus, cuja fragrncia o a acabou capturando um zango, que l cou a zunir. Ento, diga-me to a a a a encantadora garota, quantas abelhas havia no enxame todo?

E claro que tal enunciado mais interessante do que simplesmente: e Resolva a equao ca x/2 + (8/9)x + 2 = x.

Bhaskara tinha uma lha chamada Lilavati, cujo nome foi dado a um de seus livros. Ela deve ter sido uma boa aluna de Matemtica. a Veja mais um problema t pico do Lilavati.

Oh, sbio homem! Um certo rei deu a cinco cavaleiros 57 moedas. Cada a um, por ordem, obteve o dobro e mais uma moeda do que o seu antecessor. Quantas moedas obteve o primeiro cavaleiro, assim como cada um dos outros?

Mas, voltemos aos rabes, conhecendo um pouco do trabalho de Omar a Khayyam referente `s equaes cbicas. Antes, contudo, faremos uma a co u introduo apresentando um resumo histrico dessas equaes. ca o co

18.1 Omar Khayyam e as equaoes cbicas c u


Problemas cuja soluo reca numa equao cbica j eram conhecidos desde ca a ca u a a Antigidade. No entanto, o completo entendimento dessas equaes levou u co bastante tempo. O exemplo mais famoso a questo da duplicao do cubo. e a ca Veja, os gregos j haviam notado que, se fosse poss construir comprimentos a vel x y 1 ca y e z tais que = = , o problema da duplicao do cubo estaria resolvido. x y 2 Em outras palavras, se alm de rgua e compasso fossem admitidas solues e e co atravs de desenhos de parbolas (uma curva obtida por um processo mecnico), e a a o problema estaria resolvido com o seguinte diagrama: 68

UD 5

Histria da Matemtica o a

y = x2 2x = y 2

Muito bem, chegamos a Omar Khayyam (1050 - 1123), que, alm de poeta, e foi matemtico. A lenda conta que ele e mais dois amigos, Nizan e Hassan, a zeram um pacto, de que se algum deles casse rico, dividiria suas riquezas com os demais. Ao que parece, Nizan cou muito rico e, graas a isso, Omar c pde deixar seu of como arteso de tendas, razo do nome Khayyam, para o cio a a se dedicar ` Matemtica. Ele dedicou sua atividade matemtica a estudar a a a a equao cbica ca u x3 + ax2 + bx + c = 0.

Khayyam considerava diversos casos, dependendo dos sinais dos coecientes da equao e usava intersees de cnicas para determinar solues positivas, ca co o co assim como no caso da soluo de x3 = 2, dada anteriormente. ca Em resumo, lidava com casos especiais do seguinte simples resultado.

Teorema: O ponto (x, y) soluo de x3 + ax2 + bx + c = 0 e e ca y = x2 se, e somente se, soluo de (x + a)(y + b) = ab c e e ca y = x2 .

Por exemplo, para resolver a equao x3 2 = 0, Omar considera a interseo ca ca da parbola y = x2 e a hiprbole xy = 2. a e 69

UD 5

Histria da Matemtica o a

y = x2

xy = 2 3 2

Bem longe de Khayyam, um matemtico italiano se ocupa com o sistema a numrico rabe e lida com equaes cbicas. e a co u

18.2 Fibonacci e seus coelhos


O ano de 1202 assistiu ` publicao de um livro chamado Liber Abaci (Livro dos a ca Abacos), que apresentaria para a Europa o sistema numrico rabe. Esse livro e a fora escrito por um matemtico genial, Leonardo de Pisa, mais conhecido pelo a apelido de Fibonacci. Ele aprendera Matemtica na Algria, onde fora acompaa e nhando seu pai, um funcionrio diplomtico. Um dos problemas apresentados a a no livro o que gera a famosa seqncia de Fibonacci. Veja o enunciado: e ue
Suponha que as coelhas demorem um ms para gerar suas crias e que e cada coelha que prenha no in de cada ms, contando do in de cio e cio seu primeiro ms de vida. Alm disso, suponha que cada coelha gere um e e par de coelhos, um macho e uma fmea. Quantos pares de coelhos voc e e ter em 2 de janeiro de 1203 se voc comear com um s par, em 1 de a e c o janeiro de 1202, se em cada ms cada par gera um novo par, que se torna e produtivo a partir do segundo ms? e

A soluo dada por Fibonacci a seguinte: no primeiro ms, teremos um par ca e e de coelhos que se manter no segundo ms, tendo em considerao que se trata a e ca de um casal de coelhos jovens; no terceiro ms de vida daro origem a um e a novo par, e assim teremos dois pares de coelhos; para o quarto ms s temos e o um par a reproduzir, o que far com que obtenhamos, no m deste ms, trs a e e pares. Em relao ao quinto ms, sero dois os pares de coelhos a reproduzir, o ca e a que permite obter cinco pares de coelhos no m deste ms. Continuando desta e 70

UD 5

Histria da Matemtica o a

forma, ele mostra que teremos 233 pares ao m de um ano de vida do par de coelhos com que partimos. Assim, o nmero de coelhos aumenta da seguinte maneira: u 1, 1, 2, 3, 5, 8, 13, 21, 34, 55, 89, 144, 233, 377, . . . caso o processo no seja interrompido. Veja, a seqncia de Fibonacci pode ser a ue obtida da seguinte maneira. F1 = 1; F2 = 1; Fn+1 = Fn + Fn1 . O ultimo nmero da seqncia, a partir do terceiro, a soma dos dois nmeros u ue e u anteriores, sendo que os dois primeiros nmeros so iguais a 1. u a H muita informao sobre os nmeros de Fibonacci na literatura, inclusive a ca u sobre suas relaes com a razo urea. co a a

18.3 Fibonacci e a cbica u


A vida dos matemticos nos dias de Fibonacci no era nada fcil. Nada de a a a posies asseguradas em universidades, nada de aposentadoria ou outros beco nef cios. A competncia era a principal segurana de cada um. Era comum que e c fossem organizados torneios matemticos nos quais as habilidades eram testaa das. Alm disso, um matemtico podia desaar um outro para uma espcie de e a e duelo matemtico. Eles trocavam listas de problemas e o vencedor do desao, a aquele que resolvesse o maior nmero de problemas, mantinha a sua posio u ca sob ou ocupava o lugar do vencido, dependendo do caso. Nesse clima de competio, as respostas dos problemas eram apresentadas, mas ca os mtodos usados para obt-las eram segredos guardados sob sete chaves. e e Os problemas mais dif ceis que circulavam naquela poca eram aqueles que e demandavam a resoluo de uma equao de grau trs, as chamadas equaes ca ca e co cbicas, ou simplesmente cbicas, para os u u ntimos. Em 1225, Frederico II organizou um torneio que foi vencido de maneira brilhante por Fibonacci. Ele simplesmente resolveu todos os problemas propostos. Um desses problemas consistia em resolver a equao ca x3 + 2x2 + 10x = 20. 71
Frederico II era o imperador romano, coroado pelo Papa Honrio III em novembro de o 1220 na Igreja de So Pedro, a em Roma. O Imperador foi, tambm, entre 1197 e 1250, e Rei da Sic lia, e era um monarca culto, protetor das artes e das cincias. e

UD 5

Histria da Matemtica o a

Veja, se substituirmos x por 1 no lado esquerdo da igualdade, obtemos 1 + 2 + 1 10 = 13 < 20. Alm disso, se multiplicarmos a equao por e ca , obtemos 10 x+ x3 x2 + = 2. 10 5

Essas consideraes indicam que a soluo est entre 1 e 2. co ca a Fibonacci constatou alguns fatos sobre a natureza da soluo, por exemplo ca provando que ela no um nmero racional, e arma que a e u x 1 22 7 42 33IV 4V 40V I , = ou seja, x1+ = 22 7 42 33 4 40 + 2 + 3 + 4 + 5 + 6, 60 60 60 60 60 60

que d x = 1.368808108. a O que voc acha de tentar resolver o problema de Fibonacci? e

Atividade 22
Usando o mtodo geomtrico, desenhe cuidadosamente as curvas e e (x + 2)(y + 10) = 40 e y = x2 . Muito bem, voc j sabe que dif achar soluo para uma cbica. Veja a e a e cil ca u diferena com o caso das equaes quadrticas. No caso de ax2 + bx + c = 0, c co a basta usar a frmula de Bhaskara, o b b2 4ac . 2a

x=

Mas havia casos onde as solues encontradas eram mantidas em segredo. co 72

UD 5

Histria da Matemtica o a

Texto 19: Um Grande Segredo


A busca de uma frmula semelhante para o caso das cbicas, e demais equaes o u co de maior grau, era a grande questo que ocupava os matemticos naqueles dias. a a O primeiro matemtico a encontrar uma frmula que se aplica a um certo tipo a o de cbica foi Scipione dal Ferro (1465 - 1526). Ele descobriu como resolver as u chamadas cbicas comprimidas Isso , equaes da forma u e co y 3 + Ay = B, com A e B positivos. Scipione descobriu que, neste caso, bastava determinar s e t tais que 3 st s3 t3 = A, = B,

pois s t soluo de y 3 + Ay = B. Realmente, basta substituir. O problema e ca determinar s e t. Muito bem, a questo algbrica! Basta resolver a equao e a e e ca 3 st = A em s, obtendo s = A e substituir em s3 t3 = B. Isso d a 3t A 3t
3

t3 = B. Um pouco mais de trabalho e obtemos a equao ca t6 + B t3 A3 = 0 27

que pode ser resolvida em t3 pela equao de Bhaskara. Ufa! e essa apenas a ca e cbica comprimida. Veja um exemplo... Para resolver a equao y 3 + 6y 10 = u ca 0, fazemos y 3 + 6y = 10 e colocamos 3 st s3 t3 = 6 = 10.

Fazendo as substituies indicadas, chegamos ` equao t6 + 10t3 8 = 0, co a ca que tem solues t3 = 5 33. Escolhemos a soluo positiva e obtemos co ca 3 t = 5 + 33. De s3 t3 = 10, obtemos s3 = 5 + 33 e, portanto, 3 s = 5 + 33. Finalmente, y=
3

5+

33

5 + 73

33 1.300270977. =

UD 5

Histria da Matemtica o a

Scipione guardou esse segredo at quase seu ultimo suspiro. Antes de morrer, e
Tartaglia signica gago. A cidade em que ele nasceu, Bresia, Itlia, foi saqueada a em 1512 por soldados francesas. Niccolo, ainda menino, foi ferido gravemente em sua mand bula, por uma espada. Isso lhe deixou seqelas e o apelido. u

passou-o para seu assistente, Antonio Fior. Fior no esperou muito. Em 1535 desaou para um duelo um dos melhores a matemticos da poca, Niccolo Fontana Tartaglia (1499 - 1557). Tartaglia, a e acostumado `s vicissitudes da vida, sabia que os lobos estavam ` porta. Em a a pouco tempo resolveu o problema das cbicas comprimidas e foi alm. Descobriu u e uma maneira de resolver equaes do tipo co x3 + ax2 = c, (onde a e c so constantes positivas) e conseguiu vencer o desao de Fior. a Veja como bonito o truque de Tartaglia. Para resolver a equao e ca ax3 + bx2 + cx + d = 0 b . Isso dever eliminar o termo quadrado, recaindo em a 3a uma cbica comprimida. Por exemplo, para resolver a equao u ca substitua x por y x3 + 6x2 + 18x + 10 = 0, fazemos a substituio x = y 2. Assim: ca (y 2)3 + 6(y 2)2 + 18(y 2) + 10 = = y 3 6y 2 + 12y 8 + 6y 2 24y + 24 + 18y 36 + 10 = = y 3 + 6y 10.

19.1 O segredo revelado e


Est na hora de entrar na histria um personagem muito interessante. Giroa o lamo Cardano (1501 - 1576) teve uma vida, no m nimo, pitoresca. Ele deixou uma autobiograa na qual narra suas muitas venturas e desventuras. Cardano interessava-se por muitos assuntos, entre eles matemtica. E claro que ele esa tava interessado na soluo da cbica. Em 1539, Cardano recebeu Tartaglia em ca u sua casa para uma longa visita. Ele s conseguiu que Tartaglia lhe revelasse o o segredo da cbica aps ter jurado que guardaria o segredo para si. u o 74

UD 5

Histria da Matemtica o a

Assim, com a ajuda de Tartaglia, Cardano e seu aluno, Ludovico Ferrari (1522 - 1565), dominaram o mtodo de resolver equaoes cbicas. Mas, se voc e c u e no quer que seus segredos sejam revelados, no os conte a ningum! Em 1545 a a e Cardano publicou um livro, chamado Ars Magna (Arte Suprema), em que revela o segredo to ciosamente guardado por Tartaglia. a E preciso dizer que Cardano deu crdito a Tartaglia. Alm disso, s publicou o e e o livro aps ter visto a soluo parcial divisada por Scipione, em alguns de seus o ca papis pstumos. Alm disso, Cardano e Ferrari haviam feito alguns progressos. e o e Ferrari, por exemplo, havia descoberto a soluo para equaes do quarto grau ca co e isso fora inclu no livro de Cardano. De qualquer forma, isso criou uma do terr disputa com Tartaglia. Apesar de tudo, a divulgao da informao vel ca ca deu um enorme impulso no desenvolvimento da Matemtica. Cardano foi o a primeiro matemtico a divisar os nmeros complexos. Isso lhe permitiu dar um a u tratamento mais completo `s cbicas. a u

19.2 Notao matemtica ca a


Apesar das confuses causadas pelos personagens desse episdio, a soluo o o ca das cbicas mostrou que a Matemtica estava pronta para vos mais altos. u a o Nesse per odo destacam-se, tambm, os trabalhos de Franois Vi`te (1540 e c e 1603). Vi`te advogado e matemtico amador, mas suas contribuies so e e a co a muito importantes. Ele foi o responsvel pelo uso de letras para representar a incgnitas e usou trigonometria para resolver equaes de grau maior do que o co 3 e 4. Foi nesse per odo que se passou a usar os s mbolos =, + e . Com o aperfeioamento das notaes algbricas, as precises dos clculos algbricos c co e o a e ganham um estgio bem avanado e uma grande aventura estava por vir. a c Um passo gigantesco foi dado no momento em que os matemticos passaram a a usar s mbolos como =, + e , combinados com letras para representar valores a serem determinados, causando o surgimento das equaes. Fora uma co caminhada milenar para se atingir essa etapa da Matemtica. a E preciso ter isso em mente quando vemos alunos tendo diculdades em resolver um problema literal. Pense um pouco nisso enquanto voc se diverte e resolvendo o seguinte problema. 75

UD 5

Histria da Matemtica o a

Atividade 23
Ningum sabe exatamente quando nasceu ou morreu Diofanto. No entanto, e sabemos quantos anos ele viveu, devido ao enigma elaborado por um de seus disc pulos para descrever a vida do mestre:
A juventude de Diofanto durou 1/6 de sua vida; depois de mais 1/12, nasceu-lhe a barba. Ao m de mais 1/7 de sua vida, Diofanto casou-se. Cinco anos depois teve um lho. O lho viveu exatamente metade do que viveu o pai, e Diofanto morreu quatro anos depois da morte de seu lho. Tudo isso somado o nmero de anos que Diofanto viveu. e u

Atividade 24
Use o truque de Tartaglia para transformar a equao de Fibonacci x3 + 2x2 + ca 26 704 10x = 20 na equao y 3 + y = ca . Agora, use a soluo de Scipione ou ca 3 27 Cardano, para cbicas comprimidas, e calcule a raiz u
3

352 + 6 3930 +

352 6 3930 2 = 1.368808108.

Atividade 25
Resolva a equao y 3 15y = 4 usando a soluo de dal Ferro, com s e t. ca ca Depois, observando que 4 uma soluo, encontre as outras respostas usando e ca a frmula de Bhaskara. Como voc explica a diferena encontrada? o e c

76

Unidade 6
Uma Nova Matemtica para um Mundo Novo a
Nesta unidade didtica voc ver como a descoberta do Clculo introduziu a e a a vida nova ` Matemtica e mudou, denitivamente, o cenrio a a a cient co mundial. No entanto, imaginar que o surgimento dessa nova ferramenta matemtica a tenha ocorrido independentemente do contexto cient co e cultural , no e m nimo, ingnuo. e O sculo 16 assistira a descobertas e avanos cient e c cos impressionantes, mas nada que se compare com o que estava por vir.

Texto 20: Sobre os Ombros de Gigantes


O mundo se expandira, era a poca das grandes navegaes, que ocorreram com e co a ajuda de novos instrumentos e desenvolvimentos ocorridos na cartograa. Precisamos lembrar que a imprensa havia sido inventada por Gutenberg na metade do sculo 15 e isso dera um grande impulso ` difuso de informaes e e a a co conhecimento. O mundo vivia o momento histrico conhecido como Renascimento, ocorrido nas o artes e nas cincias. Um personagem t e pico desse per odo foi Luca Pacioli (1445 - 1517), um franciscano e matemtico amigo de Leonardo da Vinci. Pacioli teve a um de seus livros, o Divina proportione, sobre poliedros regulares, ilustrado pelo famoso artista. Na Matemtica, a descoberta de mtodos algbricos para a resoluo das a e e ca equaes cbicas e qurticas, resultado dos esforos de dal Ferro, Tartaglia, co u a c 77

UD 6

Histria da Matemtica o a

Cardano e Ferrari, assim como os trabalhos de Franois de Vi`te, fazendo proc e gressos na parte da notao matemtica, dava nimo aos outros matemticos ca a a a e preparava o terreno para novas descobertas. Nessa poca, o desenvolvimento tecnolgico passou a exigir da Matemtica e o a respostas para seus prprios problemas. Por exemplo, questes sobre reas e o o a volumes, associadas ao clculo de centros de gravidade, motivaram Luca Valrio a e (1552 - 1618) a aprofundar os mtodos desenvolvidos pelos antigos gregos. e Era o momento de Galileu Galilei (1564 - 1642) fazer pesquisas sobre a queda livre dos corpos, realizadas com genialidade, e mudar denitivamente a maneira de se produzir cincia. Foi ele quem apontou o telescpio para os cus, tornando e o e um brinquedo em uma ferramenta cient ca, sendo o primeiro ser humano a vislumbrar a grandeza do cosmo. Outro gigante das cincias foi Johannes Kepler (1571 - 1630) que revelou aos e homens as leis que regulam o funcionamento do sistema solar, decifrando um mistrio milenar. e A primeira lei de Kepler arma que as rbitas planetrias so elipses nas quais o a a o sol ocupa um dos focos. (Nada de c rculos, ou c rculos se revolvendo sobre outros c rculos, a soluo no era euclidiana, no sentido de rgua e compasso.) ca a e A segunda arma que o segmento (imaginrio) que une o sol ao planeta descreve a reas (setores el a pticos) iguais em tempos iguais. Isso explica por que os planetas aceleram quando se aproximam do sol e diminuem sua velocidade quando dele se afastam. Vale a pena ouvir, pelo menos uma vez, o canto de vitria de Kepler, escrito o no prefcio do seu Harmonices mundi, descrevendo a intensidade de seus sentia mentos ao vislumbrar sua descoberta:
Fui iluminado, em meio a uma contemplao muito admirvel, h dezoito ca a a meses por um primeiro claro, h trs meses por uma claridade diferente a a e e h poucos dias pelo prprio sol. a o

Veja, Kepler estava consciente da importncia de sua descoberta, mas sabia de a que ela encontraria resistncia em alguns setores. A virada do sculo 16 para 17 e e assiste ` mudana do eixo de produo matemtica da Itlia para mais ao norte a c ca a a da Europa. A necessidade de um ambiente onde h liberdade de pensamento a para a produo cient ca ca explica, em parte, esse fenmeno. E bom lembrar o 78

UD 6

Histria da Matemtica o a

que Galileu foi forado a renegar suas idias e terminou seus dias isolado. c e O sculo 17 foi inaugurado com um avano matemtico notvel a introduo, e c a a ca aperfeioamento e utilizao de uma ferramenta matemtica extraordinria c ca a a o logaritmo. Isso foi resultado dos esforos conjuntos de dois britnicos: John c a Napier (1550 - 1617) e Henry Briggs (1561 - 1631). E um pouco dif para um jovem estudante de Matemtica, que pode comcil a prar uma calculadora cient ca em qualquer esquina, entender o porqu de toda e essa importncia. No entanto, os logaritmos transformam multiplicaes e dia co vises em somas e diferenas, respectivamente, ajudando imensamente todas as o c atividades cient cas que demandavam clculos elaborados, como a astronomia. a A motivao para essa descoberta fora a comparao entre a srie aritmtica e ca ca e e a srie geomtrica, colocadas em direta correspondncia por Michael Stiefel no e e e livro Arithmetica Integra, de 1544: 0 1 1 2 2 4 3 8 4 16 5 32 6 64 7 128 8 256 ... ...

observando que a soma na seqncia da linha superior correspondia ao produto ue na srie da linha inferior. Stiefel se referia aos nmeros de cima como os e u expoentes dos nmeros de baixo. u Voc se lembra do famoso algoritmo para extrair ra quadradas? Veja que e zes extrair signica tirar sob duras penas... Muito bem, sem usar a calculadora, marcando no relgio, depois de passar uma meia hora treinando para o relembrar, realize, ento, o exerc proposto. a cio

Atividade 26
Calcule 13457.29.

Agora, quanto tempo voc levaria para dividir 9.507276246 por dois? Portanto, e se voc dispusesse de uma maneira de transformar 13457.29 em 9.507276246 e, e aplicando o processo inverso, destransformar a metade deste nmero de volta, u voc ainda tentaria usar o algoritmo da raiz quadrada? e Veja, ln 13457.29 = 9.507276246, 9.507276246/2 = 4.753638123 e e4.753638123 = 116.0055602. Portanto, 13457.29 = 116.0055602. 79

UD 6

Histria da Matemtica o a

Note que as primeiras tabelas de logaritmo, constru das por Napier, no eram na a base 10. Em 1615, com a colaborao de Briggs, foram constru ca das as tabelas onde o logaritmo de 1 zero e logaritmo de 10 1. Isso , logaritmo na base e e e 10. A constante e, base do que chamamos logaritmo natural, foi introduzida posteriormente, por Euler.

Texto 21: Preldio do Clculo u a


No in do sculo 17 os matemticos voltaram a enfrentar um velho adversrio cio e a a o innito. Havia trs tipos de questes que se apresentavam: e o as somas innitas; o clculo de reas (problemas do tipo quadratura); a a o problema das tangentes a curvas dadas. A questo das somas innitas tem suas ra a zes no passado, j na poca dos a e gregos. Por exemplo, um dos paradoxos de Zeno pode ser colocado assim: a 1 1 1 1 1 + + + + + . . . = 1? 2 4 8 16 32 Durante a Idade Mdia, foram feitos vrios progressos nesse sentido. Por exeme a plo, vale a pena conhecer um pouco da histria do noruegus Nicole Oresme o e (1323 - 1382) que estudou na Universidade de Paris e foi amigo de longa data de Carlos V, rei da Frana. Oresme denitivamente considerava a igualdade c anterior verdadeira. Ele calculou a soma da srie e 4 5 n 1 2 3 + + + + + ... + n + ... 2 4 8 16 32 2 e foi o primeiro matemtico a provar a divergncia da srie harmnica a e e o 1 1 1 1 1 + + + + + ... 1 2 3 4 5 Isso um feito e tanto. Para mostrar que a srie diverge, preciso mostrar que, e e e dado um nmero qualquer R > 0, existe um nmero inteiro N , tal que u u
N n=1

1 1 1 1 1 = 1 + + + + + > R. n 2 3 N 1 N 80

UD 6

Histria da Matemtica o a

Veja, para ultrapassar o nmero R = 10, por exemplo, necessrio tomar u e a n = 12 367. Ou seja, a soma dos 12 366 primeiros termos da srie , aproximae e damente, 9.999962148. Observe como Oresme resolveu o problema: 1 1 1 1 1 + > + = ; 3 4 4 4 2 1 1 1 1 1 1 1 1 4 1 + + + > + + + = = ; 5 6 7 8 8 8 8 8 8 2 1 1 1 1 1 1 1 1 8 1 + + + ... > + + + + = = ; 9 10 11 16 16 16 16 16 16 2 1 1 1 1 1 1 1 1 16 1 + + + ... > + + + + = = . 17 18 19 32 32 32 32 32 32 2 Prosseguindo assim, tomando partes cada vez mais compridas da srie, seguimos e somando parcelas que, de 1/2 em 1/2, ultrapassam qualquer nmero R > 0. u E claro que isso toma muitos termos, mas temos uma quantidade inesgotvel a deles.

Atividade 27
Repita um dos feitos de Oresme e calcule a soma a seguir. 4 5 n 1 2 3 + + + + + ... + n + ... 2 4 8 16 32 2

As questes do tipo clculo de reas e de volumes, assim como as questes o a a o sobre tangentes a curvas, ocuparam muitas mentes brilhantes. Novamente, a questo de lidar com innito ou innitsimos estava em pauta. a e Para o estudo do comportamento dos planetas, Kepler precisava determinar reas de setores el a pticos, mas tambm considerou, por razes mais prosaicas, e o o clculo do volume de barris de vinho. Sua abordagem era a de dividir, por a exemplo, um dado slido em um nmero innito de pedaos innitesimais, ou o u c slidos indivis o veis, de um tamanho ou forma conveniente para o problema. Outro matemtico que marcou essa poca foi Bonaventura Cavalieri (1598 a e 1647), que produziu dois alentados volumes: Geometria indivisibilibus continuorum (Geometria dos Indivis veis), de 1635, e Exercitationes geometricae sex 81

UD 6

Histria da Matemtica o a

(Seis Exerc cios Geomtricos), de 1647. A abordagem de Cavalieri era diferente e da de Kepler. Veja o chamado Princ de Cavalieri: pio
Dois slidos com a mesma altura, que tm suas sees planas de mesmo o e co n com as mesmas reas, tm o mesmo volume. vel a e

Cavalieri calculou o volume da esfera de raio R, comparando com o volume do cilindro de raio R e altura 2R menos dois cones de altura R e raio R. Para isso, basta mostrar que a rea do disco contido na esfera igual ` rea da coroa a e aa contida no cilindro menos o par de cones, na mesma altura. Veja na gura a seguir.

Como o volume do cilindro 2 R3 e o volume de cada cone R3 /3, o e e volume da esfera 2 R3 e 2 R3 (6 2) R3 4 R3 = = . 3 3 3

Para ter certeza de que a conta est correta, observe o diagrama a seguir. a

h r

R h h R

O raio r, do disco de n h, na esfera, satisfaz a relao R2 = r2 + h2 , vel ca devido ao tringulo retngulo da gura da esquerda. Portanto, a rea deste a a a disco r2 = (R2 h2 ). Por outro lado, a coroa circular consiste do disco e de raio R menos o disco de raio h, como mostra o tringulo issceles da gura a o da direita. Sua rea , portanto, R2 h2 . a e 82

UD 6

Histria da Matemtica o a No in do sculo 17, a Frana produziu algumas pessoas geniais que cio e c contribu ram fortemente para a criao do clculo. ca a Vamos conhecer algumas delas.

Texto 22: A Conexo Francesa a


Ren Descartes (1596 - 1650) conhecido pela frase Penso, logo existo e pelo e e livro chamado Discurso sobre o Mtodo para Bem Conduzir a Razo a Buscar e a a Verdade Atravs da Cincia. Descartes proporcionou aos matemticos uma e e a experincia riqu e ssima. Se duas reas da Matemtica so poderosas, juntas so a a a a imbat veis. Descartes uniu equaes `s curvas, criando a geometria anal co a tica. Estava criado o sistema cartesiano, palco de tantas aes matemticas. Isso co a tudo fazia parte de um apndice do Discurso, chamado Geometria, que era e dividido em trs partes. e Por exemplo, no segundo desses livros, Descartes considera as equaes do tipo co F (x, y) = ax2 + 2hxy + by 2 + 2gx + 2f y + c = 0 e descreve as condies para que elas representem elipses, hiprboles ou parbolas. co e a Ele mostra, ainda, como determinar tangentes a uma dada cnica. o As questes de encontrar tangentes estavam, por assim dizer, no ar. o Contemporneo de Descartes, Pierre de Fermat (1601 - 1665) cou conhecido a como Pr ncipe dos Amadores, pois se dedicava ` Matemtica nas horas de a a folga. Fermat era um conselheiro do parlamento da cidade de Toulouse, no sul da Frana, e nunca publicou um s artigo de Matemtica em toda a vida. Ele c o a divulgava suas descobertas atravs de correspondncia com outros matemticos e e a e muitas outras de suas contribuies s foram descobertas aps sua morte. co o o Como Fermat no tinha um compromisso formal, por assim dizer, com a Maa temtica, muitas de suas descobertas eram divulgadas apenas de forma fraga mentada. A histria mais famosa devido a coisas como essa a do chamado o e Ultimo Teorema de Fermat. Tudo comeou com uma nota que ele escreveu c em sua cpia do Aritmtica, de Diofanto, junto da Proposio II.8, sobre a o e ca expresso de um quadrado como a soma de dois outros quadrados. Isso , a e h nmeros inteiros quadrados que so soma de dois outros nmeros inteiros a u a u quadrados, como 52 = 32 + 42 ou 132 = 52 + 122 . 83

UD 6

Histria da Matemtica o a
E imposs dividir um cubo em dois cubos, ou uma quarta potncia vel e em duas potncias de quatro, ou geralmente qualquer potncia maior do e e que o quadrado na soma de duas iguais potncias; e eu encontrei uma e demonstrao admirvel para esse fato, mas essa margem muito estreita ca a e para cont-la. e

A armao pode ser traduzida da seguinte forma: no existem trs nmeros ca a e u inteiros x, y e z tais que xn + y n = z n , para n inteiro maior do que 2, e deveria ser chamada Conjectura de Fermat, pois no havia demonstrao. a ca Poucas vezes na histria uma questo matemtica desaou tanto a inventividade o a a de matemticos prossionais e amadores. A facilidade do enunciado certamente a contribuiu para atrair o interesse de tantas pessoas, ocultando, no entanto, a enorme diculdade do problema. Bem, no havia demonstrao, mas agora h. Em 1993 Andrew Wiles proferiu a ca a uma palestra em Cambridge apresentando os resultados de suas pesquisas. O Teorema de Fermat estaria demonstrado como conseqncia. No entanto, falhas ue na demonstrao de Wiles deixavam de fora alguns casos especiais, entre eles ca o que provaria o Teorema de Fermat. Como num lme de suspense, o vilo a se levantava mais uma vez. Finalmente, em 1994, novos argumentos foram apresentados por Wiles e Richard Taylor cobrindo todos os casos, inclusive o que demonstrava o Teorema de Fermat. Para saber os detalhes dessa histria o realmente maravilhosa, no deixe de ler o livro O Ultimo Teorema de Fermat, a de Simon Singh. Fermat deixou sua marca na teoria de nmeros e tambm foi o co-responsvel u e a pelos fundamentos da probabilidade, devido `s suas contribuies atravs de a co e cartas que trocou com Blaise Pascal (1623 - 1662), outro grande matemtico a daqueles dias. Seus progressos no desenvolvimento do que chamamos clculo a diferencial foram muitos. Ele percebeu, por exemplo, que a noo de reta ca tangente a uma dada curva poderia ser usada para detectar pontos de mximo a ou de m nimo. Veja como Fermat teria resolvido o problema a seguir. Calcule as propores do cilindro reto de maior volume que pode ser inscrito co numa esfera de raio R. 84

UD 6

Histria da Matemtica o a

Esse problema encantaria qualquer um dos matemticos gregos do passado, mas a eles no disporiam de qualquer mtodo que permitisse encontrar a resposta. a e Vamos denotar o raio da base do cilindro por r e sua altura por h. Usando o Teorema de Pitgoras, temos R2 = r2 + a h2 . Isolando o valor de r2 nessa 4 frmula e substituindo em r2 h, o volume do cilindro, obtemos a frmula do o o volume do cilindro em termos apenas de sua altura: V (h) = R2 h2 4 h.

De posse dessa equao, Fermat calcularia a inclinao da reta que contm os ca ca e pontos (h, V (h)) e (h + a, V (h + a)), como estamos habituados a fazer nos cursos de Geometria Anal tica, obtendo
V (h + a) V (h) a R2 =
2

R h/2 r

(h + a)2 4

(h + a) R2 a

h2 4

h
Corte da esfera com cilindro reto inscrito por um plano contendo a origem.

= = =

(4R (h + a) (h + a) 4R + h ) = 4a (4R2 h + 4R2 a h3 3ah2 3ha2 a3 4R2 h + h3 ) = 4a (4R2 3h2 3ha a2 ). 4

Para determinar a inclinao da reta tangente no ponto (h, V (h)), basta fazer ca (4R2 3h2 ) . O ponto mximo ocorre onde essa inclinao a ca e 4 zero, ou seja, 4R2 3h2 = 0. Portanto, o cilindro de volume mximo inscrito a 2 3R 6R na esfera de raio R tem altura h = e raio r = . 3 3 a = 0, obtendo O fato da equao que dene o volume do cilindro em termos da sua altura ser ca polinomial foi crucial para o truque funcionar. Apesar de estarmos calculando uma derivada e igualando-a a zero, em nenhum momento usamos a noo de ca limite! Isso cou escondido na passagem em que trocamos o a por zero, aps o termos feito todas as fatoraes. co Alm de Descartes e Fermat, Pascal tambm dedicou sua engenhosidade ` e e a resoluo de problemas de clculo. Ele comeou sua carreira matemtica na ca a c a 85

UD 6

Histria da Matemtica o a

geometria, provando, aos 16 anos, um teorema sobre a colinearidade dos pontos de interseo dos lados opostos de um hexagrama m ca stico inscrito em uma cnica. o
Se um hexgono ADBF CE (no necessariamente convexo) for inscrito a a em uma cnica (como um c o rculo ou uma elipse), ento os pontos de a interseo das retas que conectam vrtices opostos (AD com F C, DB ca e com CE e BF com EA) so colineares. Essa reta chamada reta de a e Pascal do hexaedro.

Veja, nas guras a seguir, duas possibilidades: hexagrama inscrito numa elipse e hexagrama inscrito numa hiprbole. e
B C F E D A B C

F E

Aos 18 anos construiu o primeiro computador de que se tem not cia, chegando a produzir e vender cerca de cinqenta mquinas. No por nada que chamau a a e mos Pascal uma das primeiras linguagens desenvolvidas para programao de ca computadores. Pascal dedicou algumas semanas de sua vida estudando a ciclide, nome dado o por Galileu Galilei ` curva obtida do trao de um ponto do bordo de um c a c rculo rolando ao longo de uma reta.

Seria injusto terminar esse momento em que falamos de matemticos francea ses geniais desse per odo sem mencionar Girard Desargues (1591 - 1661). Ele 86

UD 6

Histria da Matemtica o a

fundou as bases da geometria projetiva, mas seus trabalhos s ganharam recoo nhecimento posteriormente. Como voc pde ver, havia uma profuso de resultados do tipo clculo e o a a diferencial (problemas de tangentes a curvas) e integral (questes de clculo o a de reas e volumes). Muitos outros matemticos, como Evangelista Torricelli a a (1698 - 1647), Gilles Persone de Roberval (1602 - 1675), John Wallis (1616 1703) e Isaac Barrow (1630 - 1677) deram suas contribuies. Em particular, co Barrow foi o primeiro a notar a conexo que h entre essas duas questes. a a o

Texto 23: Newton e Leibniz dois gnios e uma idia! e e


O fato de que a rea abaixo da curva y = xn , de 0 at a ser a e inclinao da reta tangente ` curva y = ca a no lhe passou despercebido. a No entanto, dois personagens so reconhecidos como os criadores do Clculo. A a a razo disso est no escopo de suas abordagens. Eles no se ativeram a resolver a a a um ou outro problema espec co, mas desenvolveram, cada um a sua maneira, mtodos gerais de lidar com essas questes. Nisso reside o mrito de suas e o e contribuies. co an+1 e de que a n+1

xn+1 , no ponto de abscissa a, ser an , n+1

23.1 Anni mirabili


Os anos de 1666 e 1667 foram particularmente dif ceis para os ingleses. Uma terr peste, a peste bubnica, abateu-se sobre a Inglaterra, forando, incluvel o c sive, o fechamento temporrio das universidades de Oxford e Cambridge. Esse a per odo de recolhimento foi, no entanto, prop para as cincias. Um estucio e dante de Cambridge retornou para a casa de seus avs, que cava na zona rural o de Woolsthorpe, Lincolnshire. Esse jovem de 24 anos produziu ento uma srie a e de resultados cient cos que mudariam, de maneira dramtica e denitiva, o a panorama das cincias. e Seu nome era Isaac Newton (1642 - 1727). Entre as descobertas feitas por Newton neste per odo, que cou conhecido como anni mirabili (anos miracuIsaac Newton

87

UD 6

Histria da Matemtica o a

losos), temos uma generalizao do Teorema Binomial, a Teoria da Gravitao ca ca e a anlise da natureza da luz. a A importncia dos trabalhos de Newton reside na praticidade de seus mtodos a e e no tipo de problemas que ele pde resolver. Por exemplo, partindo das leis o de gravitao e usando os mtodos de clculo por ele desenvolvidos, pde deca e a o monstrar as leis de Kepler. Um exemplo da viso de Newton sua generalizao do Teorema Binomial. a e ca Esse teorema descreve como obter a expanso do binmio (a + b)n , para todo a o inteiro positivo n. Veja, (a + b)2 = a2 + 2ab + b2 , (a + b)3 = a3 + 3a2 b + 3ab2 + b3 e assim por diante. Para determinar os coecientes dessas expanses, basta tomar a correspondente o linha no chamado tringulo de Pascal: a

1
Na verdade, esse arranjo de nmeros era conhecido desde u muito antes de Pascal. Por exemplo, Omar Khayyam, matemtico rabe, e Chu a a Shih-Chieh, matemtico a chins que o menciona no e livro O Espelho Precioso dos Quatro Elementos, por volta de 1303.

1 1 1 1 1 1 6 5 15 4 10 20 3 6 2

1 1 3 4 10 15 5 6 1 1 1 1

Para obter a prxima lista, basta comear com o nmero 1 e seguir somando o c u os dois nmeros logo acima da posio a ser preenchida. Ela seria 1, 7, 21 e u ca assim por diante. O jovem Newton percebeu que poss determinar os coecientes diretamente, e vel sem a construo do tringulo linha por linha at chegar ` potncia desejada. ca a e a e Algo como fazemos agora com a frmula para determinar o coeciente do termo o 88

UD 6 ani bi : C(n, i) =
n i

Histria da Matemtica o a

n! . (n i)!i!

Dessa forma, para expandir (1 + x)3 , fazemos (1 + x)3 = 1 + 3x + 32 2 321 3 3210 4 x + x + x + ... 2 32 432

observando que, a partir do termo de grau 4, os coecientes se anularo. a Newton observou que essa frmula vale para expoentes fracionrios e negativos. o a Para calcular (1 + x)3 , ele faria 1 + (3)x + obtendo 1 = 1 3x + 6x2 10x3 + 15x4 . . . (1 + x)3 A diferena, agora, que o lado direito da igualdade uma soma innita. Ele c e e conrmou sua descoberta observando que (1 + 3x + 3x2 + x3 )(1 3x + 6x2 10x3 + 15x4 . . . ) = 1. Newton no observou que essa expresso s vale para valores de x no intervalo a a o (1, 1). Mas essas computaes eram mais do que s interessantes. Eram co o uma poderosa ferramenta de clculo. Usando esse teorema binomial, obteve a a expanso em srie da funo y = 1 x = (1 x)1/2 , obtendo a e ca 1 1 5 4 7 5 1 x x ... 1 x = 1 x x2 x3 2 8 16 128 256 Por exemplo, usando esses termos da srie, podemos calcular uma aproximao e ca para 3. Primeiro, precisamos de um truque. Observe que 3 = 4 1. Assim, 3= 4 1 1 4 = 2 1 1 e 4 = 2 227025 = 1.732063293. 262144 (3)(4) 2 (3)(4)(5) 3 x + x + ... 2 23

1 1 5 7 1 3 2 1 = 8 128 1024 32768 262144 89

UD 6

Histria da Matemtica o a

No deixe de comparar essa aproximao com aquela que voc pode obter a ca e usando uma simples calculadora cient ca. Veja que usamos apenas 6 termos da srie. e Em nossa viagem no tempo e espao, estivemos em inmeras regies. Assim, c u o chegamos ` segunda metade do sculo 17, mais especicamente na Alemanha. a e

23.2 Leibniz entra em cena


Alguns anos depois, entre 1673 e 1676, um outro gnio produziu sua verso e a do Clculo. Este foi Gottfried Wilhelm Leibniz, que comeara sua carreira a c como diplomata. Ele fora atra para a Matemtica graas ` inuncia de do a c a e Cristian Huyggens, a quem conhecera em Paris enquanto estava em uma misso a diplomtica. a Newton e Leibniz, bem como seus seguidores, se envolveram em uma polmica e sobre a originalidade da descoberta do Clculo. Isto causou grande desgaste a
Gottfried Wilhelm Leibniz (1646 - 1716) nasceu em Leipzig, Alemanha. Alm do e Clculo, Leibniz deu grandes a contribuioes no campo da c lgica. o Para saber mais sobre este tema, voc pode ler o e cap tulo Newton e Leibniz Um Choque de Tits, do a livro Grandes Debates da Cincia, de Hal Hellman, e Editora Unesp, 1998.

pessoal a cada um. A verdade que suas abordagens foram diferentes, levados e por motivaes outras. Newton apresenta o Mtodo das Fluxes como uma ferco e o ramenta que lhe permite aprofundar seus conhecimentos dos fenmenos f o sicos. Isto , uma viso cinemtica do Clculo: a derivada vista como uma taxa de e a a a variao. Newton considerava x e y variando, uindo, em funo do tempo. ca ca Leibniz, por sua vez, considerava x e y variando sobre uma seqncia de valores ue innitamente prximos. Ele introduziu dx e dy como sendo as diferenas entre o c os valores nesta seqncia. ue

23.3 O clculo diferencial e integral a


Newton via a integrao como um problema de encontrar os x e y de uma ca determinada uxo. Isto , encontrar o deslocamento de uma dada velocidade. a e Portanto, para ele, a integrao era, naturalmente, o processo reverso da dica ferenciao. Leibniz via a integrao como uma soma, no estilo que zeram, ca ca antes dele, Arquimedes e Cavalieri. Leibniz foi feliz em utilizar os innitsimos e dx e dy onde Newton usou x e y , ou seja, velocidades. Leibniz usava a palavra mnada para indicar algo to simples que no tem partes. Nenhum deles o a a considerava o que denominamos funes, pois este conceito s foi introduzido co o muitos sculos depois. No entanto, ambos, denitivamente, pensavam em tere 90

UD 6

Histria da Matemtica o a

mos de grcos. De qualquer forma, estavam travando uma luta com o innito, a no caso, o innitamente pequeno. Apesar de Newton ter desenvolvido sua teoria primeiro, coube a Leibniz o mrito e de ter publicado sua verso, em 1684, introduzindo o termo calculus summaa torius, e divulgando suas idias. Leibniz dava muita importncia ` notao, no e a a ca que estava absolutamente certo. Leibniz introduziu os s mbolos matemticos d e a de 1675, a notao ca x dx = exatamente como ns o fazemos at hoje. o e Assim chegamos ao m da unidade didtica com pelo menos uma viso global a a da enorme odissia que foi o descobrimento do Clculo. Newton e Leibniz se e a sobrepuseram a seus contemporneos devido ao escopo de suas descobertas. a Conseguiram ver alm dos outros matemticos. Pudera, eles se encontravam e a sobre os ombros de gigantes, como diria Newton. De qualquer forma, muito ainda estava por ser feito e os novos desbravadores j estavam se preparando. Os irmos Jakob e Johann Bernoulli e, a a principalmente, Leonhard Euler, continuariam a descobrir e a usar o Clculo a por todo o sculo 18 e s no sculo 19 Cauchy e Weierstrass colocariam toda e o e a teoria em bases slidas, como a estudamos hoje. o Mas, para contar um pouco sobre essa parte da histria, devemos esperar as o prximas unidades didticas. o a x2 , 2 , estabelecendo, por volta

91

Unidade 7
A equao de Euler: ei + 1 = 0 ca
Nesta unidade didtica voc conhecer alguns dos triunfos de Leonhard Euler, a e a um dos matemticos mais produtivos de todos os tempos e a o mais importante do sculo 18. e Ver como os desaos matemticos continuavam estimulando a criatividade a a dos matemticos, indicando que a sede de resolver problemas continuava, a entre os matemticos, forte como sempre. a Iniciamos mostrando como as equaes matemticas passaram a ocupar co a espao no cenrio da Matemtica, que comeava a car c a a c mais sinttica e menos literal. e

Texto 24: A Essncia da Matemtica e a


O nmero de outubro de 2004 da revista Physics World traz um artigo intitulado u The greatest equations ever, algo assim como As maiores equaes de todos co os tempos. O artigo resultado de uma pesquisa proposta aos leitores da e revista, pedindo indicaes de suas equaes preferidas, acompanhadas de uma co co justicativa da escolha. Entre as mais indicadas estavam equaes bem conhecidas e, tambm, surpreco e sas. Veja algumas.

1+1=2

E = m c2
93

F = ma

UD 7

Histria da Matemtica o a

A equao 1+1 = 2 foi indicada, por exemplo, por sua simplicidade, enquanto as ca outras duas tm a popularidade garantida devido ` sua importncia no mundo e a a da F sica. A equao E = m c 2 , formulada por Einstein, expressa a relao ca ca entre a massa e a energia, no contexto da Teoria da Relatividade e F = m a e a formulao da Segunda Lei de Newton, relacionando a acelerao provocada ca ca por uma fora atuando sobre um corpo. c O artigo tambm provocou uma discusso sobre o signicado do termo equao. e a ca Mais especicamente, as pessoas queriam saber a diferena entre termos como c equao, frmula e teorema. Como se trata de nomenclatura, a discusso tem ca o a uma importncia relativa, mas interessante. Por exemplo, bem provvel que a e e a muitas pessoas respondam com a equao a2 = b2 + c2 ao pedido de citar o ca Teorema de Pitgoras. a E verdade que a frmula no o Teorema de Pitgoras, no sentido que a o a e a igualdade pode no ser satisfeita caso a, b e c no sejam os comprimentos a a dos lados de um tringulo retngulo. E, mesmo que fossem, a igualdade s se a a o cumpre se a for o comprimento do lado maior, a hipotenusa. Mas, implicncias matemticas ` parte, no podemos negar que a a a a a2 = b2 + c2 carrega, em si, a essncia do teorema. e Outro exemplo vem da frmula de Euler para poliedros convexos, o V A+F = 2
Heinrich Hertz (1857-1894), f sico alemo que demonstrou a pela primeira vez, em 1888, a existncia de radiao e ca eletromagntica, construindo e um aparelho que produzia ondas de rdio. Seu nome a e usado para denotar a unidade de freqncia que um ue determinado evento repetitivo, como o som ou ondas eletromagnticas, e ocorre por unidade de tempo. 1 Hz (um hertz) indica que o evento ocorre uma vez por segundo.

relacionando algebricamente o nmero de seus vrtices, arestas e faces. u e Uma das citaes famosas sobre esse tema de Hertz: co e
E imposs evitar o sentimento de que essas frmulas matemticas tm vel o a e uma existncia independente e uma inteligncia prpria, que elas so mais e e o a sbias do que ns, mais sbias mesmo que seus descobridores, que consea o a guimos delas algo mais do que foi originalmente colocado nelas.

A frase de Hertz tem um extra signicado quando notamos que entre as mais indicadas na pesquisa estavam as equaes de Maxwell, que descrevem como um co campo eletromagntico varia no espao e no tempo. Vale a pena dar uma olhada e c 94

UD 7

Histria da Matemtica o a

nessas equaes, mesmo que no esteja em nossos planos nos aprofundarmos co a em tal direo. ca

E =

B = 0 E = B t
0 0

James Clerk Maxwell (1831-1879), f sico britnico a que explicou as propriedades do eletromagnetismo. Publicou um conjunto de quatro equaes diferenciais co nas quais descreve a natureza dos campos eletromagnticos e em termos de espao e c tempo.

B = 0 J +

E t X denota

Nestas equaes, E o campo eltrico, B o campo magntico, co e e e o divergente do campo X, grandezas esto representadas no sistema de unidades mks. a

X o seu rotacional e, bem, assume-se que as

As duas primeiras equaes dizem que o campo eltrico e o campo magntico co e e satisfazem ` Lei de Gauss. a Apesar de terem uma estrutura relativamente simples, as equaes de Maxwell co nos permitiram uma nova perspectiva da natureza, unicando eletricidade e magnetismo. Essa teoria enlaou a F c sica e a Matemtica de uma maneira a inovadora, fazendo mais do que descrever os fenmenos eletromagnticos. Tal o e descoberta afetaria a maneira de produzir tanto Matemtica quanto F a sica. O tema interessante mas, adiantamo-nos. Essas equaes so chamadas difee co a renciais e foram formuladas no sculo 19. Nesta unidade didtica, entretanto, e a vamos falar sobre acontecimentos ocorridos no sculo 18. Isso nos traz ` outra e a das equaes mais votadas: co

ei + 1 = 0.
Conhecida por equao de Euler, ela no descreve algum fenmeno especial da ca a o natureza, no dita uma certa identidade vlida para quaisquer grandezas, como a a a famosa sen2 + cos2 = 1, ou a concluso de um grande teorema, como e a a equao a2 = b2 + c2 , concluso do Teorema de Pitgoras. No entanto, ela ca a a tem um sentido simblico forte. o 95

UD 7

Histria da Matemtica o a

A equao rene nove conceitos bsicos de Matemtica numa s expresso. ca u a a o a Temos: e, a base dos logaritmos naturais; a operao elevar ao expoente; a ca constante ; a multiplicao; nmeros complexos; a operao de soma, repreca u ca sentada por +; o nmero 1; igualdade; o nmero 0. u u A equao merece ser citada no Guinness. (Adivinhe qual das equaes citada ca co e nesse livro!) Na verdade, a equao simboliza a diversidade, reunindo numa frase to curta ca a tantas diferentes reas matemticas. Alm do mais, um tributo a um homem a a e e genial, Leonhard Euler (1707 - 1783), que com sua simplicidade, criatividade e muito trabalho contribuiu de maneira singular para o desenvolvimento da Matemtica. a E por essa razo que consideramos as equaes a essncia da Matemtica. a co e a

24.1 Os irmos Bernoulli a


Comeamos a descrever o desenvolvimento matemtico ocorrido no sculo 18 c a e fazendo meno a dois membros de uma famosa fam de matemticos: os ca lia a irmos Jakob e Johann Bernoulli, precursores do genial Euler. a O mais velho dos irmos Bernoulli, Jakob (1645 - 1705), deu enormes contria buies ao desenvolvimento do Clculo, das sries (somas innitas) e, especico a e almente, ` probabilidade. Seu principal trabalho foi publicado em 1713, sob o a nome de Ars Conjectandi (Arte de Conjecturar ), sobre probabilidade. Jakob publicou uma prova da divergncia da srie harmnica: e e o
k=1

1 1 1 1 1 = 1 + + + + + + ... k 2 3 4 k

num livro de 1689, atribuindo, no entanto, a soluo do problema ao irmo ca a Johann Bernoulli (1667 - 1748). E interessante observar que o argumento apresentado diferente daquele usado por Nicole Oresme, mais do que dois e sculos antes. Alm dos argumentos dados por Oresme e pelos Bernoulli, o e e italiano Pietro Mengoli (1625 - 1686) apresentou uma terceira demonstrao, ca que antecedeu ` de Johann por quarenta anos. a Isso mostra como a divulgao dos resultados matemticos era deciente. Com ca a o passar do tempo isso melhoraria muito. De fato, os principais matemticos do a 96

UD 7

Histria da Matemtica o a

sculo 18 estavam ligados a alguma academia de cincia. As mais importantes e e eram a de Paris, Berlim, So Petersburgo e Londres. Essas academias estavam a diretamente ligadas aos governantes daqueles pa ses, como Lu XV e Lu XVI, s s Frederico, o Grande, Catarina, a Grande. Johann Bernoulli tambm foi um grande matemtico e teve um papel de destae a que, juntamente com seu irmo, no desenvolvimento e divulgao do Clculo. a ca a Eles mantiveram freqente correspondncia com Leibniz. Johann foi professor u e de um nobre francs, o Marqus de lHpital (1661 - 1704), que publicou o prie e o meiro livro texto de Clculo, em 1696, chamado Analyse des inniment petits a pour lintelligence des lignes courbes (Anlise dos innitamente pequenos para a o entendimento de curvas). Esse livro apresentou a conhecida Regra de lHpital, um resultado obtido por o Johann Bernoulli, que permite calcular com desembarao alguns limites, garanc tindo que, se f (x) e g(x) so funes diferenciveis tais que f (a) = g(a) = 0 a co a e, g(x) = 0 para valores diferentes x de a, sucientemente prximos a a, o lim f (x) f (x) = lim xa g (x) g(x)

xa

desde que o limite da direita exista. Uma interrupo para um pouco de prtica. ca a

Atividade 28
Use a Regra de lHpital para calcular o limite a seguir: o lim x3 6x + 6 sen x . x5

x0

24.2 O desao da Braquistcrona o


Com o desenvolvimento do Clculo, surgiu uma nova forma de equao, que a ca serviria para modelar muitos problemas provenientes de outras reas cient a cas. 97

UD 7

Histria da Matemtica o a

So equaes cujas solues no so nmeros, mas o que ns chamamos funes. a co co a a u o co Naqueles dias podiam ser chamadas simplesmente curvas. Essas equaes so as co a chamadas equaes diferenciais. Por exemplo, as equaes diferenciais lineares, co co que estudamos ainda nos cursos de Clculo, tm a forma geral a e y + p(x) y = q(x). Um exemplo de problema desse tipo foi proposto por Johann Bernoulli, em 1696, num artigo publicado na revista Acta Eruditorum, editada por Leibniz. Na verdade, Johann props um desao a seus colegas matemticos, bem no o a estilo dos desaos havidos nos dias de Tartaglia, Cardano e Ferrari. O problema consistia em descobrir a identidade da curva que ele nomeou braquistcrona, o juno das palavras gregas brachistos, que quer dizer o mais curto, e chronos, ca que signica tempo.
Descubra a curva que une dois pontos P1 e P2 num plano vertical, de tal modo que um ponto material de massa m, deslizando sem atrito sobre essa curva, sujeito apenas ` gravidade, a percorra num intervalo de tempo a m nimo.

E claro que a tentativa mais primitiva consiste em tomar a reta que une os dois tais pontos. No entanto, essa no a resposta correta. a e
P1 t

d t P2

O prazo estipulado por Johann para que a resposta fosse apresentada terminava em 1 de janeiro de 1697. Ao m desse prazo, a unica resposta apresentada fora a de Leibniz. Na verdade, o grande desao estava estendido a Newton, que nesses dias andava ocupado com a direo da Casa da Moeda inglesa. Johann ca estendeu o prazo de seu desao at a Pscoa e fez questo de envi-lo at a e a a a e Inglaterra. 98

UD 7

Histria da Matemtica o a

Quando chegou a Pscoa, Johann havia recebido cinco solues. Uma era dele a co prprio e outra dada por Leibniz. Jakob tambm apresentou sua soluo, para o e ca poss embarao de seu orgulhoso irmo, assim como o fez o jovem Marqus vel c a e de lHpital. No entanto, mais uma resposta correta chegara da Inglaterra, o porm sem nenhuma assinatura. A braquistcrona nada mais do que parte da e o e ciclide, curva conhecida e estudada por Pascal, Galileu e outros. o Conta a lenda que, ao abrir a soluo enviada por uma carta annima de Londres, ca o Johann, entre surpreso e embaraado, disse: pela pata se reconhece o leo! c a
Para saber mais! Se voc e quiser saber mais detalhes sobre o problema da braquistcrona, h o a informao, por exemplo, no ca site www.icms.sc.usp.br/ szani/bra/bra.html, da pgina da Universidade de a So Paulo, em So Carlos. a a

Chegamos aqui ao matemtico que, por sua inventividade, a constitui o tema da unidade.

Texto 25: Euler O Gnio do Sculo e e


Nesse momento histrico, to rico em atividade matemtica e de muita como a a petio, nasceu Leonhard Euler, na Basilia, Suca. Seu pai, Paul Euler, era ca e pastor luterano e havia sido aluno de Jakob Bernoulli e dera ao lho suas primeiras lies. Devido a essa conexo com a fam Bernoulli, Euler foi aluno co a lia de Johann Bernoulli e manteve amizade por toda a vida com seus lhos, Daniel e Nicolau, ambos matemticos. a Euler dividiu sua vida prossional entre duas instituies: a Academia de Cincia co e de So Petersburgo, na Rssia, e a Academia de Cincia de Berlim. Ele ocupou, a u e em 1727, a cadeira de Medicina na Academia de Cincia de So Petersburgo, e a que fora fundada dois anos antes por Catarina I, esposa de Pedro, o Grande, Czar da Rssia. Alguns anos depois, ocupou a cadeira de Matemtica, que u a cara vaga quando Daniel Bernoulli mudou-se de volta para a Suca. Em 1741 mudou-se para Berlim, onde permaneceu at 1766. Leonhard Euler era um e homem simples e nunca se sentira ` vontade na corte de Frederico, o Grande, a patrono da Academia de Berlim, na qual brilhavam nomes como Voltaire, e as discusses loscas eram muito apreciadas. Assim que uma nova oportunidade o o surgiu, Euler retornou denitivamente para So Petersburgo. a Euler teve uma vida sem diculdades nanceiras, devido aos cargos que ocupou, e constituiu uma enorme fam lia. Foi perdendo a viso do olho direito ao longo a da dcada de 1730 e viveu seus ultimos 17 anos completamente cego. e 99
Leonhard Euler (1707-1783).

UD 7

Histria da Matemtica o a

Isso, no entanto, no o impediu de seguir produzindo Matemtica de qualia a dade e em quantidade at, virtualmente, seu ultimo suspiro. Dotado de uma e memria vast o ssima, sabia de cor, no s uma enormidade de nmeros primos, a o u mas tambm suas potncias, como 3376 , por exemplo. Alm disso, tinha uma e e e habilidade estupenda para executar mentalmente clculos extremamente elaboa rados. Suas obras completas ocupam mais de 70 volumes.

25.1 Algumas das contribuioes de Euler c


Euler contribuiu, de maneira decisiva em diversas reas da Matemtica, tais a a como geometria, clculo e teoria de nmeros. Foi o responsvel pela integrao a u a ca
O Problema dos Trs Corpos e consiste em encontrar os movimentos subseqentes de u trs corpos, determinados e pelas leis da mecnica a clssica (Leis de Gravitao, a ca de Newton), dadas as posies iniciais, massas e co velocidades.

das verses de Clculo dadas por Leibniz e por Newton, introduzindo muitas o a novidades, tais como os fatores de integrao das equaes diferenciais. Estudou ca co mecnica, considerando o chamado Problema dos Trs Corpos, em sua verso a e a mais simples. Isso , Euler considerava o problema de determinar o movimento e de um corpo de uma certa massa que se movimenta na presena do campo c gravitacional de duas outras massas que esto xas no espao. a c Apenas as contribuies na rea de Teoria de Nmeros seria suciente para gaco a u rantir a Euler lugar no panteo dos matemticos. Essas contribuies consistem a a co de provas de teoremas formulados por Fermat. As armaes de Fermat chegaram at Euler pelas cartas de Christian Goldco e bach, que o conhecera pessoalmente. Essas cartas aguaram a curiosidade de c Euler que se envolveu profundamente com a rea de Teoria de Nmeros. Por a u exemplo, Goldbach perguntou sobre a conjectura proposta por Fermat, de que os nmeros da forma 22 + 1 seriam primos. Euler concluiu que a armao u ca e falsa, mostrando que 232 + 1 = 4294967297 divis por 641. Se voc acha e vel e isso pouco, lembre-se que ele no dispunha de computadores. a Para ter uma idia do feito, tente fatorar 307007 usando apenas papel e lpis! e a Voltaremos a falar um pouco mais sobre esse tema na ultima unidade didtica. a Um dos grandes triunfos de Euler na Teoria de Nmeros a prova de que todo u e nmero perfeito par tem a forma descrita pelos antigos gregos, 2n1 (2n 1), u sempre que 2n 1 for um nmero primo. u Outro resultado interessante, provado por Euler, o chamado Pequeno Teorema e de Fermat. Esse teorema arma que, se a um nmero inteiro dado qualquer e u 100
n

UD 7

Histria da Matemtica o a

e o primo p no um de seus fatores, ento p um fator de ap1 1. Esse a e a e resultado , realmente, estupendo. Veja alguns exemplos: e

a 2 8 10 2

p 13 5 7 41

ap1 1 2131 1 851 1 1071 1 2411 1

decomposio em fatores primos ca 32 5 7 13 32 5 7 13 33 7 11 13 37 3 52 11 17 31 41 61681

Voc deve ter notado que as duas primeiras linhas representam o mesmo nmero. e u Resultados desse tipo ganharam, em nossos dias, um extra interesse devido a seu uso em criptograa. Voltaremos a falar sobre esse tema na ultima unidade didtica. a Euler tentou mostrar que todo nmero inteiro positivo a soma de quatro u e nmeros inteiros elevados ao quadrado n = a2 + b2 + c2 + d2 . Por exemplo, u 3 = 12 + 12 + 12 + 02 e 7 = 12 + 12 + 12 + 22 . E claro que no h a a unicidade nessa representao pois, por exemplo, 13 = 22 + 22 + 22 + 12 = ca 32 + 22 + 02 + 02 . Apesar de seus progressos, essa faanha seria deixada para c Joseph Louis Lagrange (1736 - 1813). Goldbach entrou para a histria da Matemtica com mais uma de suas perguno a tas. Em uma carta escrita em 7 de junho de 1724, pergunta a Euler se seria poss escrever qualquer nmero inteiro maior do que dois como a soma de vel u trs primos. Goldbach considerava o nmero 1 como um primo, coisa que no e u a fazemos mais. Euler reapresentou a pergunta da seguinte forma: seria poss vel escrever qualquer nmero inteiro par como a soma de dois nmeros primos? u u Por exemplo, 12 = 7 + 5, 14 = 11 + 3 e 1248 = 337 + 911. Essa questo continua desaando os praticantes de teoria de nmeros at esse a u e momento! 101

A criptograa estuda maneiras de codicar certos dados ou informaes para co que sejam decodicados apenas por pessoas espec cas. A criptograa e muito antiga, mas nos dias de hoje ela ganhou um papel ainda mais importante, devido ao desenvolvimento tecnolgico e da comunicao o ca por computadores.

UD 7

Histria da Matemtica o a

25.2 Somas innitas, mais uma vez . . .


Jakob Bernoulli havia provado que a srie harmnica diverge usando o fato de e o a soma dos inversos dos nmeros triangulares ser igual a 2: u 1+ 1 1 1 1 2 + + + + + . . . = 2. 3 6 10 15 k (k + 1)

O clculo dessa soma fora o primeiro triunfo de Leibniz. A prxima pergunta a o era, naturalmente, a soma dos inversos dos nmeros quadrados, u 1+ 1 1 1 1 + + + + ... 4 9 16 25

Nesse caso, o problema mostrou-se mais resistente. Veja, para cada inteiro positivo k, 1 1 < , 2 k (k + 1) k 2 Como a srie dos inversos dos nmeros triangulares converge, a srie dos inversos e u e dos nmeros quadrados tambm converge, uma vez que seu termo geral menor. u e e Voc se lembra do Teste da Comparao? A questo era descobrir e ca a o resultado da soma. Ao resolver esse problema, em 1734, Euler estabeleceu denitivamente sua reputao como matemtico genial. Ele comeou observando que a soma daria, ca a c aproximadamente, 1.6449. Chegar a essa aproximao sem usar um computador ca ou mesmo uma calculadora de bolso j um feito memorvel. No entanto, essa ae a aproximao no d qualquer indicao de qual seria, exatamente, o resultado. ca a a ca A soluo dada por Euler usou o fato de que a funo seno pode ser aproximada ca ca por polinmios: o sen x = x x3 x5 x7 + + ... 3! 5! 7!

Ele sabia que, se P (x) um polinmio tal que P (0) = 1 e a, b, c, . . . , d so e o a ra de P (x), ento podemos escrever zes a P (x) = 1 x a 1 x b 1 x c ... 1 x . d

102

UD 7

Histria da Matemtica o a

Atividade 29
Efetue 16 1 + x 2 1 x 4 x 1+ 2 x 1 . 2

e mostre que suas ra so 2, 4 e 2. zes a Euler observou que sen x x2 x4 x6 = 1 + + ... x 3! 5! 7!

f (x) =

tal que f (0) = 1 e deveria ter ra e zes , 2 , 3 e assim por diante. Portanto, f (x) = = 1 1 x x2 2 1+ 1 x x2 4 2 1 x 2 x2 9 2 1+ x 2 ... = ...

x2 25 2

O prximo passo consiste em expandir o produto da direita e obter o x2 x4 1 1 1 1 + . . . = 1 + + + +. . . 2 2 2 3! 5! 4 9 25 2 x2 + (. . . ) x4 . . .

A resposta est bem depois da prxima esquina! Comparando os coecientes a o de x2 , Euler concluiu que 1 1 1 1 1 = 2+ + + + ... 2 2 3! 4 9 25 2

e, portanto,

1+

1 1 1 2 + + + ... = , 4 9 25 6

um resultado realmente surpreendente. 103

UD 7

Histria da Matemtica o a

25.3 A aurora de uma nova teoria


A criatividade deu a Euler uma capacidade mpar de resolver problemas. Ele era capaz de ver muito alm do que estava ao alcance de seus contemporneos. e a Seu nome estar para sempre associado ao nascimento da Topologia, uma rea a a que s se desenvolveria plenamente no sculo 20, aps a introduo da Teoria o e o ca de Conjuntos, por Georg Cantor. Euler resolveu, de maneira absolutamente genial, um antigo problema (mais um) conhecido como o Problema das Pontes de Knigsberg. Essa cidade cava, no o in do sculo 18, na Prssia. Cortada pelo rio Pregel, que circunda uma ilha cio e u e divide-se numa bifurcao, as diferentes partes da cidade eram conectadas por ca sete pontes. O Problema das Pontes de Knigsberg consistia em determinar se o seria poss fazer um passeio completo pela cidade cruzando cada uma das vel sete pontes uma unica vez.

Concentrando-se na essncia do problema, Euler observou que a pergunta e e equivalente a saber se o diagrama a seguir poderia ser percorrido, indo de um vrtice a outro, percorrendo cada uma das arestas uma unica vez. e

Tais diagramas so conhecidos agora como grafos. Usando uma argumentao a ca simples, porm efetiva, estabeleceu um critrio matemtico que determina se e e a um tal diagrama pode ser percorrido dessa forma: passando por todas as arestas, percorrendo cada uma delas uma unica vez. Veja o Teorema de Euler: Teorema: Um grafo G admite um circuito euleriano se, e somente se, conexo e todos os vrtices tm grau par. e e e 104

UD 7

Histria da Matemtica o a

Basta observar que todos os vrtices do grafo das Pontes de Knigsberg tm e o e grau mpar, pois a cada um deles chega um nmero u mpar de arestas.

25.4 A equao ei + 1 = 0 ca
Os matemticos anteriores a Euler consideravam senos e cossenos como sega mentos de retas relativos a arcos de um c rculo de raio R. Foi ele que passou a considerar senos e cossenos como funes, exatamente como o fazemos hoje. co Com a denio de Euler, a identidade trigonomtrica fundamental ca e sen2 x + cos2 x = 1 imediata. e Usando a unidade imaginria i = a

1, mais uma novidade introduzida por

ele, deduziu a chamada Identidade de De Moivre, (cos x + i sen x)n = cos nx + i sen nx. Usando isso, deduziu as sries trigonomtricas e e cos x = 1 e sen x = x x3 x5 + ... 3! 5! x2 x4 + ... 2! 4!

Isso permitiu que estabelecesse a relao entre a funo exponencial e as funes ca ca co trigonomtricas: e cos x = eix + eix , 2 eix eix 2

sen x = e

eix = cos x + i sen x. Em particular, para x = , obtemos ei + 1 = 0. 105

UD 7

Histria da Matemtica o a

Leonhard Euler trabalhou at seu ultimo dia de vida. O planeta Urano havia e sido recentemente descoberto e ele estava interessado no clculo de sua rbita. a o Os problemas matemticos nunca deixaram de interessar-lhe. a Atuou em todas as reas da Matemtica que eram conhecidas em seus dias, a a contribuindo substancialmente para elas. No restringiu sua ateno apenas a a ca questes de natureza estritamente matemtica, mas dedicou-se tambm a o a e problemas da mecnica e de outras reas de natureza aplicada. Sua a a genialidade nos legou novos ramos da Matemtica, grandes teoremas e a diversos algoritmos, que enfatizam os aspectos aplicados e prticos da a Matemtica. Muitos deles so usados at hoje. a a e Euler fora contemporneo de Johann Sebastian Bach, por exemplo, e a possivelmente encontrara um de seus lhos, Karl Philip Emanuel, na corte de Frederico, o Grande, um dos dspotas iluminados da poca, que poderia e e discutir losoa com Voltaire ou executar ao violoncelo peas compostas c exclusivamente para ele. No entanto, quando morreu, em 1783, uma nova ordem social estava prestes a se instalar no mundo. Trs anos depois, em e 1786, estrearia em Viena a pera As Bodas de F o garo, de Wolfgang Amadeus Mozart, com libreto de Lorenzo da Ponte, baseada numa pea homnima de c o Beaumarchais. Um novo sculo estava por vir, trazendo muitas mudanas. A e c msica seria outra. A gura matemtica que dominaria a cena nesses dias de u a mudanas era Johann Carl Friedrich Gauss, que havia nascido em 1777. c As grandes conquistas feitas no per odo de vida de Euler estabeleceram um patamar jamais sonhado na Matemtica, mas estavam sujeitas a muitas a cr ticas. Era preciso uma gerao que se preocupasse com o rigor das teorias e ca que questionasse todos os seus fundamentos. Essa gerao j estava a ca a caminho e isso ser o pano de fundo de nossa prxima unidade didtica. a o a

106

Unidade 8
Construo dos Nmeros Reais Cauchy e Dedekind ca u
Nesta unidade didtica voc conhecer como a Matemtica a e a a se tornou, no decorrer do sculo 19, uma cincia mais abstrata e e e independente da realidade f sica. Ver como a atividade matemtica ganhou um carter mais a a a especializado, o que reetiu na diviso de reas, a a como a Geometria e a Algebra, em reas mais espec a cas, assim como no surgimento de novos campos de interesse.

Texto 26: Uma Longa Jornada Rumo ` Abstrao a ca


Nos dias de Euler, os problemas que ocupavam os matemticos provinham, a principalmente, da mecnica e da astronomia, alm das questes de Teoria de a e o Nmeros. u Os matemticos eram membros de academias cient a cas, raramente davam aulas e dedicavam todo o tempo `s pesquisas. A maioria deles escrevia seus trabalhos a em latim, a l ngua da cincia. e Essas caracter sticas estavam prestes a se alterar, assim como haveria mudanas c em outros setores da atividade humana, com a chegada do sculo 19. e Elas ocorreriam na ordem social, atravs de fenmenos como a Revoluo Frane o ca cesa (1789) e a Revoluo Industrial. Esta, iniciada nos meados do sculo 18, ca e na Inglaterra, espalhou-se por toda a Europa no decorrer do sculo 19 e chegou e ` Amrica, por exemplo: a e 107

UD 8

Histria da Matemtica o a

em 1807, Robert Fulton construiu o primeiro barco a vapor; em 1814, George Stephenson construiu a primeira locomotiva a vapor; em 1836, Samuel Morse inventou o telgrafo. e A eletricidade passou a ser usada como fonte de energia aps as descobertas o feitas por Georg Simon Ohm, sobre a corrente eltrica, em 1827, e a divulgao e ca das noes bsicas de eletromagnetismo por Michael Faraday, em 1831. co a As artes seriam palco de movimentos como o romantismo. Basta citar o poeta e cientista Johann Wolfgang von Goethe (1749 - 1832) e o compositor Ludwig van Beethoven (1770 - 1827), que viveram nesse per odo de transio. Na pintura, ca as obras de Joseph Turner (1775 - 1851) comeam a apresentar ind c cios de mudanas, que culminariam no movimento impressionista. c Tambm as cincias seriam afetadas. Em 1859 o naturalista ingls Charles e e e Darwin (1809 - 1882) abalou o mundo cient co com a publicao de A origem ca das espcies por meio da seleo natural. e ca Na Matemtica, os conceitos do Clculo careciam de uma fundamentao mais a a ca slida. Nesse sentido, as primeiras contribuies viriam de Gauss, com a noo o co ca de convergncia de sries, que seria levado a termo com a criao da Anlise e e ca a Matemtica, explicitada nos trabalhos de Cauchy, Weierstrass e Dedekind. a A Matemtica ganharia maior abstrao e especializao, e passaria a ser exa ca ca pressa na l ngua nativa dos matemticos, facilitando sua divulgao. a ca Voc j viu, na quarta unidade, como a questo do Quinto Postulado de Euclides e a a acabou gerando novos tipos de geometrias. Na Algebra, essa onda de abstrao ca tomou a forma da teoria de grupos e surgiram os primeiros exemplos de produtos no-comutativos, algo que antes era inconceb a vel. Os matemticos passariam a ocupar posies nas universidades e outras escolas, a co como academias militares, assumindo o papel de professores. A preocupao com o rigor no estabelecimento dos axiomas e denies, assim ca co como nas argumentaes, estaria mais em evidncia. co e E importante lembrar que nesse per odo, devido ` maior divulgao cient a ca ca e aos avanos sociais, as mulheres passaram a contribuir diretamente para a c Matemtica. a 108

UD 8

Histria da Matemtica o a

No entanto, tudo isso aconteceria aos poucos. Para entender melhor este per odo de transio, vamos falar da obra de um matemtico que o viveu, ca a mencionando algumas questes para as quais ele contribuiu. o

26.1 Um matemtico modesto Joseph-Louis Lagrange a


Em 12 de agosto de 1755, um jovem matemtico de Turino, na poca capital a e do reino da Sardenha, hoje na Itlia, enviou uma carta a Euler descrevendo os a resultados que obtivera sobre o problema da tautcrona, usando um mtodo de o e determinar mximos e m a nimos que desenvolvera. Muito bem, ele se chamava Joseph-Louis Lagrange (1736 - 1813) e o mtodo e estudado nos cursos de Clculo: multiplicadores de Lagrange. e a A tautcrona a curva sobre a qual o per o e odo de oscilao (desprezados o atrito ca e a perda de energia) no depende da amplitude do movimento. Em outras a palavras, se duas bolinhas forem soltas de diferentes pontos da tautcrona, o chegaro ao seu ponto mais baixo no mesmo intervalo de tempo. a Quando Euler retornou para So Petersburgo, Lagrange mudou-se para Berlim, a ocupando a direo da Academia de Cincias, em 1766. Ele se mudou mais ca e uma vez, em 1787, para Paris, onde permaneceu ativo at o m da vida. e Criou uma teoria sistemtica sobre as equaes diferenciais, incluindo um mtodo a co e de resoluo chamado variao de parmetros. Publicou, em 1788, o livro ca ca a Mcanique analytique (Mecnica anal e a tica). Em Rexions sur la rsolution e e algebrique des quations (Reexes sobre a resoluo algbrica das equaes) e o ca e co estudou a razo das equaes de grau at 4 poderem ser resolvidas por radicais. a co e Nesse trabalho considera as ra zes de uma equao como quantidades abstraca tas, em vez de terem valores numricos. Assim, foi o primeiro a estudar as e permutaes das ra co zes, o que levaria ao desenvolvimento da Teoria de Grupos, por Galois e Cauchy. Lagrange produziu resultados importantes em Teoria de Nmeros, como as u provas de que todo nmero inteiro positivo a soma de quatro nmeros inteiros u e u elevados ao quadrado (5 = 22 + 12 + 02 + 02 , 7 = 22 + 12 + 12 + 12 ) e, tambm, e o Teorema de Wilson, que arma: p primo se, e somente se, p divide (p 1)! + 1. e 109
Uma permutao de um ca conjunto ordenado X um e novo arranjo de seus elementos. Por exemplo, se X a tripla (1, 2, 3), h seis e a permutaoes. c A palavra tautcrona provm o e do grego tauto e quer dizer mesmo, como em tautologia, e cronos signica tempo, como em braquistcrona. o Bem, essa curva (surpresa!) e a ciclide e a carta o impressionou Euler, que a respondeu em poucos dias. Joseph-Louis Lagrange Lagrange trabalhou em problemas de astronomia, mecnica, mecnica dos a a uidos, probabilidade. Por exemplo, escreveu um trabalho explicando por que a lua exibe sempre a mesma face para a terra.

UD 8

Histria da Matemtica o a

Atividade 30
Encontre o raio e a altura do cilindro circular reto aberto de maior rea que a pode ser inscrito em uma esfera de raio r, usando multiplicadores de Lagrange.

Atividade 31
Verique o Teorema de Wilson para os nmeros 4, 5, 6 e 7. u

Atividade 32
Escreva 99 como a soma de quatro quadrados de trs maneiras diferentes. e Voc j sabe como foi importante a atuao de Carl Friedrich Gauss na e a ca resoluo dos trs clssicos problemas da Matemtica, apresentados na ca e a a primeira unidade didtica, alm de suas contribuies para a questo do a e co a Quinto Postulado de Euclides. No prximo texto, conhecer outros resultados associados a Gauss, o a um dos maiores matemticos de todos os tempos. a Uma de suas frases mais marcantes : e A Matemtica a rainha das cincias, e a Aritmtica, a rainha da a e e e Matemtica. a

Texto 27: Ligget se, disse o jovem Gauss

Nascido de pais muito simples, Gauss foi reconhecido como uma criana prod c gio e recebeu ajuda do Duque de Brunswick. Estudou na Universidade de Gttigen e o obteve seu doutorado pela Universidade de Helmstedt, no ano de 1799. De 1807 at e sua morte, em 1855, trabalhou como professor, astrnomo e diretor do o observatrio da Universidade o Gttigen. o

E folclrica a histria de como Gauss, ainda um garoto de dez anos, impressionou o o seu professor da escola primria. O professor, aparentemente, pretendia manter a os alunos ocupados por algum tempo e mandou-os somar os nmeros inteiros de u 1 at 100. Mal havia enunciado o problema e o jovem Gauss colocou sua lousa e sobre a mesa, dizendo ligget se , algo assim como aqui est [a resposta]. a O professor, evidentemente, no podia crer que Gauss tivesse a resposta correta, a mas para a sua surpresa, l estava: 5050. E claro que o padro de crescimento a a constante da srie de nmeros no passou despercebido ao menino. Ele notou e u a que a soma de elementos em posies simtricas constante: 1 + 100 = 101, co e e 110

UD 8

Histria da Matemtica o a

2 + 99 = 101, 3 + 98 = 101 etc. Como h 50 parcelas desse tipo, a 1 + 2 + 3 + 4 + + 97 + 98 + 99 + 100 = 50 101 = 5050.

27.1 O Teorema Fundamental da Algebra


Carl Friedrich Gauss foi, muito possivelmente, um dos maiores matemticos a de que temos not cia. Seus trabalhos conservam alguns traos da Matemtica c a do sculo 18: ele se dedicou tanto ` Matemtica pura como ` aplicada, viveu e a a a relativamente isolado, ocupava-se de problemas ligados ` astronomia e escrevia a os trabalhos em latim. O escopo de sua obra imenso, apesar do nmero e u limitado de publicaes. Lembremo-nos de um de seus motos: pouca sed matura co (pouco, porm maduro). e Gauss tinha uma enorme preocupao com o rigor, uma das caracter ca sticas que marcaria a Matemtica do sculo 19. a e Em sua juventude, provaria muitos resultados que j haviam sido descobertos a anteriormente, alm de alguns por ele mesmo. No entanto, suas demonstraes e co eram, em geral, mais rigorosas e completas do que as de seus antecessores. Ele estudou as obras dos matemticos do passado e era bem cr a tico. Na verdade, a insatisfao com as argumentaes anteriores era uma fonte de motivao. ca co ca Sua preocupao com o rigor teve grande inuncia na atitude geral dos maca e temticos a partir de ento, devido ` sua reconhecida importncia. a a a a No dirio em que registrava suas descobertas, em 10 de julho de 1796, est a a escrito: EPHKA ! num = + + . Gauss acabara de provar que todo nmero inteiro positivo pode ser escrito como u a soma de trs nmeros triangulares. Isto , os nmeros 0, 1, 3, 6, 10, 15, . . . e u e u Por exemplo, 28 = 1 + 6 + 21. Essa descoberta foi to importante que ele a saudou com a arquimediana exa clamao: eureka! ca Sua estria na carreira matemtica no poderia ter sido mais impressionante: e a a aps um per o odo de perto de 2000 anos, algum descobre mais um pol e gono regular que pode ser constru com rgua e compasso: o heptadecgono, um do e a pol gono de 17 lados. A soluo desse problema equivalente a chegar ao ca e 111
Euler, Laplace, Lagrange e Legendre eram agraciados por Gauss com o elogio clarissimus, em latim, claro. e Newton, no entanto, era considerado sumus.

UD 8 seguinte resultado:
16 cos 2 17 = 1 + 17 + 34 2 17 + 2 17 + 3 17

Histria da Matemtica o a

34 2

17 2

34 + 2

17.

Em sua tese de doutorado ele demonstra o chamado Teorema Fundamental da Algebra, que pode ser apresentado como: Toda equao polinomial de grau n tem exatamente n ra comca zes plexas ou reais, coincidentes ou distintas. Na verdade, h outras formulaes equivalentes do teorema. Uma delas diz que a co toda equao polinomial de coecientes reais pode ser expressa como o produto ca de fatores lineares ou quadrticos (irredut a veis) com coecientes reais. Por exemplo, 3x5 x4 3x3 5x2 + 8x 2 = (3x 1)(x2 + 2x + 2)(x 1)2 . Note que x2 + 2x + 2 = x2 + 2x + 1 + 1 = (x + 1)2 + 1 irredut e vel, isto , e no tem ra reais, e o fator (x 1) tem multiplicidade 2. a zes Antes de Gauss, vrios matemticos haviam tentado demonstrar este teorema, a a como Jean dAlembert (1717 - 1783), que enunciou o teorema e tentou uma demonstrao, sem sucesso. ca Euler ( claro) mostrou a validade do teorema para polinmios de grau at 6 e e o e tentou uma demonstrao para o caso geral. ca Em sua dissertao, apresentada em 1799, Gauss critica as provas dadas anteca riormente. Sobre a prova de dAlembert, arma: uma prova rigorosa poderia ser constru nestas mesmas bases. A prova dada por Gauss de natureza toda e polgica e tambm pass de cr o e e vel tica, sob nosso ponto de vista. De qualquer forma, em 1816, Gauss publicou uma segunda prova, seguindo a abordagem dada por Euler. Essa prova correta. e Como era t pico, retornou ao tema e, ainda em 1816, apresentou uma terceira prova, que tambm de natureza topolgica. e e o

Atividade 33
Fatore (segundo o Teorema Fundamental da Algebra) os seguintes polinmios: o x2 1, x3 1, x3 + 1, x4 1, x4 + 1. 112

UD 8

Histria da Matemtica o a

27.2 Investigaes Aritmticas co e


Em 1801, Gauss publicou o livro Disquisitiones Arithmeticae (Investigaes co Aritmticas), um texto sobre Teoria de Nmeros que um marco na literatura e u e matemtica (escrito aos 24 anos), em que apresenta, de maneira organizada, a resultados obtidos por estudiosos como Fermat, Euler, Lagrange e Legendre, alm de novos e importantes resultados. e Gauss introduziu a noo de congruncia entre dois nmeros inteiros: sejam a, ca e u b e m nmeros inteiros diferentes de zero. Dizemos que a congruente a b u e mdulo m se m dividir a diferena a b, e escrevemos o c a b mod m. Por exemplo, 13 2 mod 11 e 25 1 mod 4. Esse conceito parece muito abstrato, ` primeira vista, mas todo mundo conhece e usa com desenvoltura a alguns exemplos. Veja, quem nunca brincou de par ou mpar, ou zerinho ou um, como as crianas dizem agora? Isto , cada nmero inteiro congruente c e u e mdulo 2 a 0 ou a 1, e isso divide os inteiros em dois tipos: pares e o mpares. Outro caso conhecido a maneira como contamos as horas do dia, tomando-as e mdulo 12 ou 24, ou os minutos, que tomamos mdulo 60. o o Usando essa linguagem, Gauss apresenta no Disquisitiones Arithmeticae o Pequeno Teorema de Fermat, Teorema de Wilson e outros resultados de forma organizada e sistemtica. a Po exemplo, a concluso do Pequeno Teorema de Fermat: se p primo e a a e e um inteiro qualquer, ento p divide (ap a), pode ser escrita usando a noo a ca de congruncia como ap = a mod p. e Gauss foi o primeiro matemtico a enfatizar a importncia da propriedade de a a fatorizao unica dos nmeros inteiros em fatores primos, conhecida como o ca u Teorema Fundamental da Aritmtica, que ele enunciou e provou explicitamente. e Esse fato j havia sido usado na demonstrao da innitude dos primos, por a ca Euclides. No in do Disquisitiones Arithmeticae Gauss aborda a questo da resoluo cio a ca de equaes do tipo co ax b mod m, chamadas congruncias lineares. e 113

UD 8

Histria da Matemtica o a

As equaes lineares so as mais simples que podemos esperar. Por exemplo, co a em R, a equao ca a x = b, a = 0, sempre tem soluo, que unica: x = ca e b . a

No entanto, no caso das congruncias lineares, mesmo essas equaes podem e co nos reservar algumas surpresas. Como exemplo, vamos analisar as equaes co 2 x b mod 6, para diferentes valores de b. Devido `s congruncias, basta tomarmos b = 0, 1, 2, 3, 4 ou 5, os poss a e veis restos de divises por 6. o Muito bem, se b = 1, 3 ou 5, a equao 2 x b mod 6 no tem soluo, como ca a ca voc pode testar por inspeo direta, para x = 0, 1, 2, 3, 4 ou 5. Contudo, para e ca os casos de b = 0, 2 ou 4, a equao ter duas solues. Veja: 2 x 4 mod 6 ca a co tem solues 2 e 5, pois 2 2 = 4 e 2 5 = 10 = 4 + 6 4 mod 6. co O estudo das congruncias lineares a x b mod m se resume ` anlise do e a a mximo divisor comum de a e m, denotado por d = (a, m). A equao ter a ca a soluo apenas no caso de d dividir b: d | b. Alm disso, se d | b, o nmero de ca e u solues ser d. co a No exemplo 2 x b mod 6, o mximo divisor comum de 2 e 6 2 (d = 2). a e Assim, como 2 no divide 1, 3 ou 5, a equao no tem soluo no caso de b a ca a ca ser congruente mdulo 6 a algum desses nmeros. Em contrapartida, 2 divide o u 0, 2 e 4 e a equao tem duas solues distintas sempre que b for congruente ca co mdulo 6 a algum desses nmeros. o u

Atividade 34
Determine todas as poss veis solues de cada uma das congruncias lineares a co e seguir: 4x 6 mod 18; 4x = 8 mod 12; 2 x = 7 mod 13.

Equaes lineares com mais de uma soluo indicam que h algo de novo no co ca a ar. Imagine, ento, lidar com congruncias quadrticas. Resolver esse tipo de a e a 114

UD 8

Histria da Matemtica o a

equao a novidade do livro: a Lei da Reciprocidade Quadrtica, chamado ca e a Theorema Aureum, por Gauss. Esse resultado j havia sido descoberto por a Euler e Legendre, mas ambos falharam em suas demonstraes. Gauss tinha co fascinao por esse teorema, para o qual deu diversas demonstraes, ao longo ca co de sua vida. A questo se resume em lidar com equaes do tipo a co x2 a mod m. Se a equao tem soluo, dizemos que a um res ca ca e duo quadrtico de m. Por a exemplo, 1 e 1 so res a duos quadrticos de 5, uma vez que 42 = 16 = 53+1 a e 22 = 4 = 5 1 1. Como comum nas questes de Teoria de Nmeros, o problema pode ser e o u reduzido aos nmeros primos. u Dados dois nmeros primos p e q, a Lei da Reciprocidade Quadrtica diz que u a as congruncias x2 p mod q e x2 q mod p so ambas solveis ou ambas e a u insolveis, no caso em que p e q sejam congruentes a 1 mod 4. No caso de u p e q serem congruentes a 3 mod 4, uma das equaes ter soluo e a outra co a ca no. H uma formulao muito sinttica desse teorema usando o s a a ca e mbolo de Legendre. (Veja, por exemplo, o livro Introduo ` Teoria dos Nmeros, de J. ca a u P. de Oliveira Santos, da Coleo Matemtica Universitria, IMPA). ca a a
Os poss veis restos da diviso por 4 so 0, 1, 2 e 3. a a Portanto, h quatro classes a de congruncia mdulo 4. e o Assim, os n meros u mpares se dividem em duas classes, aqueles que so da forma a 4k + 1 so congruentes a 1 a mod 4; os da forma 4l + 3 so congruentes a 3 mod 4. a

27.3 Outras contribuioes de Gauss c


Aps a publicao do Disquisitiones Arithmeticae, Gauss voltou-se para questes o ca o de natureza mais aplicada. Em 1 de janeiro de 1801, ano da publicao desse ca livro, o astrnomo Giuseppe Piazzi descobriu um planetide que denominou o o Ceres, mas s conseguiu observar 9 graus da rbita, antes que ele desaparecesse o o atrs do sol. a Usando essas informaes, vrios astrnomos calcularam sua rbita e zeram co a o o previses sobre onde ele reapareceria. A previso de Gauss era muito diferente o a das outras e (quem diria?) foi conrmada, em 7 de dezembro de 1801. Apesar de no divulgar como zera os clculos, ele havia usado uma tcnica que a a e desenvolvera, chamada mtodo dos m e nimos quadrados. Seu segundo livro, Theoria motus corporum coelestium in sectionibus conicis so115

UD 8

Histria da Matemtica o a

lem ambientium (Teoria do movimento dos corpos celestes por sees cnicas), co o publicado em 1809, um tratado sobre os movimentos dos corpos celestes. e Gauss tinha um grande interesse em Geometria Diferencial, devido a seus trabalhos com geodsicas. Sua maior contribuio a esse campo foi a obra Disquie ca sitiones generales circa supercies curva, de 1828, em que aparecem a noo de ca curvatura gaussiana e o famoso Teorema Egregrium (Teorema Notvel). a O teorema arma que a curvatura de uma superf uma propriedade intr cie e nseca a ela. Isso quer dizer que poss determinar a curvatura de uma superf e vel cie, isto , como ela se curva dentro do espao tridimensional no qual est mergue c a lhada, usando apenas medidas de ngulos e distncias na prpria superf a a o cie. O teorema notvel porque a denio da curvatura gaussiana faz uso da funo e a ca ca (mergulho) da superf no espao tridimensional. cie c A partir de 1831, com a chegada a Gttingen do f o sico Wilhelm Weber, Gauss interessou-se pelas pesquisas sobre a teoria do campo magntico terrestre e e escreveu vrios trabalhos sobre o assunto. a

27.4 Matemtica a distncia a a


Gauss manteve correspondncia com vrios matemticos, entre eles Gotthold e a a Eisenstein (1823 - 1852), que o visitou em Gttingen, por duas semanas em o junho de 1844. Gauss tinha grande admirao por Eisenstein, que lamentavelca mente teve uma carreira muito breve. Alm de Eisenstein, Gauss correspondeu-se por um bom tempo com um jovem e matemtico francs, Monsieur Le Blanc. Esse era, no entanto, o pseudnimo a e o de Marie-Sophie Germain (1776 - 1831), uma jovem e talentosa matemtica a
Em 1830, graas aos esforos c c de Gauss, a Universidade de Gttingen concordou em o conceder a Sophie o doutorado, mas ela faleceu antes de receber a honraria.

francesa, que usava o nome Le Blanc para vencer as barreiras colocadas pela sociedade daquela poca. e Gauss soube a respeito de sua verdadeira identidade em 1806.

Dois dos mais famosos alunos de Gauss foram Bernhard Riemann e Richard Dedekind, de quem falaremos no prximo texto. o 116

UD 8

Histria da Matemtica o a

Texto 28: Cortes de Dedekind


Com a onda de rigor que percorria a Matemtica, fortemente inuenciada pelas a contribuies de Gauss, uma questo que mais uma vez se apresentava era a co a necessidade de estabelecer uma correspondncia denitiva entre os nmeros e e u a reta, estabelecendo em denitivo o que chamamos de conjunto dos nmeros u reais um desao quase to velho quanto a Matemtica. Os nmeros racionais a a u so mais fceis de estabelecer, a partir dos nmeros inteiros, e bastam para a a u o nosso dia-a-dia. No entanto, como a questo da no-racionalidade de 2 a a mostrou, eles no so sucientes para medir tudo. a a
Richard Dedekind (1831 - 1916)

A convergncia da srie e e prtico. a

1 2 , mostrada por Euler, por exemplo, para k2 6 pode ser usada para denirmos assim esse nmero, mas o procedimento no u a e
k=1

A formulao de uma teoria que colocasse um ponto nal sobre a questo da ca a existncia de nmeros irracionais (pelo menos do ponto de vista da Matemtica) e u a viria dos trabalhos de Dedekind, o ultimo aluno de Gauss. Ele se interessou por essa questo quando ensinou Clculo, pela primeira vez. A a a idia lhe ocorreu no dia 24 de novembro de 1858 e consiste em representar cada e nmero real como uma diviso, um corte nos nmeros racionais. Isto , todo u a u e nmero real r divide os nmeros racionais em duas partes distintas, os maiores u u e os menores do que ele. Um corte de Dedekind um par ordenado (A, B), no qual A e B so sube a conjuntos no-vazios de nmeros racionais, tais que A no possui elemento a u a mximo, a unio de A e B o conjunto de todos os racionais e, dados x A a a e e y B quaisquer, x < y. Essas idias foram publicadas, em 1872, no trabalho Stetigkeit und Irrationale e Zahlen (Continuidade e Nmeros Irracionais): u
Em cada caso em que h um corte (A1 , A2 ) que no produzido por a a e qualquer nmero racional, ento criamos um novo nmero a, irracional, u a u que ser considerado como completamente denido por este corte; diremos a que este nmero a corresponde a este corte, ou por ele produzido. u e

Alm dessa importante contribuio ` Matemtica, Dedekind nos legou a dee ca a a 117

UD 8

Histria da Matemtica o a

nio de conjuntos nito e innito, assim como trabalhos em Teoria de Nmeros. ca u Ele tambm introduziu a noo algbrica de ideal, que tem papel de destaque e ca e na teoria de anis, mais tarde desenvolvida por Hilbert e, posteriormente, por e Emmy Noether. Dedekind foi grande amigo de Georg Cantor, de cuja obra falaremos na prxima unidade didtica. o a

Atividade 35
Seja A = {x Q ; x2 < 2 } e seja B = {x Q ; x2 > 2 }. Mostre que (A, B) um corte de Dedekind. A qual nmero real esse corte corresponde? e u Cauchy louco e no h nada que se possa fazer sobre isso, apesar de, e a a atualmente, ele ser a unica pessoa a saber como Matemtica deve ser feita. a Niels Abel

Texto 29: Augustin Louis Cauchy


A frase de Niels Abel (1802 - 1829), um matemtico noruegus que conheceu a e Cauchy quando visitou Paris para tentar a sorte, o descreve bem, apesar de sua contundncia. e Nascido em Paris, no ano da Revoluo Francesa, Augustin Louis Cauchy (1789 ca - 1857) foi um matemtico que se dedicou ao ideal de rigor na Matemtica. a a
Augustin Louis Cauchy Cauchy era um dos responsveis pela aprovao a ca de um importante trabalho de Abel para a publicao. O ca trabalho havia sido submetido em 1826 e quando Abel faleceu, em abril de 1829, Cauchy ainda no havia dado a seu veredito.

Suas aulas de Clculo no faziam sucesso entre os alunos pois ele insistia em a a provar rigorosamente cada um dos teoremas que citava. Apesar dessa baixa popularidade, Cauchy legou enormes contribuies em co Anlise. Entre elas podemos citar uma primeira denio de continuidade de a ca funes (que seria colocada em termos dos famosos e psilon e delta por co e Weierstrass).

29.1 A denio de limite, segundo Cauchy ca


O ponto fraco de toda a teoria criada por Newton e Leibniz e posteriormente desenvolvida por Euler e tantos outros estava na falta de uma denio precisa ca do limite, to necessrio para lidar com as quantidades innitamente pequenas, a a seja l o que isso pudesse querer dizer. a 118

UD 8

Histria da Matemtica o a

Newton usava a noo de razo ultima de quantidades que desaparecem: ca a [. . . ] deve ser entendido como a razo ultima das quantidades, a no antes delas desaparecerem, no depois, mas aquela na qual a a elas desaparecem. Leibniz fala de quantidades innitamente pequenas. Isto , quantidades que, e apesar de no serem iguais a zero, no podem ser tomadas ainda menores. Algo a a como os tomos, da Qu a mica, suas quantidades innitamente pequenas eram como blocos indivis veis, a coisa mais prxima a zero. o E claro que esse tipo de denio dava margem a preocupaes e cr ca co ticas, sendo de George Berkeley as mais severas. Cauchy apresentou sua contribuio: ca
Quando os valores sucessivamente atribu dos a uma particular varivel a aproximarem indenidamente um valor xo, de forma que a diferena se c torne to pequena quanto se queira, essa ultima chamada limite de todas a e as outras.

Na verdade, esta denio apresentou imensos progressos, pois contm as idias ca e e principais do limite: a noo de proximidade e o famoso to pequeno quanto ca a se queira. Cauchy ainda nos legou o critrio de convergncia de seqncias, que leva seu e e ue nome, assim como uma extenso do Teorema do Valor Mdio. Suas contria e buies para a anlise complexa tambm foram profundas, como o Teorema de co a e Res duos. O chamado Teorema da Integral de Cauchy tem uma frmula muito o bonita: Se f (z) uma funo anal e ca tica denida em uma regio simplesa mente conexa R, ento a f (z) dz = 0

para qualquer contorno fechado completamente contido em R. Um matemtico que no , tambm, algo de poeta, a a e e nunca ser um perfeito matemtico. a a Karl Weierstrass 119

UD 8

Histria da Matemtica o a

Texto 30: Weierstrass Um Grande Professor

Karl Theodor Wilhelm Weierstrass (1815 - 1897), ao contrrio de muitos maa temticos, que comearam suas carreiras logo na juventude, iniciou sua produo a c ca cient ca relativamente tarde, por volta dos quarenta anos. O interesse por Matemtica ocorrera desde cedo, mas esse in a cio tardio foi devido, muito provavelmente, ` oposio que seu pai fazia para que ele se tora ca nasse matemtico. De qualquer forma, a Matemtica no perdeu por esperar. a a a
Weierstrass teve uma vida relativamente desregrada durante seus anos na universidade, em Bonn. Dos 26 anos at o momento que e sua primeira publicao lhe ca rendeu uma posio ca universitria, ele foi professor a de ensino mdio. e

Weierstrass foi um matemtico excelente. Por exemplo, em 1861 deu um exema plo de uma funo cont ca nua que contrariava todas as expectativas dos analistas matemticos. Veja, cada aluno de Clculo sabe que a funo f (x) = |x| um a a ca e exemplo de uma funo cont ca nua no diferencivel. Isso ocorre devido ao bico a a que seu grco apresenta, na origem. Ou seja, a funo no diferencivel apea ca a e a nas na origem. O exemplo de funo dado por Weierstrass, apesar de cont ca nua, no diferencivel em qualquer ponto de seu dom a e a nio. Tal funo obtida ca e como o limite de uma seqncia de funes que converge uniformemente. Isso ue co garante sua continuidade. A funo denida por ca e

f (x) =
n=1

sen k 2 x . k 2

O padro de rigor estabelecido por Weierstrass teve profundo efeito na Maa temtica. Ele deniu nmeros irracionais como limites de sries convergentes, a u e convergncia uniforme, funes denidas por produtos innitos, teste de cone co vergncia de sries e vrias outras coisas. Os alunos de Clculo o conhecem por e e a a sua genial tcnica para integrar funes racionais de sen x e cos x, chamada e co de arco metade, baseada na igualdade t = tg (x/2), que acarreta
Grco de f . a

sen x =

1 t2 2t e cos x = . 1 + t2 1 + t2

30.1 A denio de limite, segundo Weierstrass ca


A partir da abordagem de Cauchy, Weierstrass estabeleceu a denio de limite ca que todos aprendem em Clculo ou Anlise. a a 120

UD 8

Histria da Matemtica o a
Leia: para todo psilon maior e do que zero, existe delta maior do que zero tal que, se 0 < |x a| < , ento a |f (x) L| <

Ao dizermos limite de f (x) quando x tende a a L estamos dizendo: e > 0, > 0 tal que, 0 < |x a| < = |f (x) L| < . Note: 0 < |x a| < quer dizer x pertence ao intervalo (a , a + ) mas diferente do prprio a. Analogamente, |f (x) L| < quer dizer e o f (x) pertence ao intervalo (L , L + ). Eis aqui a noo de distncia, to ca a a importante para se estabelecer o limite. A grande diferena entre a abordagem c de Cauchy e de Weierstrass que esta ultima simblica, tornando precisa e e o a noo to pequeno quanto se queira. O que anima a denio dada por ca a ca Weierstrass o s e mbolo . Para que a denio seja satisfeita, devemos mostrar ca que verdadeira para todo > 0, os grandes e os pequenos. e Quando a denio se cumpre, usamos a notao ca ca
xa

lim f (x) = L.

30.2 Alunos de Weierstrass


O nmero de alunos de Weierstrass grande e entre eles guram alguns dos u e maiores matemticos da nova gerao. S para citar uns poucos, temos Marius a ca o Sophus Lie, Gsta Mittag-Leer, Georg Frobenius, Hermann Minkowski, Hero mann Schwarz. Um deles, Felix Klein, trabalhou em geometrias no-euclidianas, a teoria de grupos e props o chamado Erlangen Programme, para classicar geoo metrias pelo correspondente grupo de simetrias. Klein particularmente conhee cido dos alunos de topologia pela sua famosa garrafa, um exemplo de superf cie fechada no orientvel. a a

Outros destacados estudantes foram Sonya Kovalevskaya, Georg Cantor. Kovalevskaya recebeu o doutorado da Universidade de Gttingen e uma o posio em Estocolmo com a ajuda do mestre. E famosa por suas ca contribuies na teoria das equaes diferenciais parciais. co co As inestimveis contribuies de Cantor, assim como de Hilbert, sero a co a abordadas na prxima unidade didtica. o a
Garrafa de Klein

121

Unidade 9
Teoria de Conjuntos e Nmeros Transnitos de Cantor u
Nesta unidade didtica voc conhecer como as contribuies de Cantor e de a e a co Hilbert ajudaram a moldar o desenvolvimento da Matemtica no sculo 20. a e Um velho conhecido dos matemticos mais uma vez ocuparia um lugar de a destaque nos seus problemas. Veja a frase de Hilbert: O innito! Nenhuma outra questo jamais moveu to profundamente o a a esp rito humano.

Texto 31: O Surgimento de uma Nova Matemtica a


O sculo 19 fora particularmente prop para a Matemtica. Desde os trae cio a balhos de Jean Baptiste Joseph Fourier (1768 - 1830), sobre a teoria do calor, que resultou na publicao de Thorie analytique de la chaleur (Teoria anal ca e tica do calor), em 1822, passando pela obra monumental de Gauss, dos trabalhos de Bolyai e Lobachevsky sobre geometria no-euclidiana, tudo o que se via era a novas idias e resultados. e Vieram ` luz contribuies para a Algebra, com os trabalhos de Evariste Galois a co (1813 - 1832) e de Niels Abel (1802 - 1829). O primeiro exemplo de um produto no comutativo foi dado em 1843, por a William Rowan Hamilton (1805 - 1865). Ele introduziu a noo de nmeros ca u quatrnios, que generaliza o conceito de nmeros complexos. e u A anlise matemtica foi estabelecida, de in com os trabalhos de Cauchy, que a a cio partira da noo de funo dada por Lagrange. Esse trabalho seria completado ca ca 123

UD 9

Histria da Matemtica o a

por Weierstrass e Riemann. Cauchy tambm desenvolveu a teoria das funes e co de uma varivel complexa. a Alm de Weierstrass, questes sobre os nmeros reais haviam recebido as cone o u tribuies de Dedekind. co Enm, tudo parecia estar assentado e resolvido. No entanto, muitas novidades ainda estavam por vir. As noes de estruturas algbricas dariam ` Algebra co e a um carter ainda mais abstrato. Essa noo no caria, no entanto, limitada a ca a ` Algebra. Surgiram a Anlise Funcional, a Teoria de Medida e a integral de a a Lebesgue. Topologia Algbrica e Diferencial surgiriam como fortes reas de e a pesquisa, alm de outras, como a que chamamos Sistemas Dinmicos. e a A comunidade se dividiria em debates, tomando posies sob nomes tais como co formalismo, intuicionismo e logicismo. Mas, antes que tudo isso tomasse forma, a comunidade matemtica teria que assistir ` criao da teoria de conjuntos. Os a a ca questionamentos que ela acarretaria marcariam, de maneira indelvel, a virada e do sculo 19 para o sculo 20. e e

31.1 A universalizao da Matemtica ca a


Um fenmeno importante marcou a Matemtica nas primeiras dcadas do sculo o a e e 20. Voc deve ter percebido como, ao longo da histria, a atividade matemtica e o a concentrou-se em algumas partes do mundo. Na poca do Renascimento, a e Itlia foi o palco dessa atividade que, com o tempo, mudou-se para a Frana a c e, depois, para a Alemanha. Nos ns do sculo 19, a atividade matemtica e a concentrava-se nos pa europeus, principalmente na Frana e Alemanha, e ses c era praticada, essencialmente, por matemticos. a Com a chegada do sculo 20, houve mudanas dramticas. Basta lembrar que e c a entre 1914 e 1945 ocorreram dois conitos de propores continentais. Como co conseqncia de todas essas transformaoes, a Matemtica ganhou um carter ue c a a mais universal. Por exemplo, os Estados Unidos da Amrica receberam cientistas e europeus, e muitos matemticos entre eles. Outros pa passaram a contribuir a ses com grandes nomes para a Matemtica, como o Japo e a Rssia. a a u Alm disso, por ser uma cincia bsica, a Matemtica pde se desenvolver com e e a a o facilidade em pa em desenvolvimento. Assim, ses India, Brasil, Mxico e pa e ses do leste europeu tambm colaboraram para o avano da Matemtica. e c a 124

UD 9

Histria da Matemtica o a

No prximo texto, voc conhecer as idias principais de Cantor a respeito do o e a e innito. A ferramenta que ele desenvolveu para lidar com essa questo cou a conhecida como a Teoria de Conjuntos. A teoria desenvolvida por Cantor encontrou muita resistncia antes de ser e aceita, mas acabou se impondo.

Texto 32: O Innito Contra-ataca


A teoria de conjuntos desempenha papel relevante na abordagem de questes o envolvendo noo de innito e surgiu, como assunto de pesquisa, por volta ca de 1874, criada por Georg Cantor, um dos matemticos mais originais de que a se tem not cia. Com o passar do tempo, tornou-se a linguagem com a qual expressamos a Matemtica. a No passado, antes do aparecimento da teoria de conjuntos, os matemticos lia davam com nmeros, guras geomtricas, equaes etc. A mesma coisa ocorre u e co hoje, com a diferena que, agora, os nmeros so elementos de conjuntos, as c u a guras geomtricas so vistas como subconjuntos do plano, e o prprio plano e a o considerado como um conjunto. As equaes estabelecem condies que deco co nem conjuntos. Podemos dizer que os objetos com que os matemticos lidam so conjuntos, a a seus elementos e suas propriedades, assim como com as relaes entre eles. co
Georg Cantor nasceu em 3 de maro de 1845,em So c a Petersburgo, lho de um casal de amantes da arte e da cultura. Cantor sabia tocar violino e, alm da e Matemtica, tinha um grande a interesse por losoa e por literatura. Formou-se na Univesidade de Berlim e passou sua vida como professor na Universidade de Halle.

E tudo isso comeou com Cantor. Como matemtico ele sempre foi genial, mas c a os trabalhos que antecederam a teoria de conjuntos no levavam a crer que a pudesse produzir matemtica de tamanha originalidade. Aqui est uma de suas a a frases mais bonitas: A essncia da Matemtica a sua liberdade. e a e Nenhuma nova teoria, porm, que carregue tanta originalidade, recebida sem e e resistncia. As idias de Cantor receberam oposio, principalmente de um dos e e ca l deres da comunidade matemtica daquela poca: Leopold Kronecker (1823 a e 1891). Kronecker era um homem rico, um grande matemtico, e chegara a uma posio a ca de destaque na Universidade de Berlim por seus prprios mritos. Por que, o e 125

UD 9

Histria da Matemtica o a

ento, fazia tanta oposio a Cantor? No cerne da disputa estava uma questo a ca a quase to antiga quanto a prpria Matemtica: nito versus innito. a o a Voc j viu como essa questo desempenhou papel importante em diferentes e a a per odos da histria. Basta lembrar de Zeno e seus paradoxos. o a O innito, porm, foi adotado e usado por matemticos geniais, como atestam e a os trabalhos de Eudoxo e Arquimedes, na Antigidade, e a obra de Newton e u Leibniz, que criaram o clculo diferencial e integral. a O prprio Gauss tinha reservas sobre o tema: o
[. . . ] Eu protesto sobretudo contra o uso de uma quantidade innita como algo completo, que em Matemtica nunca permitido. O Innito a e e apenas uma maneira de falar [. . . ]

Cantor estava disposto a considerar, por exemplo, o conjunto de todos os nmeros como um objeto matemtico leg u a timo. Assim entendemos a resistncia e que a teoria de conjuntos recebeu do segmento liderado por Kronecker. Era uma questo de princ a pios. Kronecker acreditava que a Matemtica deveria lidar com a um nmero nito de objetos e com procedimentos que envolvessem apenas um u nmero nito de passos. u A respeito desse fenmeno completamente novo, que distinguia no s entre o a o colees nitas e innitas, mas entre colees com innitos elementos, Cantor co co escreveria ao seu amigo matemtico Julius Dedekind a Vejo, mas no posso acreditar! a No prximo texto voc conhecer mais detalhes dessa histria. o e a o

Texto 33: Como contar innidades?


Quando queremos comparar duas classes de coisas, temos a tendncia de contar e o nmero de cada uma delas. Ao contarmos cinco dedos em cada mo, sabemos u a que temos o mesmo nmero de dedos em cada uma delas. Mas, essa no u a e a unica maneira de fazer isso. O chefe de uma tribo de nativos de certa ilha, que no dispe de nmeros para contar, pode, ainda assim, determinar se uma a o u fam maior ou menor do que uma fam rival. Basta que ele faa com que lia e lia c as duas fam lias se disponham em duas las paralelas. Se a cada membro de 126

UD 9

Histria da Matemtica o a

uma das fam corresponder um membro da outra, elas tm o mesmo nmero lias e u de elementos. Caso contrrio, o chefe poder perceber qual das duas maior. a a e Cantor explorou essa idia estabelecendo o seguinte: e
Dois conjuntos M e N so equivalentes se poss coloc-los, por ala e vel a guma lei, em tal relao um com o outro, de tal forma que a cada elemento ca de um deles corresponde um e somente um elemento do outro.

Usando a linguagem atual, dizemos que, nesse caso, os conjuntos M e N tm e a mesma cardinalidade. Por exemplo, observe que podemos estabelecer uma correspondncia um-a-um e entre os nmeros naturais e os nmeros pares, atravs da lei n 2n. u u e 1 2 2 4 3 6 4 8 5 10 6 12 ... ... ...

Cantor estabeleceu que todo conjunto cujos elementos podem ser colocados em correspondncia um-a-um com os nmeros naturais dito enumervel, tem a e u e a mesma cardinalidade do conjunto dos nmeros naturais, que ele denotou 0 . u Assim ele introduzia um novo cardinal, transnito.
O s mbolo 0 deve ser lido alef-zero. Alef ( ) a e primeira letra do alfabeto hebraico.

Atividade 36
Estabelea uma correspondncia um-a-um entre os conjuntos de nmeros nac e u turais e inteiros.

33.1 A enumerabilidade dos nmeros racionais u


Usando essa denio de cardinalidade, alguma luz pode ser lanada sobre o ca c Paradoxo de Galileu: Considere C1 e C2 dois c rculos concntricos, um com raio igual ao dobro do e outro. A circunferncia do c e rculo de raio maior o dobro da circunferncia e e do outro. Apesar disso, para cada ponto na circunferncia menor corresponde e um unico ponto da circunferncia maior, e vice-versa. Basta tomar, para cada e ponto P1 da circunferncia menor, o raio OP1 e prolong-lo at a circunferncia e a e e 127

UD 9

Histria da Matemtica o a

maior, determinando seu ponto correspondente P2 . Veja bem, apesar de uma circunferncia ser o dobro da outra, ambas so conjuntos de mesma cardinalie a dade.
P2 P1

O C1 C2

Este tema havia sido considerado por Bernard Bolzano (1781 - 1848), um telogo, lsofo e matemtico tcheco que acreditava no conceito de innito. o o a Foi ele quem estabeleceu a diferena entre as colees nitas e innitas. Uma c co coleo innita pode ser colocada em correspondncia um-a-um com uma de ca e suas partes prprias, como atesta o exemplo de correspondncia entre os natuo e rais e os pares. Isto, denitivamente, no ocorre com colees nitas. Por exemplo, o conjunto a co de nmeros {0, 1, 2, 3, 4, 5} no pode ser colocado em correspondncia um-au a e um com o subconjunto {1, 2, 3, 4, 5}. Podemos armar categoricamente: como estamos com apenas um nmero nito de nmeros, poss escrever uma lista u u e vel com todas as correspondncias entre esses conjuntos e constatar que nenhuma e delas um-a-um. E claro que nesse caso mais fcil contar. e e a Se o fato dos conjuntos de nmeros naturais e pares terem a mesma cardinaliu dade, apesar de um ser subconjunto prprio do outro, causa alguma surpresa, o mais surpresas ainda esto a caminho. a Os conjuntos de nmeros naturais, inteiros e racionais tm todos a mesma u e cardinalidade. Isto , a cardinalidade dos racionais 0 . e e O fato dos nmeros racionais serem densos no conjunto dos nmeros reais u u enquanto os nmeros naturais so isolados mostra como a intuio pode nos u a ca enganar, quando se trata de conjuntos innitos. Dizer que o conjunto dos nmeros racionais denso nos reais signica que to prximo quanto quisermos u e a o de qualquer nmero real h nmeros racionais. u a u Para estabelecer uma correspondncia um-a-um entre os naturais e os racionais e positivos, vamos coloc-los em la indiana. Basta organizar os racionais na a 128

UD 9

Histria da Matemtica o a

forma de uma tabela. Cada linha tem fraes com o mesmo numerador e cada co coluna tem fraes com o mesmo denominador. Na verdade, a tabela contm co e todos os nmeros racionais com repeties. Por exemplo, toda a diagonal u co principal formada por cpias do nmero 1. Ainda assim, podemos colocar os e o u elementos da tabela em correspondncia um-a-um com os nmeros naturais. e u
/ 1/2 / 1/4 / 1/3 1/5 = = | {{ {{ {{ {{ || { { { { || {{ {{ {{ {{ || {{ {{ {{ {{ }{ { }{ { ~| 2/1 2/3 2/5 2/2 2/4 > = = || {{ {{ {{ {{ | { { { { | { { { { || {{ {{ {{ {{  {{ }{{ {{ }{{ || { {{ {{ { }{{

1/1

3/1

3/2

4/1

. . .

}> }} }}  }}} }

4/2

= {{ {{ {{ {{

3/3

}> } }} }} }} }} } }} } } }} }} }} ~}} ~}} }} . . . . . . . .

4/3

{ {{ {{ { }{{

3/4

4/4

= {{ {{ {{ {{

3/5

4/5

|| || || ~||

..

33.2 A no-enumerabilidade dos nmeros reais a u


At 1874 parecia que os conjuntos se dividiam em dois tipos: nitos e innitos, e porm enumerveis, de cardinalidade 0 . Foi ento que Cantor publicou o artigo e a a Ueber eine Eigenschaft des Inbegries aller reellen algebraischen Zahlen (Sobre uma propriedade da coleo de todos os nmeros algbricos). Nesse trabalho ca u e Cantor mostra um conjunto innito no-enumervel. Ele mostrou que qualquer a a intervalo de nmeros reais, no importa de qual comprimento, no pode ser u a a colocado em correspondncia um-a-um com os nmeros naturais. e u Vamos provar os seguinte: Teorema: O intervalo (0, 1) de todos os nmeros reais entre 0 e 1 u no enumervel. a e a A prova se baseia no fato de que podemos representar cada nmero x (0, 1) u usando uma expanso em casas decimais. Assim, os racionais seriam aqueles a cuja expanso nita ou tem um padro de repetio, que chamamos d a e a ca zima 129

UD 9 peridica. Assim, o

Histria da Matemtica o a

1 12 1 = 0.25, = 0.118811881188 . . . = 0.1111111 . . . 4 101 9 Alguns nmeros racionais podem ter um padro de repetio longo, como u a ca 13 = 0.764705882352941176470588235294117647058823529411 . . . e evita17 mos coisas como 0.499999999999 . . . colocando 0.5 no lugar. No entanto, os nmeros irracionais so caracterizados por no apresentarem u a a qualquer padro de repetio em sua expanso decimal. Por exemplo, a ca a = 6 2 0.52359877559829887307710 . . . e = 0.70710678118654752440084 . . . . 2 A prova dada por Cantor consiste em supor que existe alguma correspondncia e um-a-um entre o conjunto dos nmeros naturais e o intervalo (0, 1) e notar que u isso implica uma armao absurda. Em linhas gerais, supondo que haja uma ca tal correspondncia, podemos colocar todos os nmeros do intervalo (0, 1) em e u uma la, escrevendo, por exemplo: N 1 2 3 4 . . . n . . . (0, 1) 0.3667346777 . . . 0.3667346777 . . . 0.2500000000 . . . 0.1231231231 . . . . . . 0.a1 a2 a3 a4 a5 . . . . . .

x1 x2 x3 x4

. . .

= = = =

xn = . . .

O absurdo que essa armao implica que existe elemento do intervalo (0, 1) ca e que no est listado. Realmente, basta considerar o elemento cuja expanso a a a decimal e 0.b1 b2 b3 b4 b5 . . . no qual b1 diferente do d e gito da primeira casa decimal do elemento x1 , digamos b1 = 2 = 3; b2 diferente do d e gito da segunda casa decimal do elemento x2 , digamos b2 = 7 = 5; e assim por diante, passando por bn = an . Resumindo, mudando cada elemento na diagonal principal, produzimos um 130

UD 9

Histria da Matemtica o a

elemento de (0, 1) que difere em pelo menos uma casa decimal de todos os elementos listados. Isso contradiz a armao de que todos os elementos de ca (0, 1) j estavam na lista. a

Atividade 37
Mostre que quaisquer dois intervalos (a, b) e (c, d) tm a mesma cardinalidade. e

Atividade 38
Mostre que o conjunto dos nmeros reais e o intervalo (0, 1) tm a mesma u e cardinalidade. Assim, o conjunto dos nmeros reais tem outra cardinalidade, diferente de u 0 . No entanto, Cantor no quis dizer que a cardinalidade dos reais 1 , pois a e no sabia se haveria algum conjunto com cardinalidade intermediria, entre 0 , a a a cardinalidade dos naturais e a cardinalidade dos reais. Chamou, ento, a a cardinalidade dos reais de c, signicando cont nuo. A oposio de Kronecker no foi o principal problema que a teoria de conjuntos ca a enfrentou. O mais grave foi notado pelo prprio Cantor, em 1899, quando ele o considerou qual seria o nmero de elementos do conjunto de todos os conjuntos. u Isto levaria a uma contradio, pois j havia sido provado que um conjunto tem ca a menos elementos do que o conjunto de suas partes. Em 1902, Bertrand Russell (1872 - 1970) e Ernest Zermelo (1871 - 1956) descobriram, independentemente, outro paradoxo que abalava novamente a teoria. Eles deniam o seguinte conjunto: A = {X X no um elemento de X } a e
Uma verso popular do a paradoxo proposto por Russell a seguinte. Em e uma cidade h um barbeiro a e os homens da cidade dividem-se em dois conjuntos. H os que fazem a a barba com o barbeiro e os que fazem a sua prpria o barba. Responda rpido: a em qual conjunto se encontra o barbeiro? c o c gtico. e o

As armaes A um elemento de A e A no um elemento de A levam co e a e a uma contradio. ca Apesar dessas diculdades, a teoria dos conjuntos chegara para car. Lgicos o e matemticos posteriormente deram suas contribuies para que tais cona co tradies fossem evitadas. Entre eles o prprio Russell, Zermelo e, tambm, co o e Adolf Fraenkel (1891 - 1965), cumprindo assim a profecia de Hilbert: 131

UD 9

Histria da Matemtica o a Ningum nos expulsar do para criado por Cantor. e a so

No prximo texto voc conhecer algumas contribuies ` Matemtica feitas o e a co a a por Hilbert, um criador de frases lapidares, como: A arte de fazer Matemtica consiste em encontrar aquele caso especial que a contm todos os germes da generalidade. e

Texto 34: Uma Lista de Problemas um sculo para resolve e los


A virada do sculo 18 para 19 trouxera muitas mudanas ao mundo, mas no e c a se compara com o que estaria por vir com a entrada do sculo 20. A msica era e u outra. Ouvia-se sinfonias de Mahler, o dodecafonismo de Schoenberg, Berg e Webern. Em Frana soaria a requintada msica de Debussy e Ravel. O mundo c u conheceria o jazz. Em 1913 estrearia em Paris o bal Le sacre du printemps (A e sagrao da primavera) de Stravinsky, gerando um escndalo sem precedentes ca a no mundo da msica. Na pintura, a arte de Pablo Picasso um bom exemplo u e de como a velocidade das mudanas passaria para uma escala vertiginiosa. O c Brasil assistiria ` Semana da Arte Moderna, em 1922. a A mesma coisa ocorreria na Matemtica, em que haveria uma exploso de a a atividade. Ao longo do sculo 20, ela chegou a n e veis nunca antes imaginados. Os interesses se multiplicaram gerando uma mir ade de novas reas de interesse. a A classicao geral das reas de pesquisa da AMS, a American Mathematica a cal Society, soma perto de 100, entre as quais: teoria de nmeros, geometria u algbrica, K-teoria, medida e integrao, equaes diferenciais ordinrias e pare ca co a ciais, anlise de Fourier, geometria diferencial, topologia algbrica, teoria de a e probabilidades e processos estocsticos, teoria de jogos, economia, biologia e a outras cincias naturais, educao matemtica. e ca a A classicao completa est acess em www.ams.org/msc. ca a vel Na virada do sculo, o matemtico mais proeminente era David Hilbert. e a Ele tinha interesse em vrias reas da Matemtica, especialmente sobre proa a a blemas de lgebra de polinmios. Por exemplo, provara que qualquer ideal a o
David Hilbert (1862 - 1943)

polinomial admite base nita. Alm disso, iniciara uma corrente de pensamento matemtico chamada fore a 132

UD 9

Histria da Matemtica o a

malismo e tinha esperana de que tudo na Matemtica poderia e deveria ser c a provado a partir de axiomas bsicos. a Em 1899 publicou o livro Grundlagen der Geometrie (Fundamentos da Geometria), uma exposio rigorosa da geometria euclidiana. ca No dia 08 de agosto de 1900, em Paris, Hilbert fez uma palestra no II Congresso Internacional de Matemtica em que props vinte e trs problemas de a o e diferentes reas da Matemtica. Ele acreditava que esses problemas norteariam a a o desenvolvimento da Matemtica ao longo do sculo que estava para comear. a e c Alguns foram resolvidos logo, mas muitos continuam a inspirar os matemticos a e permanecem sem soluo. ca Na apresentao dos problemas, Hilbert teria dito: ca
Se quisermos ter uma idia do desenvolvimento provvel do conhecimento e a matemtico no futuro imediato devemos fazer passar por nossas mentes as a questes no resolvidas e olhar os problemas que a cincia de hoje coloca o a e e cujas solues esperamos no futuro. co

O primeiro problema da lista cou conhecido como a Hiptese do Cont o nuo: h a alguma cardinalidade entre 0 e c, a cardinalidade dos nmeros reais? u A resposta veio em 1963-4, dada por Paul Cohen, nascido em 1934, no estado de New Jersey, Estados Unidos da Amrica. Ele mostrou que a Hiptese do e o Cont nuo formalmente imposs de ser decidida na teoria de conjuntos de e vel Zermelo-Fraenkel (a teoria de conjuntos que usamos no nosso dia-a-dia). O problema 7 pergunta se, dados os nmeros a, algbrico, e b, irracional, seria ab u e transcendente? O problema foi resolvido de maneira positiva simultaneamente por Aleksandr Gelfond (1906 - 1968), em 1934, e Theodor Schneider, nascido em 1911. No entanto, vrias questes continuam abertas. Uma das mais famosas provm a o e do problema nmero 8, sobre nmeros primos, conhecido como a Hiptese de u u o Riemann. Versa sobre a distribuio dos zeros de uma funo denida por ca ca Riemann, chamada (zeta). Assim como no caso da Conjectura de Poincar, e h um prmio de um milho de dlares, oferecido pelo Clay Institute, para quem a e a o resolv-la. e Em 2004, Xavier Gourdon vericou a Hiptese de Riemann at os primeiros dez o e trilhes de candidatos usando computadores. Apesar de toda essa evidncia, o e 133

UD 9

Histria da Matemtica o a

os matemticos reconhecero a Hiptese de Riemann como teorema apenas a a o quando uma prova for apresentada. A prxima e ultima unidade didtica tratar de nmeros primos e criptograa. o a a u Voc ver como resultados de natureza pura da Matemtica podem, de e a a repente, tornar-se de interesse aplicado. A busca de solues de problemas per si e a possibilidade de suas aplicaes co co tem mantido o grande interesse pela Matemtica. Isso deve continuar por a muito tempo.

134

Unidade 10
Nmeros e Codicao de Mensagens u ca
Agora trataremos de assuntos atuais: computadores, informao e cdigos. ca o Criptograa um assunto em que ocorre uma feliz conuncia do passado e e e do presente, dos aspectos puro e aplicado da Matemtica. E um bom exemplo a de por que as questes matemticas so importantes e continuam a atrair o o a a interesse e a energia de tantas pessoas.

Texto 35: A Matemtica `s Portas de um Novo Milnio a a e


Terminamos a unidade didtica anterior mencionando Hilbert e sua lista de 23 a problemas, que ele acreditava guiariam o desenvolvimento da Matemtica no a sculo 20. e Voc deve estar se perguntando: o que aconteceu com a Matemtica no sculo e a e que ndou? Teria algum matemtico to famoso quanto Hilbert elaborado uma a a lista de questes visando o sculo 21? o e Isso sem contar que inauguramos, tambm, um novo milnio. e e Pois bem, quanto ` primeira pergunta, podemos dizer que a Matemtica superou a a as mais audazes expectativas. Em termos de volume de produo, diversicao, ca ca criao de novas reas de interesse e atividades interdisciplinares, o crescimento ca a tem sido exponencial. A atividade de pesquisa continuou fort ssima por todo o sculo 20, mesmo nos e momentos mais dif ceis pelos quais a humanidade passou. A comunidade matemtica manteve-se em contato, trocando informaes sobre a co 135

UD 10

Histria da Matemtica o a

a maioria dos tpicos de pesquisa, at durante os per o e odos mais duros. Por exemplo, enquanto americanos e os, ento, soviticos se engalnhavam a e na chamada corrida pelo espao, matemticos desses pa mantinham suas c a ses relaes em bons termos. co E tradio na Matemtica que toda informao nova seja disponibilizada perca a ca mitindo que matemticos de diferentes partes do mundo desenvolvam seus poa tenciais completamente.
Teorema das Quatro Cores Quatro cores so sucientes a para colorir qualquer mapa plano, dividido em regies, de o tal forma que regies vizinhas o no partilhem a mesma cor. a Regies que s se tocam num o o ponto no so consideradas a a vizinhas.

Uma coisa que marcou denitivamente a atividade matemtica no sculo 20 foi a e o advento dos computadores, como o fez em todos os outros setores da atividade humana. Isso ocorreu diretamente, em certos casos. Um exemplo famoso a e demonstrao do Teorema das Quatro Cores, que s pode ser completada com ca o a ajuda deles.

Os computadores ampliaram imensamente nossa percepo matemtica. Gaca a nhamos amplos poderes computacionais, antes restritos a poucas e privilegiadas pessoas, como Euler e Gauss, alm de uma melhor visualizao de objetos mae ca temticos. E claro que isso apenas a ponta de um iceberg. a e Tambm cumprem importante papel na comunicao. Grupos de pesquisas e ca espalhados pelo mundo mantm-se em contato, trocando informaes o tempo e co todo. Isso reexo de uma outra caracter e stica marcante da Matemtica nos a nossos dias: ela tornou-se uma atividade bastante gregria. a Aqui entra o tema dessa unidade: a criptograa. Mais do que nunca, em nossos dias a informao tornou-se um bem valios ca ssimo. Trocar e acumular dados de modo seguro , a um s tempo, tarefa dif e relevante. e o cil Mas, antes de falarmos sobre esse assunto e sobre os temas de Matemtica a que esto a ele relacionados, voc precisa saber se h uma nova lista de a e a problemas para o prximo sculo. Anal de contas, h uma pergunta o e a a ser respondida.

35.1 O Ano Mundial da Matemtica a


A Unio Matemtica Internacional, em congresso realizado em 6 de maio de a a 1992, no Rio de Janeiro, props o ano 2000 como o Ano Mundial da Matemtica o a 136

UD 10 e deniu trs objetivos para o 2000AMM: e

Histria da Matemtica o a

indicao dos grandes desaos da Matemtica para o sculo 21; ca a e promulgao da Matemtica, pura e aplicada, como uma das principais chaves ca a para o desenvolvimento; reconhecimento da presena constante da Matemtica na sociedade de informao. c a ca

No lugar de uma lista com problemas, o encontro produziu o livro Mathematics: Frontiers and Perspectives (Matemtica: Fronteiras e Perspectivas), editado a por quatro expoentes da Matemtica: Vladimir Arnold, Michael Atiyah, Peter a Lax e Barry Mazur. Trata-se de uma coletnea de artigos em que matemticos a a de uma grande variedade de reas contribuem com suas impresses e expectaa o tivas. Um olhar sobre os colaboradores para a coletnea deixa a certeza de que a a Matemtica continuar a brilhar por todo o sculo e, depois, tambm. Alm a a e e e dos quatro editores, cooperaram para o livro nomes como Steve Smale e Peter Lax. Smale americano, ganhador da Medalha Fields de 1966, e escreveu o e artigo Mathematical problems for the next century (Problemas matemticos a para o prximo sculo), que contm uma lista de 18 problemas. Lax foi diretor do o e e Courant Institute, em New York, e tem interesse em Matemtica e Computao, a ca como podemos ver no t tulo de sua contribuio: Mathematics and computing ca (Matemtica e computao). a ca Apesar de sujeito a cr ticas, o livro uma prova da diversidade e vigor da e atividade matemtica. a Agora, enfocaremos especicamente criptograa.

Texto 36: Criptograa

O objetivo da criptograa obter mtodos de codicar uma mensagem de modo e e que apenas seu destinatrio possa interpret-la. a a Do ponto de vista matemtico, queremos um isomorsmo, uma maneira de a transformar uma mensagem que se pretende transmitir em algo ileg vel, mas 137

A palavra criptograa tem origem (surpresa!) grega. Cryptos signica secreto, oculto.

UD 10

Histria da Matemtica o a

que, ao chegar ao destinatrio, este possa transform-la de volta, na mensagem a a original. Chamamos cifrar o processo de transformar a mensagem original em um texto ileg e decodicar o processo reverso. Reservamos a palavra decifrar vel para signicar a descoberta do processo todo. Isto , estamos fazendo uma e distino quanto ` ao de decodicar uma mensagem cifrada por um mtodo ca a ca e criptogrco, sem descobrir como ele funciona globalmente, da ao de decifrar a ca o mtodo, completamente. e Um exemplo simples de cdigo consiste em permutar cada letra do alfabeto o usada na mensagem pela letra seguinte. Por exemplo, a frase A vida bela e seria escrita codicada como BWJEBFCFMB. O processo reverso daria AVIDAEBELA. Esse mtodo bastante simples e fcil de ser decifrado. Note, por e e a exemplo, que a mensagem cifrada comea com a letra B, que se repete mais c duas vezes, enquanto que F aparece, ao todo, duas vezes. E conhecido que A e a letra mais usada em portugus, o que nos faz crer que B o codicado de A. e e

36.1 A Cifra de Vigen`re e


Em 1586, o francs Blaise Vigen`re publicou um livro contendo um mtodo de e e e cifrar mensagens que cou conhecido pelo nome cifra de Vigen`re. e O progresso feito em relao aos mtodos anteriores consiste no fato de a ca e maneira de permutar as letras para gerar a mensagem cifrada varia de letra para a letra. A variao depende de uma palavra (ou frase) chave e h uma tabela conhecida ca a de todos que codicam segundo o mtodo. No in do processo o codicador e cio escolhe uma palavra chave e a usa na codicao. Para que a mensagem seja ca
Blaise Vigen`re (1523 - 1596) e Em funo de sua atividade ca diplomtica, comeou a se a c interessar por criptograa e props a tabela no livro o Trait des chires ou secr`tes e e mani`res dcrire (Tratado e e das cifras ou modos secretos de escrever).

decodicada, o destinatrio precisa da palavra chave. a Para dar um exemplo, precisamos da tabela. (Veja adiante.) Para codicar a frase A vida bela usaremos a palavra chave Roma. e O processo consiste em colocar sobre a mensagem a ser cifrada a palavra chave, repetindo-a sucessivamente, de modo que a cada letra da mensagem corresponda uma letra da chave. Observe o exemplo: R A O V M I A D R A O E M B A E R L O A

138

UD 10

Histria da Matemtica o a

Para codicar a letra A, usamos a linha 17 da tabela correspondente ` letra a R. A letra A se encontra na coluna 1. Portanto, A ser codicado como R. a Verique na coluna 1 da linha 17, na tabela. Para codicar a letra V usamos a linha 14, relativa ` letra O. A letra V est na a a coluna 22, corresponde ` letra J. Veja na coluna 22 da linha 14, na tabela. a Prosseguindo assim, obtemos a mensagem cifrada: RJUDRSNECO. Para decodicar a mensagem, dispomos a palavra chave sobre a mensagem cifrada e executamos o processo reverso. R R O J M U A D R R O S M N A E R C O O

Veja, a letra S, da mensagem cifrada, aparece sob a letra O. A linha correspondente ao O a 14, na qual o S se encontra na coluna 5. Na tabela, a coluna 5 e tem ao alto a letra E, que aparece acentuada na frase: A vida bela. e
A B C D E F G H I J K L M N O P Q R S T U V WX Y Z 1 2 3 4 5 6 7 8 9 10 11 12 13 14 15 16 17 18 19 20 21 22 23 24 25 26 B C D E F G H I J K L MN O P Q R S T U V WX Y Z A C D E F G H I J K L MN O P Q R S T U V WX Y Z A B D E F G H I J K L MN O P Q R S T U V WX Y Z A B C E F G H I J K L MN O P Q R S T U V WX Y Z A B C D F G H I J K L MN O P Q R S T U V WX Y Z A B C D E G H I J K L MN O P Q R S T U V WX Y Z A B C D E F H I J K L MN O P Q R S T U V WX Y Z A B C D E F G I J K L MN O P Q R S T U V WX Y Z A B C D E F G H J K L MN O P Q R S T U V WX Y Z A B C D E F G H I K L MN O P Q R S T U V WX Y Z A B C D E F G H I J L MN O P Q R S T U V WX Y Z A B C D E F G H I J K MN O P Q R S T U V WX Y Z A B C D E F G H I J K L N O P Q R S T U V WX Y Z A B C D E F G H I J K L M O P Q R S T U V WX Y Z A B C D E F G H I J K L MN P Q R S T U V WX Y Z A B C D E F G H I J K L MN O Q R S T U V WX Y Z A B C D E F G H I J K L MN O P R S T U V WX Y Z A B C D E F G H I J K L MN O P Q S T U V WX Y Z A B C D E F G H I J K L MN O P Q R T U V WX Y Z A B C D E F G H I J K L MN O P Q R S U V WX Y Z A B C D E F G H I J K L MN O P Q R S T V WX Y Z A B C D E F G H I J K L MN O P Q R S T U WX Y Z A B C D E F G H I J K L MN O P Q R S T U V X Y Z A B C D E F G H I J K L MN O P Q R S T U V W Y Z A B C D E F G H I J K L MN O P Q R S T U V WX Z A B C D E F G H I J K L MN O P Q R S T U V WX Y A B C D E F G H I J K L MN O P Q R S T U V WX Y Z

139

UD 10

Histria da Matemtica o a

Uma inconvenincia desse cdigo o fato de que a mensagem, assim como a e o e chave, devem chegar ao destinatrio. Isto , a Cifra de Vigen`re um exemplo a e e e de criptograa de chave simtrica. Apesar dessa caracter e stica, ela foi usada por muito tempo.

Charles Babbage (1791 1891) Projetou a Mquina a das Diferenas e a Mquina c a Anal tica. Eram calculadoras sosticadas, precursoras do computador.

Por volta de 1855, um ingls chamado Charles Babbage descobriu como quebrar e o cdigo de Vigen`re. Babbage foi um gnio, porm um tanto excntrico. Ele o e e e e tambm divisou a construo de um computador, em pleno sculo 19, contudo, e ca e no chegou a realiz-la. Babbage no divulgou sua descoberta de como decifrar a a a o cdigo de Vigen`re e, em 1863, o criptgrafo Friedrich Wilhelm Kasiski, ocial o e o da reserva do exrcito prussiano, publicou um mtodo que permitia quebrar o e e cdigo de Vigen`re. O princ desse mtodo consiste em descobrir a palavra o e pio e chave, comeando por determinar seu comprimento. c Que tal uma interrupo na leitura? Aqui est uma oportunidade para voc ca a e experimentar como funciona a decodicao do cdigo de Vigen`re. ca o e

Friedrich Wilhelm Kasiski nasceu em novembro de 1805 numa pequena cidade da Prssia ocidental. Seu u interesse por criptologia comeou durante a carreira c militar. Publicou Die Geheimschriften und die Dechirierkunst(As escritas secretas e a arte da decifrao) em 1863. ca

Atividade 39
Decodique a mensagem QZQVSQVQHWIJWOSFVUHS usando o cdigo de Vigen`re e a palavra chave ERDOS. o e O hngaro Paul Erds foi um dos mais singulares e brilhantes u o matemticos do sculo 20. a e Seu principal interesse era a Teoria de Nmeros e a frase que voc decifrou na u e atividade anterior, usando o nome dele como chave, era a saudao que ele ca
Paul Erds (1913 - 1996) o no tinha posio alguma em a ca universidades ou institutos de pesquisa e passou a vida viajando, visitando amigos matemticos com quem a trabalhava.

usava ao chegar para visitar algum de seus muitos colaboradores de pesquisa. Ele costumava dizer que um matemtico uma mquina de transformar caf a e a e em teoremas. Veja, agora, como foi importante o papel de outro matemtico na decifrao a ca de um verdadeiro enigma. 140

UD 10

Histria da Matemtica o a

Decifrando a Enigma
Voc deve estar se perguntando se haveria algum erro no t e tulo dessa seo. ca Bem, a resposta no. A Enigma era uma mquina de codicar. Depois das e a a descobertas de Kasiski e Babbage, os criptgrafos necessitavam de novas cifras. o Assim, foram inventadas mquinas criadoras de cdigos que desempenharam a o um papel importante, especialmente durante os vrios conitos vividos pela a humanidade na primeira metade do sculo 20. e Durante a Segunda Guerra Mundial, os alemes desenvolveram uma mquina a a de codicar que cou conhecida por Enigma. Quebrar o cdigo da Enigma o passou a ser uma questo literalmente de vida ou morte. Um matemtico a a ingls desempenhou, nesse episdio, papel fundamental. Ele se chamava Alan e o Turing. Turing foi convocado pelo governo britnico e trabalhou na quebra do cdigo a o aparentemente inviolvel gerado pela Enigma. Os esforos de mais de sete mil a c funcionrios, somados com as descobertas feitas por um criptlogo polons, a o e Marian Rejewski, de como funcionava uma verso primitiva da Enigma, e mais a a perspiccia de Turing permitiram uma proeza fenomenal que deu aos aliados a uma vantagem sem igual na virada do conito. E pena que Alan Turing no tenha recebido em vida as devidas homenagens. a Primeiro, devido ` natureza sigilosa da atividade, os trabalhos desse time de dea cifradores permaneceu em segredo por dcadas. Mas triste saber que Turing e e sofreu por intolerncia e preconceito da sociedade em que viveu, por ser homosa sexual declarado. As presses e humilhaes foram tantas que ele, deprimido, o co cometeu suic dio. O papel da Matemtica na produo de novas cifras e na atividade de a ca quebr-las passou a ser cada vez mais importante. a Teoria de Nmeros desempenha papel crucial num dos cdigos mais usados na u o transmisso de dados por computadores, chamado RSA. a
Alan Turing (1912 - 1954) Era professor no Kings College, Cambridge e trabalhava com teoria de mquinas, em projetos que a anteciparam os modernos computadores.

Texto 37: Nmeros Primos, de Novo... u


As questes t o picas de Teoria de Nmeros sempre exerceram um grande fasc u nio sobre matemticos amadores e prossionais, seja pela simplicidade de suas fora 141

UD 10

Histria da Matemtica o a

mulaes, seja pela diculdade que geralmente carregam. co Vamos passar rapidamente em revista alguns principais tpicos desse assunto. o Os gregos, em particular os pitagricos, estudaram os nmeros extensivamente o u
Nos Elementos de Euclides encontramos a prova de que h uma innidade de primos, a a primeira demonstrao por ca absurdo de que temos not cia e, ainda, que todo nmero da u forma 2n1 (2n 1), com 2n 1 primo, perfeito. e

e perceberam a importncia dos nmeros primos. Eles tinham um profundo a u interesse pelos nmeros perfeitos e pelos pares de nmeros amigveis. u u a Posteriormente, Eratstenes descobriu como determinar todos os nmeros prio u mos menores do que um certo nmero dado, usando o crivo de Eratstenes. u o Aqui esto os primos menores do que 49. a

/ 1 / 8 /// 15 /// 22 29 /// 36 43

2 / 9 /// 16 23 /// 30 37 /// 44

3 /// 10 17 // 24 / 31 // 38 / // 45 /

/ 4 11 /// 18 /// 25 /// 32 /// 39 /// 46

5 // 12 / 19 // 26 / // 33 / // 40 / 47

/ 6 13 // 20 / // 27 / // 34 / 41 // 48 /

7 // 14 / // 21 / // 28 / // 35 / // 42 / // 49 /

Novos progressos s ocorreram com os trabalhos de Fermat, no in do sculo o cio e 17. Fermat conhecia algum mtodo de fatorao, pois foi capaz de fatorar e ca 2027651281 = 44021 46061 em relativamente pouco tempo. Outra grande contribuio sua foi o Pequeno Teorema de Fermat. ca

37.1 Testes de primalidade


Uma questo de relevo a respeito de nmeros inteiros decidir se um dado a u e nmero primo ou no. Caso o nmero seja decompon u e a u vel, a questo seguinte a descobrir sua fatorao, algo substancialmente mais dif e ca cil. O resultado de Fermat importante porque d um critrio de primalidade. e a e Como exemplo da diculdade dessa questo, examinemos os nmeros Mn = a u 2n 1, conhecidos como nmeros de Mersenne. Uma condio necessria para u ca a que Mn seja primo que n seja primo. No entanto, M11 = 2047 = 23 89 e e um nmero composto, apesar de 11 ser primo. u O maior nmero primo conhecido (em fevereiro de 2005) o quadragsimo u e e segundo nmero de Mersenne primo, M25 964 951 , que tem 7 816 230 d u gitos. 142

UD 10

Histria da Matemtica o a Neste ponto da histria, entra em cena um de nossos o campees Leonhard Euler. o A abordagem usada por Euler para mostrar que 232 + 1 um nmero e u composto uma prova de que temos muito o que aprender e com as idias dos mestres do passado. e Veja qual foi a estratgia por ele usada. e

37.2 Euler e os nmeros primos u


Fermat acreditava que os nmeros da forma 22 + 1 seriam primos. Isso u e verdade para n = 1, 2, 3 e 4, onde temos 22 + 1 = 5, 17, 257 e 65 537, respectivamente, todos primos. Porm, 232 + 1 um pequeno gigante 4 294 967 297. e e Como voc j sabe, Euler demonstrou vrios resultados enunciados por Fermat, e a a inclusive o seu Pequeno Teorema. Note que Euler usou esse resultado para provar que 232 + 1 composto, contrariando a hiptese de Fermat. e o Euler comeou observando o seguinte: c Se a um nmero par e p um primo que no um fator de a mas e u e a e divide a2 + 1, ento p = 4k + 1, para algum inteiro k. a Por exemplo, seja a = 8. Ento, 82 + 1 = 65 = 5 13, tem fatores primos 5 e a 13. Em ambos os casos, p da forma 4k + 1. e Veja, a seguir, como Euler mostrou, usando o Pequeno Teorema de Fermat, a armao acima. ca O fato de p dividir a2 + 1, um nmero u mpar, indica que p e mpar. Agora, todo nmero u mpar da forma 4k + 3 ou 4k + 1. Portanto, para provar o resultado, e bastava mostrar que a possibilidade p = 4k + 3 no ocorre. a A demonstrao ser por absurdo. Suponhamos que p = 4k + 3. Como p ca a e primo, o Pequeno Teorema de Fermat garante que p divide ap a = a (ap1 1). Por hiptese, p no um fator de a. Conclu o a e mos que p divide ap1 1. Assim, p divide ap1 1 = a4k+2 1. Usaremos essa informao em breve. ca Por outro lado, a fatorao ca (a2 + 1) (a4k a4k2 + a4k4 + a4 a2 + 1) = a4k+2 + 1, 143
n n

UD 10

Histria da Matemtica o a

e o fato de que p divide, por hiptese, a2 + 1, garante que p divide a4k+2 + 1. o Reunimos, agora, essa informao com a anterior: ca Como p divide a4k+2 1 e a4k+2 + 1, tambm divide a diferena, ou seja: e c p divide (a4k+2 + 1) (a4k+2 1) = 2. Mas isso uma contradio, pois p e ca e mpar por hiptese. o Logo, p da forma 4k + 1, para algum k, como foi enunciado. e Para que voc ganhe um pouco mais de percepo desse fato, e ca tente fazer a atividade a seguir.

Atividade 40
Determine os fatores primos de 84 + 1 e mostre que eles so da forma 8k + 1. a Agora, vamos continuar com Euler que, prosseguindo, provou uma seqncia de ue resultados que culminou em: Se a par, p primo e p divide a32 + 1, ento p da forma 64k + 1. e e a e Armado dessa informao, ele atacou a questo de fatorar 232 +1. Os resultados ca a obtidos indicavam bons candidatos ` fatorao. Quais so os primos da forma a ca a 64k + 1? Isso ocorre para os casos de k = 3, 4, 7, 9 e 10, por exemplo, onde p = 193, 257, 449, 577 e 641, respectivamente. Os quatro primeiros primos no dividem 232 + 1. Mas quando Euler fez a conta a com 641, obteve a recompensa pelo trabalho: 232 + 1 = 4 294 967 297 = 641 6 700 417.

Uma das grandes contribuies de Euler para a co Teoria de Nmeros foi u perceber que certas tcnicas e de anlise matemtica a a poderiam ser usadas no estudo dos nmeros. u

37.3 Outras contribuies para a Teoria de Nmeros co u


Se nos surpreendemos com o fato de que a srie harmnica e o
1 n=1 n

diverge,

surpresa ainda maior vem do fato de que a soma dos inversos dos nmeros u primos 1 1 1 1 1 1 + + + + + + ... 2 3 5 7 11 13 144

UD 10 tambm diverge. e

Histria da Matemtica o a

Uma das questes que interessou as geraes de matemticos aps Euler foi a o co a o distribuio dos primos. Por exemplo, Dirichlet demonstrou em 1837 o seguinte ca teorema, conjecturado por Gauss: Se (a, b) = 1 (a e b so relativamente primos), ento a progresso a a a aritmtica a+b, 2a+b, 3a+b, . . . contm uma innidade de nmeros e e u primos. Esse resultado generaliza o teorema da innitude de primos. Veja algumas questes abertas a respeito de nmeros primos: o u h uma innidade de nmeros primos da forma n2 + 1? a u sempre h um primo entre n2 e (n + 1)2 ? a a seqncia de Fibonacci contm um nmero innito de nmeros primos? ue e u u h uma innidade de primos gmeos (como 11 e 13, 41 e 43)? a e A falta de um padro aparente de distribuio dos nmeros primos entre os a ca u nmeros inteiros indica a riqueza do tema. Gauss e Legendre foram pioneiros u ao tratarem desse assunto. Chamamos de (n) o nmero de primos menores ou iguais a n. u Por exemplo, (10) = 4, (20) = 8, (50) = 15. Legendre conjecturou que (n) n , ln (n) 1.0836

dando uma estimativa para (n). O nmero 1.0836 chamado de constante u e de Legendre. Gauss, por sua vez, acreditava que
n E admirvel o esforo que a c Gauss e Legendre despenderam. Devemos lembrar que eles no a dispunham de computadores ou sequer uma simples mquina de calcular. a

(n) Li(n) =
2

1 dx. ln x

145

UD 10

Histria da Matemtica o a

Estima-se que Gauss tenha contado todos os primos at trs milhes. Ou seja, e e o que ele tenha calculado (3 000 000). O 216 816-simo primo o nmero 2 999 999 e o seguinte 3 000 017. e e u e O que cou conhecido como Teorema dos Nmeros Primos u e
Uma rea que desperta muito a interesse atualmente consiste em encontrar formas de determinar se um dado nmero primo. H dois u e a tipos de testes, basicamente: testes probabil sticos em que se usam computadores; testes de natureza terica o como o teste de Lucas-Lehmer. Pode-se dizer que h grande a demanda por nmeros u primos.

lim

(n) = 1. n/ ln(n)

O Teorema dos Nmeros Primos foi demonstrado em 1896 pelo matemtico u a francs Jacques Hadamard (1865 - 1963) e, independentemente, por Charles e Jean Gustave de la Valle Poussin (1866 - 1962), matemtico belga. Parece e a que Teoria de Nmeros uma espcie de elixir da longa vida. u e e

Veja, agora, como os nmeros primos exercem papel importante na u criptograa RSA.

Texto 38: Criptograa RSA


A sigla RSA provm dos sobrenomes de Ronald L. Rivest, Adi Shamir e Leonard e Adleman, que trabalhavam no MIT (Massachusetts Institute of Technology) quando inventaram esse cdigo, em 1978. o RSA um exemplo de criptograa de chave assimtrica. Isto , h uma chave e e e a pblica, que todos podem conhecer, que serve para cifrar a mensagem. Porm, u e h uma chave privada, que usada para decodicar a mensagem. a e Veja como funciona, em linhas gerais: para cifrar uma mensagem utiliza-se um nmero n = p q, produto de dois primos. Essa a chave pblica. u e u A chave privada consiste dos dois fatores primos de n: p e q. Para decodicar a mensagem no basta conhecer n, necessrio usar seus fatores primos. Parece a e a estranho, no? Conhecendo n, sabemos, teoricamente, seus fatores primos. No a entanto, o funcionamento do mtodo se baseia no fato de que, para nmeros e u grandes (mais de 150 algarismos), com os mtodos atuais, imposs detere e vel minar p e q. No irnico que um mtodo deposite toda a sua ecincia na a e o e e incapacidade de se fazer alguma coisa teoricamente poss vel? 146

UD 10

Histria da Matemtica o a

H muita literatura dispon sobre esse fascinante aspecto da Matemtica a vel a atual. Assim chegamos ao m de nossa disciplina, mas no ao m da histria. a o Esperamos que no decorrer desse tempo que passamos juntos voc tenha e percebido a importncia das questes matemticas. a o a Lembre-se, as questes podem ser antigas, mas as idias so atemporais. o e a Conclu mos com uma frase memorvel de Godfrey Harold Hardy, um a matemtico do sculo 20, que, ao lado de John Edensor Littlewood e Srinivasa a e Ramanujan, atuou em Teoria de Nmeros. u A frase aparece num livro que ele escreveu, chamado A Apologia de um Matemtico. a Eu acredito que a realidade matemtica existe fora de ns, que a o nossa funo descobrir ou observ-la, e que os teoremas que ca e a provamos, e que descrevemos com grandiloqncia como nossas ue criaes, so simplesmente notas de nossas observaes. co a co

147

Complemente o Estudo
Filmes
A Matemtica no tem sido muito favorecida como tema de lmes. Mas, recena a temente trs lmes chamaram a ateno para temas matemticos. E verdade e ca a que eles servem mais para diverso do que para nos informar sobre fatos, mas a essa a principal razo para se fazer lmes, no mesmo? e a a e No entanto, eles podem ser uteis como ponto de partida para discusses sobre o temas de Matemtica. a

Nome do Filme: Enigma (2001)


Direo: Michael Apted ca

Sinopse: Em maro de 1943, a equipe de elite dos decodicadores da Inglaterra c


tem uma responsabilidade monumental: decifrar o Enigma, um cdigo ultra o seguro utilizado pelos nazistas para enviar mensagens aos seus submarinos. Para liderar este trabalho chamado um gnio da matemtica que consegue realizar e e a tarefas consideradas imposs veis pelos especialistas.

Nome do Filme: Uma Mente Brilhante (A Beautiful Mind) (2001)


Direo: Ron Howard ca

Sinopse: John Nash um gnio da matemtica que, aos 21 anos, formulou um e e a


teorema genial. Aos poucos ele se transforma em um sofrido e atormentado homem, que chega a ser diagnosticado como esquizofrnico. Aps anos de luta e o para se recuperar, ele consegue retornar ` sociedade e acaba recebendo o Prmio a e Nobel de Economia em 1994.

Nome do Filme: Gnio Indomvel (Good Will Hunting) (1997) e a


Direo: Gus Van Sant ca

Sinopse: Em Boston, um jovem de 20 anos, servente de uma universidade,


revela-se um gnio em Matemtica. Por determinao legal, precisa fazer terae a ca 149

Histria da Matemtica o a pia, mas nada funciona, pois ele debocha de todos os analistas, at se identicar e com um deles. Neste lme voc ver, como cenrio, o MIT (Massachusetts Institute of Teche a a nology), um dos templos da cincia, e da Matemtica, em particular. e a

Livros
A literatura de divulgao de temas matemticos rica, mas a publicao em ca a e ca portugus no muito grande. No entanto, alguns livros se destacam. e a e

Nome do Livro: A Experincia Matemtica (1982) e a


Autores: Philip J. Davis e Reuben Hersh Esse livro aborda diversos tpicos e contm bastante informao. Alm disso, o e ca e pode ser lido por partes, uma vez que suas sees so independentes. co a Nome do Livro: O Ultimo Teorema de Fermat (1998) Autor: Simon Singh O livro conta, de maneira empolgante, toda a trajetria de um grande problema o livro para ser lido em um unico flego e relido, diversas vezes. de Matemtica. E a o

Nome do Livro: O Livro dos Cdigos (2001) o


Autor: Simon Singh Desta vez o autor narra a histria da Criptograa. o

Nome do Livro: Nmero: A Linguagem da Cincia u e


Autor: Tobias Dantzig Esse livro dif de achar pois est fora do prelo. Entretanto, se encontrado na e cil a biblioteca de algum amigo ou em alguma loja de livros usados pode dar muito prazer alm de muita informao. e ca

Sites
http://www-history.mcs.st-and.ac.uk/history/index.html. Este site sobre Histria da Matemtica excelente. o a e http://mathworld.wolfram.com Este site uma enciclopdia de Matemtica. Com a ferramenta de busca voc e e a e poder encontrar denies e teoremas sobre assuntos clssicos e atuais. a co a 150

Soluo de algumas atividades ca


Apresentamos, a seguir, a soluo para as atividades propostas no mdulo. No ca o a inclu mos todas por considerar que algumas no necessitam de gabarito. No a entanto, se voc tiver alguma dvida, tanto nas respostas apresentadas, como e u na resoluo das outras atividades, sugerimos que consulte a tutoria. ca Atividade 2 Como sabemos dividir um ngulo em dois, com rgua e compasso, podemos a e dizer que sabemos construir certas fam liasde pol gonos. Por exemplo, o tringulo, assim como o hexgono, o dodecgono, e assim por diante. Tambm a a a e podemos construir o quadrado, o octgono, e assim por diante. o 2 1 5 Como cos = + , podemos construir o pentgono usando rgua e a e 5 4 4 compasso, assim como o decgono, o dodecgono, e assim por diante. a a A lista parou por aqui, cerca de uns dois mil anos, at Gauss mostrar ser poss e vel construir, com rgua e compasso, o pol e gono de dezessete lados. Posteriorn + 1)-lados foram agregados ` lista dos que podem mente, os pol gonos com (4 a ser assim constru dos. Atividade 5 31 62 124 248 496 26 13 6 3 1

/ / /

Assim, segundo esse algoritmo, 31 26 = 62 + 248 + 496 = 806. Atividade 6 A tripla pitagrica gerada pelos nmeros u = 64 e v = 27 (3456, 3367, 4825). o u e Os nmeros 4601 e 6649 fazem parte da tripla pitagrica (4800, 4601, 6649), u o 151

gerada pelos nmeros u = 32 = 25 e v = 75 = 3 52 . u Atividade 8 A decomposio em fatores primos de 12 285 33 5 7 13. A soma de ca e seus divisores prprios 14 595, o outro elemento do par de nmeros amigveis: o e u a (12 285, 14 595). Atividade 10 1+ 5 = 1+ 2 1 1+ 1+ 1+ 1 1 1 1 + ...
34 21

Por exemplo, 1 + 1/(1 + 1/(1 + 1/(1 + 1/(1 + 1/(1 + 1/(1 + 1)))))) = 1.619047619. Uma boa aproximao de ca Atividade 11 No existe inteiro n tal que a 1 = 0. 2n =
1+ 5 2

1.618033989. e

Sabemos que 1 + r + r2 + r3 + . . . rn

1 rn+1 . Se |r| < 1, ento a 1r

rn =
i=0

1 . 1r

Atividade 12

Por exemplo, se m < 2 n, ento m 2 < 2 n. Basta multiplicar a primeira a desigualdade por 2. Os gregos do tempo de Eudoxo no dispunham desse a poss comprovar a veracidade da armao usando um arguargumento. E vel ca mento geomtrico. Veja, um quadrado de lado n 2 tem diagonal 2 n. Agora, e se m < 2 n, podemos desenhar um quadrado de lado m contido no quadrado de lado 2 n e sua diagonal m 2 menor do que a diagonal do quadrado e maior, que 2 n. e Atividade 14 K= . 4 Atividade 15 Os fatores primos de 3 5 7 + 1 so 2 e 53, primos diferentes de 3, 5 e 7. a Atividade 17 152

Conhecendo o volume v e a massa m da coroa, assim como as constantes e , podemos resolver a equao em x e descobrir quanto ouro e prata h nela ca a 3 e = 10 500kg/m3 , descubra o sem destru -la. Sabendo que = 19 300kg/m peso do ouro no caso da coroa ter volume v = 0.15m3 . Atividade 18 A prxima linha 21, 23, 25, 27 e 29. Note que 21+23+25+27+29 = 125 = 53 . o e Atividade 19 64 = (32 +22 ) (12 +22 ) = (3122)2 +(32 21)2 = 49+16 = 1+64. Atividade 20 Nem todas as propriedades das somas nitas so verdadeiras no caso das somas a innitas, como o caso da propriedade distributiva. e Atividade 22

1.3688

Note que a hiprbole (x + 2) (y + 10) = 40 contm os pontos (2, 0) e (0, 10). e e A projeo do ponto comum sobre o eixo x ocorre entre 1.3 e 1.4. ca Atividade 23 Colocando numa equao, temos x = x/6 + x/12 + x/7 + 5 + x/2 + 4. A ca soluo x = 84. ca e Atividade 24 A substituio x = y ca 3 + 26 y = 704 . y 3 27 Fazendo 3st =
26 3 2 3

transforma a equao x3 + 2x2 + 10x = 20 em ca

e s3 t3 =

cuja soluo positiva ca Sabemos que s t e

3 704 a ca 6 704 3 26 27 chegamos ` equao t + 27 t 272 = 0, t3 = 352+6 3930 . Assim, s3 = 704 + t3 = 352+6 3930 . e 27 27 27 3 + 26 y = 704 . Assim, soluo de y ca 3 27 3

y= Finalmente, x = y
2 3

352 + 6 3930 + 3

352 6 3930

leva ` soluo indicada na atividade. a ca 153

Atividade 25 Fazendo de conta que voc no tenha notado que na equao x3 15x = 4, e a ca A = 15 < 0, aplicamos a tcnica de clculo das ra por radicais, fazendo e a zes 3 t3 = 4. Isso leva ` equao t6 + 4t3 + 125 = 0, cujas ra 3st = 15 e s a ca zes 3 = 2 121 = 2 11 1. so t a Agora, os problemas. Primeiro, a raiz negativa. Se voc no conhece a teoria e a dos nmeros complexos, como era o caso de Cardano, pode seguir com a conta u usando essa resposta formal. A outra possibilidade que voc a tome como um e e 3 positivo, como zemos nmero complexo. De qualquer forma, como escolher t u antes, se t3 no mais um nmero real? Neste caso, escolhemos qualquer a e u 3 = 2 + 11 1. Isso nos d a s3 = 4 + t3 = 2 + 11 1 e um, digamos t a st = 3 2 + 11 1+ 3 2 11 1 uma estranha soluo de x3 15x = 4. e ca O mistrio se desfaz ao observarmos que (2 + 1)3 = 2 + 11 1, assim e como (2 1)3 = 2 11 1. Portanto, st=
3

2 + 11 1 +

2 11 1 = 2 + 11 1 + 2 11 1 = 4

apenas uma maneira um tanto diferente de escrever o nmero 4. e u Atividade 27 Uma sugesto consiste em arranjar a soma da maneira a seguir. a
1 2 + + 1 4 1 4 + + + 1 8 1 8 1 8 + + + + 1 16 1 16 1 16 1 16 + + + + + 1 32 1 32 1 32 1 32 1 32 + + + + + + 1 64 1 64 1 64 1 64 1 64 1 64 + ... + ... + ... + ... + ... + ... . . . + ... =

1 2

2 4

3 8

4 16

5 32

6 64

A resposta um nmero inteiro. e u Atividade 32 Aqui esto as trs maneiras de escrever 99 como uma soma de quatro quadrados: a e 99 = 1 + 1 + 16 + 81 = 1 + 9 + 25 + 64 = 9 + 16 + 25 + 49. 154

Voc sabia que podemos escrever 98 de quatro maneiras diferentes como soma e de quatro quadrados? Atividade 33 Essa interessante. Faa x4 + 1 = (x2 + ax + 1)(x2 ax + 1) e calcule a. e c Atividade 34 4x 6 mod 18 tem duas solues (residuais): 6 e 15; 4x 8 mod 12 tem co quatro solues: 2, 5, 8 e 11; 2x 7 mod 13 tem uma soluo. co ca Atividade 36 Por exemplo, 0, 1, 1, 2, 2, 3, 3, 4, 4, . . . . Atividade 37 A funo f (x) = ca (a, b) e (c, d). Atividade 38 A funo f (x) = tg ca (0, 1) e a reta real. Atividade 39 A linda frase que Paul Erds usava para saudar seus amigos e colegas pesquisadores o Minha mente est aberta. e a Atividade 40 84 + 1 = 4097 = 17 241. Note que 17 = 2 8 + 1 e 241 = 30 8 + 1. 2 x 1 2 estabelece uma bijeo entre o intervalo ca ad bc cd x+ estabelece uma bijeo entre os intervalos ca ab ab

155

Referncias e
ANGLIN, W. S. Mathematics: A Concise History and Philosophy. New York: Springer Verlag, 1994. BELL, E. T. Men of Mathematics. New York: Touchstone, 1965. BOYER, C. B. Histria da Matemtica. So Paulo: Edgar Blcher, 1974. o a a u COURANT, R. e ROBBINS, H. O que Matemtica? Rio de Janeiro: Cincia e a e Moderna, 2000. COUTINHO, S. C. Nmeros Inteiros e Criptograa RSA. Rio de Janeiro: IMPA u / SBM, 1997. DANTZIG, T. Nmero: A Linguagem da Cincia. Rio de Janeiro: Zahar, 1970 u e DAVIS, P. e HERSH, R. A Experincia Matemtica. Rio de Janeiro: Francisco e a Alves, 1982. DORRIE, H. 100 Great Problems of Elementary Mathematics: Their History and Solution. New York: Dover, 1965. EVES, H. Histria da Matemtica. So Paulo: UNICAMP, 1996. o a a GRIFFITHS, H. B. e HILTON, P. J. Matemtica Clssica Uma Interpretao a a ca Contempornea. So Paulo: Blcher / USP, 1975.3 v. a a u HEFEZ, A. Curso de Algebra. v. 1. Rio de Janeiro: IMPA, 1997. (Coleo ca Matemtica Universitria). a a SANTOS, J. P. de O. Introduo ` Teoria de Nmeros. Rio de Janeiro: IMPA, ca a u 1998. (Coleo Matemtica Universitria). ca a a SINGH, S. O Ultimo Teorema de Fermat. Rio de Janeiro: Record, 1998. SINGH, S. O Livro dos Cdigos. Rio de Janeiro: Record, 2001. o STRUIK, D. J. A Concise History of Mathematics. New York: Dover, 1987.

157

Vous aimerez peut-être aussi